You are on page 1of 84

‫ﺗﻢ ﺑﻔﻀﻞ ﷲ ﺗﺠﻤﯿﻊ اﻻﺳﺎﻟﺔ واﻻﺟﻮﺑﺔ اﻟﻤﺘﺸﺎﺑﮭﺔ ﻟﺘﺴﮭﯿﻞ ﺣﻔﻈﮭﺎ‬

‫ﻣﻠﻒ ﺷﺎﻣﻞ ﻟﻠﺒﯿﺪﯾﺎ ‪ ،‬ﻣﺼﺪر اﻻﺳﺎﻟﺔ ﺟﻠﻮري ﺗﯿﻢ ﻓﻘﻂ‪.‬‬


1Supportive management

🌹Child patient after eat steak has fever and abdomen pain what is the management?
A-Conservative ✅ B _antibiotic

🍇 3years old presented with watery diarrhea,cramps , dehydration after being exposed to colleague
with same presentation at day care , mx: === Supportive treatment
‫ اﻟواﺿﺢ اﻧﮫ‬، ‫ﯾﻌﻧﻲ طﻔل ﺗﻌرض ﻟﻠﻌدوى ﻻن زﻣﯾﻠﮫ ﻛﺎن ﻋﻧده اﺳﮭﺎل ﻓﺎﻻﺧﺗﯾﺎرات ﯾﻘوﻟك ﻛل اﻧواع اﻟﻣﺿﺎدات اﻟﺣﯾوﯾﺔ و ﻻ ﺑس ﻋﻼج دﻋم‬
supportive care ‫ﻓﺎﯾرس ﻓﺧﻼص ﺑس‬

🍇 4-year-old child presented with several episodes of passage of frequent non-bloody, watery stool and
vomiting. Management?
A. Metronidazole B. Doxycycline C. Amoxicillin. D. Supportive management✅✅

🍇 Child with Hx of eating in a restaurant with abd pain, nausea and vomiting, 10 days later bloody
diarrhea, urinalysis shows 10 RBCs , ttt?
A. Anti diarrhea medication. B. Metronidazole. C. Supportive therapy D. Antibiotics
…..
🍇 Child develops petechia in the legs, he has history of URTI. Labs showed: normal CBC except platelet
count was : low. What is the next step in management?
A. Platelettransfusion B. Immunoglobulin ( if sever bleeding like untracranial hemhhrage
C. Steroid ( if mild bleeding ) D. Supportive ( if plattlet more than 30.000 and there is no bleeding ) ✅✅
…..
🌹Child develops petechia in the legs, he has history of URTI. Labs showed: normal CBC except platelet
count was : low. What is the next step = Supportive 🌹
…..
🌹Child with watery diarrhea , history of sick contact = Supportive management 🌹
..
🌹5 years old boy has RUQ pain + normal bowel sounds ,lap show severe drop in hemoglobin and
increased reticulocytes count. Blood smear report: target cells and inclusion bodies. diagnosis?
Sickle cell disease ( Not appendicitis)

Child with a long history of watery diarrhea abdominal bloating and pain, what’s the Dx?
A. chronic giardiasis ✅ Can Giardia last for years? (chronic) giardiasis, which causes persistent or
repeated bouts of diarrhoea that can last for up to two years.
….
🌹treatment of dengue fever? Supportive and avoid NSAIDs🌹
….
🌹dengue mosquito time of activity?🌹 early morning ✅

🌹2-3years old presented with watery diarrhea,cramps , dehydration after being exposed to colleague
with same presentation at day care , mx:
A-Ciproflaxacin B-Metronidazole. C-Supportive treatment✅. D-Fourth antibiotic

Child with watery diarrhea bloodless management: is Supportive ‫ دم‬bd c ‫ ﻣﺎدام ﻣﺎ‬hd i ‫ ﻣﻨﻄ‬.
.....
🌹Rota virus: = reassure 🌹
......
Neonate with vaginal mucoid discharge and concerned mother what to tell her?
A-Reassure ✅ B-Take vaginal smear C-Canʼt recall other choices

Feverish child T 38, cough, Bilateral infiltrated lung. Nothing else mentioned. Management?
A- Reassure cuz viral ✅. B- Discharge and oral ABx C- Admission and IV ABx
……
🍓 8-month-old with asymmetrical breast enlargement and no other symptoms what is the cause =
reassure 🍓
🍓A child with birth weight 3.5kg now weighing 3.1. Breastfeed about 3 times every 15 minutes. Advise to
mum? A. Improve matemal nutrition B. Reassure that all is well✅ C. Admit for full work up
….
🌹Two weeks year old child suddenly had erythematous rash on his body (the child was not febrile and he
was stable) what to do = Assure mom.

🌹Mucus vaginal discharge in newborn baby: reassure the mother

🌹 Child presented to pre-diagnosis clinic with systolic ejection murmur , no sx ?
A. refer to cardio for cath B. reassure and discharge C.start ACE and
....
🌹 Child with 15 months, can only babble and his mother is afraid because he cannot say 2-3 word
sentences, hearing test done when he was 5 months old and was normal:
A. reassure as this is a normal variant B. re-evaluate at 24 months C. Refer to ENT
....
Child presents with fever, vomiting and diarrhea on exam of chest there is reduced air entry to right side
and murmur was heard , the child CVS and chest exams were previously normal (prior visits) what
is the management of murmur?
A. Urgent echo B. Reexamine after these symptoms subsides ✅✅✅ C. Refer to cardio pediatrician
....
🌹3 year are going for dental operation had murmur when he stand and disappear when sitting What to
do? A. Reassuring ✅ B. Reevaluate C. Cardio consultation
....

🌹 Asymptomatic child scheduled for a dental procedure, on examination found to


have a murmur that changes with position, what to do?
A. consult cardio
B. cancel procedure and for further testing
C. innocent murmur✅‫ ﻫﺬا اﺳﻢ اﻟﻤﺎرﻣﺮ وﺻﺢ‬+ no need to worry and cont✅‫ﺬا اﻟﻤﺎﻧﺞ‬P‫و‬

🌹 Child 3 years old, fell from bed, immediately cried afterwards, vomited twice, headache, physical
exam and neuro exam are normal: A- Brain MRI B- Brain CT D- Observation
🍒Child was brought to hospital with airway swelling and skin lesions all over the body the mother stated
that he was in a party at his friends house: Food allergy🍒
..
🌷If language delay > do hearing test.🌷

# what to be screen in neonates = metabolic diseases, HYPOTHYROIDISM, HEARING.

🍇Clear case of congenital prolonged QT syndrome Jervell and Larged-Neilson
Syndrome associated with : sensorineural hearing loss

🌹2 years old child can't talk probably and doesn't understand, he have decreased hearing, what 1st test
you want to do Hearing loss examination. ✅
..
Baby abdominal distended what is the first investigation = a. Xray. b. Ct. c. Us✅✅
….
Newborn what to examine?m. A- Hearing✅✅. B- Vision. C- Both ‫ اذﻧﻪ ﻗﺮان‬Š‫„ ﻧﻘﺮء ﻋ‬d … ‫ اول ﻣﺎ ﻳﻮﻟﺪ اﻟﺒﻴ‬:)
….
A child has kernicterus sign what you have expect that you tell his parents ?
ARetina detachment B.Blindness C.Hearing loos ✅
‫ = ر•ﻂ‬kids = ker = ‫•ﺎء‬œ‫”ء ﻳﺰﻋﺠﻨﺎ ﻓﻴﻬﻢ ﻫﻮ اﻟﺼﻮت واﻟ‬ • “
d ’‘‫ ا‬hearing .
..............
….
🌹15 month old , parents concerned about his development that only babbles and cant say 2-3 words yet,
he has normal auditory examination, what should you do? Reassurance and come at 24 months

🌹3 years old boy came with his mother, she's concerned he might have abnomial development. Ho goes
to day care, understand only 2 words command (order) and strangers can understand only 75% of his
talk. Your action? A. Reassure B. Refer ta ENT C. Delayed speech disorder ✅

🌹Newborn examinations = vision and hearing (vision for red eye reflex )

🌹Aspiration meconium treatment = Surfctant 🌹


….

🍓mom comes to your clinic with her neonate who is 1 week of age, she is worried because her neonate
lost 1.5 kg from his birth weight? reassure


🌹Infant with erythematous macules on erythematous base in the back and trunk?
Reassure

🌹case of watery diarrhea ( what is the electricity abnormalities will find ) ?
Answer is : HYPONATREMIA, HYPOKALEMIA, METABOLIC ACIDOSIS
‫ة |ﺎﻻﺳﻬﺎل‬sz‫ﺖ ﻛﺜ‬v‫وﻻ‬str‫ﺪ =ﻓﻘﺪ اﻟ‬n‫ﻂ =اﺳﻬﺎل =اﺳ‬i‫ر‬
.....
🌹Primimother worried about her 6week baby , he stepping all the formula since birth , what most likely:
A. HSD B. pyloric stenosis C. physiological reflex
...
🌹 And there was another Q near to the previous, ask about the next step :
A. burping the baby and semi setting while feeding
B. immediate call surgeon

🌹 9 year boy with growth pains, management? -Reassurance

infantile colic
...
🌹Pt with infantile colic scenario, mother mentioned (normal feeding) didn’t say if it was breastfeeding or
formula, treatment: The options were similar to something like this:
A. Change his milk formula because he might be allergic B. Colic drops C. Reassure mother that it’s normal
start on 6 weeks (N.B. Infantile colic onset is 6 weeks of age and goes away by 6 months of age)
.‫ ﺧﻣﺳﺔ = زودوا واﺣد وﯾﻛون ﺳت اﺳﺎﺑﯾﻊ‬colic = ‫رﺑط = ﻋدد ﺣروف‬
.....
🌹 Infantile colic mc occur:
A. First 6 weeks B. First 3 weeks C. First 3 months D. First 6 months

🌹 Most common time for presentation of infantile colic ?
A. 6 months B. 2 years C. 6 weeks D. 12 months

🍇 Case describe infantile colic ... what is the usual duration ?1st ٦ month
….
🌹 Scenario of infantile colic without mentioning the dx. The age of this condition usually? First6 month
..
🌹 consul mother regarding infantile colic?
A. 80% of children have infantile colic B. will resolve at 6 month C. gharib water have evidence in treat
infantile colic
…..
A child with excessive crying and can't stop crying. There is history of *bottle feeding*. On examination,
the child is restless, and abdomen distended. No other significant finding on examination. Dx?
A. Infantile Colic✅ B. Infant constipation C. Cow milk intolerance
….
🍓2 months baby the mother complained about that he cried excessively since 2 weeks they mentioned
that the mother indroduce 1 bottle feeding at night recently. his growth on 50% What is the cause :
A. Infantile colic✅ B. Cow’s milk allergy

🌹What’s true about infantile colic = w large passage of flatus ’ª‫ﻜﻮن ﻋﻨﺪﻫﻢ ﻏﺎزات ﻛﺜ‬¤
….
🌹Typical symptoms of infantile colic, all normal, what is most appropriate management?
A. Colic drops for colic B. Changing formula C. Behavioral adaptation
….
🌹6 week baby he spilt out whatever he drink he is now on cow milk his weight in birthday : 2,7 kg and
now he is 5.3 kg : A. observation B. precautions of reflux and reassure
....
Baby 4 months, distrnded abdomen, stool yellow and it becomes the color more ‫ﻌﺪ ﻳﻮم‬- ‫ﻟﻮﻧﻪ اﻓﺘﺢ ﻳﻮم‬
,after birth he passed stool. Dx? A- allergy to formula given(formula intolerance )✅ B- Volvulus C-
Diverticulum inflammatory
….
🌹Infant on breastfeeding after that take bottle feeding and complaining from constipation and
.distension dx? A-Cow milk intolerance B-Infant colick Answer is A
….
🌹Child 12 month ago on breastfeeding for first 9 months then use bottle feeding for another 3 months.
Came with symptoms and signs of anemia with splenomegaly
+ hypochromic microcytic anemia with retIculocytes number normal *RBC* count Peripheral blood
smear *Target cell* What should be restored = Iron

🌹Mother C/O child spitting after each feed cow's milk, Mx?
A- Elevate head during feeding✅. B- U/S C-Reassure
‫ﺼﻖ‬œ‫ق او ﻳ‬²•± ‫ﺔ ﻟﻮ ﻣﺎرﻓﻌﻨﺎ رأس اﻟﻄﻔﻞ راح‬¹‫ﻌ‬¹‫„ اﻟﺮﺿﺎﻋﺔ اﻟﻄﺒ‬i‫ ﺣ‬، ‫ ﻣﺎﻟﻪ ﻋﻼﻗﻪ‬، ‫ﻘﺮ‬œ‫ﺐ اﻟ‬¹‫•ﻄﻚ وﻛﺘﺐ ﺣﻠ‬Â‫ﺨ‬¤ ‫ اﻟﺴﺆوال ﺣﺎول‬، ‫ﺐ‬¹‫ﻄﻠﻊ اﻟﺤﻠ‬¤ ‫ﻞ رﺿﻌﺔ‬È ‫ﻌﺪ‬- ‫ﻃﻔﻞ‬
‫ﺐ ﻓﺎاﻟﺤﻞ ﻧﺮﻓﻊ رأﺳﻪ اﺛﻨﺎء اﻟﺮﺿﺎﻋﺔ‬¹‫ﺣﻠ‬
….
🌹 baby with “spitted up” after feeding his weight at birth 3.4 and now 5.5 how should you council the
mother ? let the mother time spent purpine and elevate the head of baby after feeding
‫ﻌﺪ اﻟﺮاﺿﺎﻋﺔ‬- ‫ اﻻم ﺗﺮﻓﻊ راﺻﻪ‬Šd‫ﺸﻨﺞ =ﺧ‬± ‫غ او‬Â‫ﺴﺘﻔ‬± ‫ﻞ رﺿﻌﺔ اﻟﻄﻔﻞ‬È ‫ﻌﺪ‬- „d c ‫ﻌ‬¤
🌹1 month old baby with sob, low grade fever, tachypnea,prolonged expiratory phase and in cxr there is
bilateral infiltration symptoms is the most common to be presented in this condition ?
1. Cough 2.Poor feeding ✅ 3.Peripheral cyanosis 4.Nasal flaring
‫ﻼء‬- ‫ﻪ‬¹‫ﻠﻤﺔ ﻣﻌﻨﺎﻫﺎ اﻟﺮﺿﻴﻊ ﻓ‬Ó‫ )اﻋﺮف ﻟﻮ ام ﻗﺎﻟﺖ ﻟﻚ ﻫﺬە اﻟ‬. ‫ﻪ ﻫﻮ (ﺗﻘﻮﻟﻚ اﻻم رﺿﺎﻋﺘﻪ ﻗﻠﺖ ﻣﺎﺳﺎر ﻳﺮﺿﻊ‬¹‫”ء ﻳﺘﺎﺛﺮ ﻓ‬•
d ‫ﻋﻤﻮﻣﺎ اي رﺿﻴﻊ اذا ﺗﻌﺐ اول اول‬

🍒Child shifted to cow’s milk, pale with wt loss? Iron def.🍒
,,,
🌹Child with cow's milk compline of palor = iron deficiency anemia

🌹Child with drinking 3 large glasses of milk, he is bicky in food choices, what type of anemia he has? Iron
deficiency

🌹vitamin black color= Iron🌹
..
child took tablets, came with black vomiting, most likely material ingested?
A. iron ✅ B. hypervitaminosis C. Wrong choice

🍓Child who is lethargic and losing his concentration, Hgb is 10.5, what to give?
A. IM iron✅ ‫ﺣﻞ ﺟﻠﻮري‬. B. Oral ferrous sulphate. C. Forti cereal. D. Vit,b12
… 🍏🍏🍏🍏🍏
🌹 Neonate in first routen visit had low hemoglobin 105 and give ora ferrous sulfate, in this visit also Iow
hg 10.3, what next investigation to reach dx?
A. level of serum iron and serum ferritin B. bone marrow C. give it oral ferrous sulfate.
….
🌹Breast milk =: rich in protein
....
🌹Another breast milk q content: A- lgA ✅ B-lg E C-lgG
....
🌹Toddler with pigmentations in his teeth, they describe the location of the pigments blah blah,
diagnosis:?Sleep with milk bottle

🌹colostrum high in?! A. Protein B.fat
Ýc ª ‫ﻢ ﺑﺮوﺗ‬¹‫ =ﻧﻌﻤﻞ رﺟ‬Û‫’ول ﻋﺎ‬i‫’ام =ﻛﻮﻟﻮﺳ‬i‫ر•ﻂ =ﻛﻮﻟﻮﺳ‬
d
..
🍓9 days newbon come with jaundice only in the face not extended to the rest of
the body.. otherwise he is healthy was delivered by NVD with no completing. and
he was breastfeed immediately. what is the cause of his jaundice?
A. Breast milk jaundice✅✅
N.B: Breastfeeding jaundice appear in the first week. Breast milk jaundice appear after the first week and
peaks around 2 weeks.
…..
🌹Baby with tooth discoloration, he sleep with bottle at night What the cause
A. Syphilis B. Bottle dental caries
...
🌹2 months old baby wakes at night and cries for 1-2 hrs was happening several times he’s fine at what
age this usually occurs? A. 6 weeks B. 2 months C. 3 months D. 6 months
.......
🍓8 yr boy wet his bed at night, the cause is? A. detrusor muscle🍓 B. Pelvic muscles
‫ = ر•ﻂ‬Wet ‫ ﻣﻊ‬Det

🍒 Night bed wetting considered normal till ?


„ „
‫ﻜﻮن‬¤ ‫ اي ﻋﻤﺮ‬Û‫ ا‬bed wetting ‫ﻪ‬n‫ﺤﺘﺎج ﻓﺤﻮﺻﺎت ﻣﺮﺿ‬v ‫ﻨﺎ‬P ‫ﻞ‬n‫ اﻟﻠ‬ƒ… ‫ﺎ ﻟﻮ ﻣﺎزال ﻳ™ﻠﻞ ﻧﻔﺴﻪ‬P‫ ﺳﻨﻮات |ﻌﺪ‬٥ ‫ ؟ ا• ﻋﻤﺮ‬، •
… n‫ﻞ ﻋﻨﺪ اﻻﻃﻔﺎل ﻃﺒ‬n‫ اﻟﻠ‬ƒ…
A. 5✅

🌹most important study to do in cases of enuresis in child: a.Urine analysis✅ b. urine culture c. VCUG
.....
🌹Most worrying sign of Child abdominal pain ? Late night pain ( not Pain for more than 10 min or Early
morning pain)

🌹A baby girl complains of dehydration and clitorymegaly ,signs of dehydation. next step :Steriod

🌹 Old male present with acute confusional state and postural hypotension .. investigation just
hypercalcemia what is the next ?? Hydrocortisone

1heart diseas
🌹A young boy complains of arthritis , rashes , nodule subcutaneous.He had pharyngitis two weeks back =
next step Echo ( Dont choic steroid )
….
🌹Boy collapsed during sport On Ex Jerky carotid pulse. what’s Dx?
A. HCOM B. wolff Parkinson’s white syndrome
...
🌹Pt came with HCOM Waht tx =Metoprolol

🌹child with radiofemoral delay pulse, Coartication of Aorta

🍓child/baby has no distal pulse what you need to check before reduction? pulses in other side
.......
🌹Baby will be prepared for open heart surgery, mother is very worry about him what is the best way to
deal with her worry? explain what will happen before and after the surgery ✅🌹
......

🌹What is Drug decreases the mortality in HF = ACEI/ARB + beta block 🌹


🌹Management of pericarditis = NSAID like Ibuprofen 🌹



🌹Sharp pain relieved by leaning forward, pericardial friction rub ==pericarditis🌹
..
PT pos-inferior MI, few hours developed hypotension, raised JVP, clears lungs on auscultation ? Right
ventricular infarction
right ventr ‫ ﻓورااااا ﻓﻛروا ﻓﻲ‬hypotenstion ‫ و اﻟﻛﻠﻣﺔ اﻟﻣﮭﻣﻣﻣﺔ ﺟدااااا‬posterior infarctio ‫( ﻻن داﯾم ﻟﻣﺎ ﻧﺷوف ﻛﻠﻣﺗﯾن‬
infarcrion )

🌹ECG show 2:1 heart block
A. 1st degree B. 2nd degree C. 3rd degree D. 4th degree

heart block2:1 ‫ ﯾﻌﻧﻲ‬Absent ‫ﺑس ھﻲ‬QRS . ‫اﻟدواﺋر اﻟﺣﻣراء ھذه ﻣﻔروض ﯾﻛون ﻓﯾﮭﺎ‬
....
First degree = ‫ﻜﻮن ال‬¤ PR pronlong just.
Scond degree 1 = ‫ﻜﻮن‬¤ PR Irregular
Scond degree 2 = ‫ﻜﻮن‬¤ rythm ‫’ ﻣﻨﺘﻈﻢ‬ª‫ﻏ‬
2:1 heart block=
= Absent‫ ﻏﯾر ﻣوﺟود‬QRS ‫ﯾﻛون ال‬
....
🌹Heart failure due to left ventricular hypertrophy
A. Systolic dysfunction B.Diastolic dysfunction ✅🌹
..
🌹 Pediatric patient with sob and productive cough with white sputum for one week that became yellow 3
days ago, on examination there's bilateral crackles increased in the right middle lobe. On x ray report
there's bilateral pleural effusion with
consolidation in right middle lobe. What is the most appropriate cause:?
A. Excabitation of heart failure✅

🌹2 months old baby with congestive heart failure and the mother asking about
nutrition requirement
A. Less than requirement for healthy baby
B. Greater than requirement for healthy baby
C. Same as healthy baby
D. Same as baby of 2 years old

🌹Congestive heart failure due to systolic left ventricular hypertrophy = Beta block +diurtic🌹

🌹Baby with congenital heart disease present with symptoms of heart failure (pulmonary edema, crackles,
etc ) Heart rate 250-300 what is the Dx? ’ª‫ﺐ او ﻓﻼﺗ‬¹‫ﻂ اﻟﻘﻠﺐ ﻧﺤﻄﻂ اذا ﻓ‬¹‫ ﺣﺴﺐ ﺷ•ﻞ ﺗﺨﻄ‬Š‫ﻋ‬
A.Atrial flutter. B.Atrial fibrillation. ✅✅. C.Ventricular fibrillation. D.SVT.
How does AFib lead to heart failure? ... Blood can “back up" in the pulmonary veins (the vessels that
return oxygen-rich blood from the lungs to the heart.) which can cause fluid to back up into the lungs.
When AFib causes heart failure,
........
🍓14M infant with had recurrent syncopal attacks worsened the HF and i think FTT on examination she
has diffuse crepitations on ECG she has cardiac arrhythmia narrow complex QRS and Hr 250-300 Bpm
what is the most probable cause:
A. Atrial fibrillation. B. Atrial flutter. C. Ventricular tachy. D. Supraventicular arrhythmia✅✅

1TOF
Baby after every feeding develop apnea and loss of conscious, what is the cause of admission in hospital?
a-Seizure b-Apnea c-Syncope ✅✅
‫ ﻣﺎﻧﺨﺎف‬، ‫ﺎب ﻣﻌﻴﻨﺔ‬œ‫ﻜﻮن ﻋﺎدي ﻟﻪ اﺳ‬¤ ‫ﺎﻧﺎ‬¹‫ اﺣ‬، ‫ﻌﺪ اﻟﺮﺿﺎﻋﺔ‬- ‫ﻨﻤﺎ اﻧﻘﻄﺎع اﻟﻨﻔﺲ‬â‫ ﺑ‬، ‫ﺎﻟﻘﻠﺐ‬- ‫’ة‬ª‫ ﻣﺸ•ﻠﺔ ﻛﺒ‬bd c ‫ﺎﻟﺬات ﻫﺬا ﻣﻌﻨﺎﻫﺎ‬- ‫اﻻﻏﻤﺎء‬
‫ﺐ‬é‫„ ﻧﻌﺮف اﻟﺴ‬i‫ و ﻓﺤﻮﺻﺎت ﺣ‬hc‫ﺸ‬è‫ﻌﺪ اﻟﺮﺿﺎﻋﺔ ﻫﺬا ﻻزم ﻣﺴ‬- ‫ﻪ‬¹‫ ﻋﻠ‬ç‫ﻐ‬¤ ‫’ ﻟﻤﺎ ﻃﻔﻞ‬ª‫ ﻏ‬، ‫ﻣﻨﻬﺎ‬.

children with cyanosis and systolic ejection murmur best heard at the left sternal border. Dx?
A. Tetralogy of fallot ✅✅ B.Transposition of great vessels C.Down syndrome
….
🌹Child noticed having cyanosis with feeding, with Physical exam a Holosystolic murmur was noted:
A. VSD B. TOF C. ASD D. Transposition of great arteries
….
🌹TOF 5 yo kid with history of surgical correction at 6 months. Now came with new left parasternal
decrescendo diastolic murmur with single S1 with left parasternal impulse, no radiation. What is the dx?
A. Mitral stenosis B. pulmonary regurg C. tricuspid regurg
tricuspid=right....‫ﻟو‬pulmonary ‫ ﯾﻌﻧﻲ‬left ‫ وﻻن ﻗﺎل‬regurg ‫ دوروا ﻋﻠﻰ ﻛﻠﻣﺔ‬diastolic murmur ‫اذا ﻗﺎﻟﻛم‬
…..

🌹Diastolic decresendo murmur in left sternum woith prevous TOF = pulmonary regurge ,
..... .....

🌹 case of TOF . How does it appear on ECG? right axis deviation🌹



🌹 Pt with Hx of TOF repair present with murmur in parasternal area it increase with inspiration =
pulmonary regurgitation, becz TOF has pulmonary stenosis.
.‫ ﻧطﻠﻊ ﺑرة ﻧﺷم ﺷوﯾﺔ ﺟو وﻧﻔس‬para = ‫رﺑط= ﯾزﯾد ﻣن اﻟﺷﮭﯾق ﺗﻧﻔس = رﺋﺔ‬
…..
🌹 TOF( Tetralogy of Fallot )mangment . ?
A-Dirotic iv B-pain relif and sedation ... ✅✅ C-no oxygen in answer

🌹classic case of DiGeorge syndrome (check the manifestations please) asking about the heart
abnormality:
A.TOF ✅ B.PDA C. TGA D. Endocardium cushion defect
foot.............‫ ﻟل‬. ‫ = ﻧﺑﺣث ﻓﯾﮭﺎ ﻋن ﺷوز ﻛوﯾس‬Googl ‫ = ﺟري‬Gerog = ‫ = ﻧﻌﻛﺳﮭﺎ ﯾطﻠﻊ ﻓوت‬TOF ‫رﺑط‬
.....
🍒Child with peripheral and central cyanosis? TOF
.. . . . . .
270- Child with cyanosis central and peripheral what is cardiac anomaly? A- TOF✅ B- PDA
‫ﻜﻮن ازرق‬¤ ‫ اﻻﻃﺮاف واﻟﻔﻢ‬bd c ‫ﻞ ﺟﺴﻤﻪ‬È Šd‫ﺨ‬¤ ‫ﺔ اﻧﻪ‬¹‫ﺎﻓ‬È ‫ﺎﻟﻘﻠﺐ‬- ‫ﺸﻮﻫﺎت‬î ‫ ار• ـﻊ‬hd i ‫ =ف ﻣﻨﻄ‬ì c
d ‫„ ر•ﺎ‬d ‫ﻌ‬¤= ë…d ‫’اﺗﻮﻟﻮ‬ª‫ ﻣﻦ اﺳﻤﻬﺎ ﺗ‬.
.... .... ...
🌹Family known about CongenitaL heart disease of baby but not known what it by examination central
and peripheral cyanosis = Tetralogy Of faLlot 🌹

🍓Neonate developed cyanosis (2nd or 3rd week after delivery) and there is finding on auscultation I
cannot remember exactly but they didn't mention about machinery murmur the qs about the
management? A. NSAID B. Steroid C. Prostaglandin✅
The “5 Ts” DO cyanotic Congintal Heart Disease =
Tetralogy of Fallot, Transposition of the great vessels, Tricuspid atresia, Total anomalous
pulmonary venous return, and Truncus arteriosus.
‫وﺳﺘﺎﺟﻼﻧﺪﻳﻦ‬s̈‫ﻠﻬﻢ ﻋﻼﺟﻬﻢ |ﺎﻟ‬-

🌹child with transposition of great vessels. Further evaluation of his mother could reveal? Elevated fasting
blood glucos 🌹

...........
🍒 Same Q and ask about concern sign (croup ) ? cyanosis 🍒🍒
‫ = ر•ﻂ‬croup = ‫ ﻛﺮب اذا ﺷﻮﻓﻨﺎ وﺟﻬﻪ زرق‬bd c ‫🍒🍒 ﻛﺮوب =ﻧﻜﻮن‬
.....
🌹Concerning symptom in croup? • Blue lips✅ • Tachypnea. • Expiratory stridor
............
🍇 croup given epi what's next step? steroids ✅(no epinphron on choic)

🌹 Egg shaped heart shadow. What's the congenital heart disease? TGA✅🌹

🍒🍒 Pt congenital heart disease and discretion in x ray egg shape what diagnosis ? Transpostion Great
arterios

🌹Neonate developed cyanosis (2nd or 3 rd week after delivery) + there is finding on on auscultation
machinery murmur = PDA🌹
. ‫ ﯾﻼ اﻟﻣﺣﺎﺿرة ﺑدﺋت = ﻣﺎﺷﯾﯾن اﻻن ﻟﻼﻣﺣﺎﺿرة‬، ‫ ﻣﺎﺷﯾﯾن‬machin = ‫ﻣﺎﺷﯾن‬PDA =‫رﺑط= ﺑدء‬
.............
Q about cyanotic spell, Mx ? knee-chest position, oxygen
….
🌹child crying and cyanotic= sedation and relieve the pain
..
🌹baby cyanosed with parasternal heave no murmur what to give? Prostaglandin
..
‫د‬Child pt cyanotlc . can't complete one sentence= intubation
..
🌹Child, k/c CHD , presented with cyanosis , progresing, o2 sat 85, looks ill, cyanosed, crying
A. immediate catheterization B. diuretic IV C. sedation + analgesia
..
🌹2 months old baby mother complaining of apnea usually happens after feeding with 10 mins of
cyanosis, why will u admit this case?
A. acute life threatening event B. Seizures C. sepsis
....

Baby born at 27 weeks GA developed SOB, tachypnoea. No X-ray. Diagnosis? apnea of premature
‫ﺎ‬P‫ﻜﻮن ا¸ﺘﻤﻞ ﻧﻤﻮ‬v‫ع ﻟﺴﺔ ﻣﺎ‬µ‫ واﻟﺮﺋﺔ ﺑﻬﺬا اﻻﺳﺒ‬، ‫ ﻻن وﻟﺪ |ﺪري‬.

🌹 Mother brought her 2 years old child to the ER with a history of upper respiratory tract infection for the
last 3 days with mild respiratory distress. This evening the child started to have hard barking cough with
respiratory distress. Which of the following are the most worrisome signs in this condition.
A. Nasal flaring B. Barking cough Cyanosis if not in choices > A
..
🌹Neonate cyanotic, low o2, Dx : A. hypoxic ✅ B.Hypocapnic C.Hyoercapnic.= Ýc ª ‫ اوﻛﺴﺠ‬bd c ‫ﻣﻦ اﺳﻤﻬﺎ ازرق ﻟﻮﻧﻪ ﻻن ﻣﺎ‬
….
What is the least physical activity duration required in pediatrics: 60 min
‫ﺎﺿﺔ‬ñ‫ﻌﻤﻠﻮا ر‬ñ‫ﺎﻟﻴﻮم و‬- ‫ﻤﺸﻮا ﺳﺎﻋﺔ‬¤ ‫„ اﻻﻃﻔﺎل ﻟﻠﺰم‬i‫ﺣ‬

🍒Patient playing sports and frequently developing sudden attacks of LOC,
examination: mid-systolic murmur in left side, What’s dx? Hypertrophic cardiomyopathy 🍒
‫ﻠﻌﺐ‬¹‫ﻠﻤﺔ اﻧﻪ ﺷﺎب ﺑ‬È ‫ اذا ﺷﻮﻓﺘﻮا‬sports ‫ﺎﻟﻘﻠﺐ‬- ‫ﻢ اﻧﻪ ﻋﻨﺪە ﻣﺸ•ﻠﺔ‬Ó‫ﻮا ﻟﻤﺢ ﻟ‬ñ‫ﻨﺎر‬â‫ واﻟﺴ‬، ‫ﻢ ﻣﺮض‬Ó‫ﺎﻟ‬- bd c ‫ﻪ ﻻاازم ﺗﺤﻄﻮا‬¹‫ ﻋﻠ‬ç‫وﻣﺎت او اﻏ‬
، ‫ ﻣﺮض‬bd c ‫ ﻃﻮل‬Š‫ ﻓﻜﺮوا ﻋ‬Hypertrophic cardiomyopathy ، ‫’ وﻫﻢ‬ª‫ﺎب ﻋﻤﺮﻫﻢ ﺻﻐ‬œ‫ ﺑﺘﻼﻗﻮا ﺷ‬، ’ª‫ﻘﺔ ﺑﺘﺤﺼﻞ ﻛﺜ‬¹‫وﻟﻠﻌﻠﻢ ﻫﺬا ﺣﻘ‬
‫ﻠﻌﺒﻮا‬¤ sports ‫ﻦ ﺗﻈﻬﺮ ﻣﻊ‬õ‫ ﻟ‬، ‫ﻈﻬﺮ اي ﻋﻼﻣﺎت‬¹‫ اﻟﺮاﺣﺔ ﻣﺎ ﺑ‬bd c ôd “ ‫ ورا‬hd i ‫ اﻟﻘﻠﺐ ﺧﻠ‬bd c ‫ ﻋﻨﺪە ﺗﻀﺨﻢ‬Ûd‫ﺐ اﻧﻪ ا‬é‫ واﻟﺴ‬، ‫ ﻋﻠﻴﻬﻢ‬ç‫ﻐ‬¤ ‫ﻓﺠﺎة‬
‫ﺔ‬¹‫ﻤ‬È ‫ﻘﻞ‬¹‫ﻪ ﻓ‬¹‫ ﻓ‬Ûd‫ﺐ اﻟﺘﻀﺨﻢ ا‬é‫ﺴ‬ø ‫ﺪﺧﻞ ﻟﻠﻘﻠﺐ‬¤ bd c ‫ﺎ‬È ‫ﻪ ﺗﻀﺨﻢ ف ﻻ ﻳﻮﺟﺪ دم‬¹‫ﻘﻮة وﻓ‬- ‫ﺾ‬œö‫ﻘﺪر ﻳ‬¤‫ وﻗﺘﻬﺎ اﻟﻘﻠﺐ ﻣﺎ‬، ‫ﺬل ﻣﺠﻬﻮد ﻗﻮي‬-
‫”ء ﻳﺘﺎﺛﺮ‬ •
d ‫ اول‬Ûd‫ و•ﺎﻟﺘﺎ‬، ‫ﻀﺔ‬œ‫ﻘﻞ ﻗﻮة اﻟﻨ‬ñ‫ ﺗﺪﺧﻞ ﻟﻠﻘﻠﺐ و‬Ûd‫ اﻟﺪم ا‬brain ‫ﻘﺪر‬¤‫’ ﻣﻊ اﻟﻤﺠﻬﻮد ﻣﺎ‬ª‫ وﻣﻤﻜﻦ ﻟﻮ اﻟﺘﻀﺨﻢ ﻛﺒ‬، ‫ﺤﺼﻞ اﻻﻏﻤﺎء‬ñ‫و‬
‫ﺾ اﻟﻘﻠﺐ ﺧﻼص ف ﻳﺘﻮﻓﻮا‬œö‫ ﻳ‬.
.. . . . .. . . . .
🌹Preschool checkup for asymptomatic 7 years male, with grade 3 systolic murmur best heard in the
lower left sternal border, intensity increases with standing, the most likely diagnosis: HOCM or MVP 🌹
….
🌹Pediatric with holosystolic murmur in left 3rd intercostal space = ASD ( Not PDA)🌹 holosystolic
murmur usually caused by ventricular septal defect, mitral regurgitation or tricuspid regurgitation,
...............
🌹8 year , with late systole ,mid sternum ,crescendo decrescendo, high pitched : systolic regurgitation

🍓 baby with Ejection systolic murmur with click heard in the left second intercostal and left parasternal
heave distended JVP and ejection systolic murmur increase with inspiration:
A. AS. B. coarctation of aorta. C. VSD. D. congenital pulmonary stenosis ✅✅
Click + Ejection systolic murmur +scond intercostal + left = pulmonary stenosis
.............
🌹 Pistol shot murmur in pediatrics case : aortic regurgitation

🍒VSD picture , symptomatic = refer for surgery 🍒

🌹child with mother death + he is not concentrantion in his lecture = dissociation
‫ﺣﻨﺎان‬ú‫ﻌﺪ ﻣﺎ ﺗﻮﻓﺖ اﻣﻪ ﺟﻠﺲ ﺳﺎاا‘ﺖ و‬- ، ‫ =ﺳﻜﻮووت‬Ý c ª ‫ﺎﺷ‬È‫ﺎﺷﻦ =داي =ﻣﻮت ﺳﻮ‬È‫ر•ﻂ =داي ﺳﻮ‬

1laryngomalacia
🍓Child with noisy breathing in prone position decreased when the child is sitting, the mother is worried
A. tell her his condition will get worse B. -He needs surgical intervention
C. He will get better spontaneously at the age of 1 year old✅✅

🍓Pediatric male pt presented with sign of “ laryngomalacia “ what’s the TTT:
A. Referral to surgical department B. reassure and wait for 1 year✅
Answer: no need for intervention so B It will resolve within 2 years
..
🌹 larngyomalicia what you well do? reassure its will resolve in her first birhday
NB :If there's in2 years more accurate
….
🌹Infant with airway obstruction that Increase with supine, decrease with prone= Laryngomalacia

🌹baby has breathing sounds his age is 3 months mother is worried
a. normal b. will outgrow it at age 1 year 90% resolve with time✅ N.B. laryngomalacia

typical case of laryngomalacia, dyspnea worse when prone and better when supine
w*o*rse when pr*o*ne

🌹Baby crying when change position decreased what couse? laryngomalacia
….
🌹3 months old mother's complain of noisy breathing sounds during sleep and disappeared when he
prone position and increased in supine position What is diagnosis?
A. Nasal atria B. Laryngomalacia C. Vocal cord syndrome D. Subglottic stenosis

Baby developed SOB , whezzing DR start to give broncodilator then no response, think of laryngomalacia
dx test is ? Bronchocsopy ( no laryngioscopy in choices)

🌹Laryngomalacia case and ask about the best investigation ? laryngoscope

# tracheomalacia on children = What is diagnostic image ? By Bronchoscopy

‫ ﻣﺗﻛرر وھو اﻧﮫ ﻟﻣﺎ ﯾﺎﺧذ ﻧﻔس اﻟﻘﺻﺑﺔ اﻟﮭواﺋﯾﺔ ﺗﺗوﺳﻊ وﻟﻣﺎ ﯾطﻠﻊ اﻟﮭواء ﺑﺎﻟزﻓﯾر ﺗﺗوﺳﻊ‬wheezing ‫ھذا اﻟﻣرض ﯾﻛون ﺑﺎﻻطﻔﺎل ﯾﺗﻣﯾز ب‬
‫اﯾﺿﺎ اﻟﻘﺻﺑﺔ اﻟﮭواﺋﯾﺔ ﻟﻛن ﻣﻊ اﻟﻣرض ھذا ﺑﺎﻟﺻوؤة ﻟﻣﺎ ﯾطﻠﻊ اﻟﮭواء اﻟﻘﺻﺑﺔ ﺗﻛون ﻣﺎزاﻟت ﺿﯾﻘﺔ ﻣﺛل اﻟﺻورة ھذه‬

………………
🌹barking cough=Laryngotracheobronchitis
….
Physical exam findings (auscultation) in croup ( patient was having nasal congestion , barking cough) :
A-expiratory wheeze and prolonged expiratory phase ✅✅
B-inspiratory sounds die subglottic obstruction
# barking cough = stridor = croup = expiratory wheeze.
….
🍒A child with inspiratory stridor, barking cough, most likely diagnosis?
- laryngotracheobronchitis🍒🍒
….
🌹croup ++ barking cough = expiratory wheeze and prolonged expiratory phase ✅

🍒barking cough and respiratory distress: croup
‫رﺑط = ﻛروب‬croup = ‫ ﻛرب = ﻟﻣﺎ ﯾﻛون اﻟواﺣد ﻓﻲ ﻛرب ﯾﺣﺗﺎج ﺑرﯾك راﺣﺔ‬bark = ‫ﺑرﯾك‬

🌹Barking cough ( croup ) what will listion on auscultation ?


Increase inspirantion sound due to subglottic narrow ( inspirantion stridor ) (Not expirantion ❌)
.. . . . . . .
Barking cough ( croup ) what will give ?
Ÿ Inhaltion epiniphrin and oral steroid ✅
Ÿ Inhlation steroid ❌
Ÿ Empric anticiotc ❌
..
🌹Croup classical case barking cough + inspiratory stridor + Auscultation: inspiratory stridor DX : croup
cause : parainfluenza
‫ ﻧﺷم ﺷوﯾﺔ ﺟو‬para ‫ﻛروب = ﻧطﻠﻊ ﺑرة‬
......
🌹child is having barking cough = viral croup

🌹3 years old Patient with hx of mild atopic dermatitis, presented with Barking cough and stridor, what's
the dx: A- bronchial asthma B- spasmodic croup
....
🌹 Kid with inspiratory stridor, mild respiratory distress, hoarseness of voice, barking cough which of the
following is concerning symptoms: - A. Tachypnea B. Expiratory stridor. C. Nasal flaring D. Blue lips
…..
🌹hoarseness of voice in Croup ?
inspiratory stridor with subglottic stenosis (Not expirantion)
‫ ر•ﻂ =ﻛﺮوب‬croup = ‫ﺮب‬õ‫ واﻗﻠﻞ ﺷﺪة اﻟ‬ý i
d ‫ﻴﻖ ﺣ„ ارﻓﺢ ﻋﻦ ﻧﻔ‬â‫ﻴﻖ ﻋﻤﻴ‬â‫ﺤﺘﺎج اﺧﺬ ﺷﻬﻴ‬¤
...
🍓CROUP given epinephrine and after 30 min the symptoms came agine = manag by = Reapate again

🍇 croup given epi what's next step? steroids ✅(no epinphron on choic)
….
🌷Croup----------steeple sign on X-ray🌷
‫ ﻛﺮوب =ﻛﺮب =ﻻزم ﺗﻜﻮن ﺻﺎﺑﺮ‬steepl
..... ...... ......
● Fever, Severe sore throat, high-pitched sound when breathing in (stridor), Difficult and painful
swallowing, Drooling, Anxious, restless child . Feeling better when sitting up or leaning forward what is the
diagnosis ?? A. Epiglottis. B. bronchiolitis. C. Pharyngitis

🌹drolling saliva? A-CRUP B-EPiglottis ✅

🌷Epiglottis-----------X-ray thumb sign🌷 ‫ﺐ‬¹‫ ﻋ‬Epi= ‫👍 اﻻﺻﺒﻊ‬
….
🌹Child with fever ,sob , drooling what next = intubantion and mutiblspichil team = Epiglotitis ( ( Dont
choice x ray, bez he ask nest step we will not wait the x.ray)) .🌹
…..
Child preschool age has VSD 2mm,asymptotic,what will you do= Watchful and waiting ✅
.‫اﻻھم ھو ﻟو ﻓﯾﮫ اﻋراض ﻻزم ﻋﻣﻠﯾﺔ‬mm 5 ‫ اذا اﻟﺛﻘب ﻛﺑر ﻋن‬، ‫ ﯾﻌﻧﻲ ﺻﻐﯾرة وﻻن ﻣﺎﻓﻲ اﻋراض اذا ﻧﻘول ﻧﻧﺗظر ﻋﻠﯾﮫ‬mm 2 ٢ ‫ﻻن‬
.....
🌹child have vsd 2mm asymptotic == watchful waiting✅🌹‫¾ء‬ ¿ ‫ﺔ وﻻ‬n‫ة واﻗﻞ ﻣﻦ واﺣﺪ ﺳﻢ ﻓﻤﺎ ½ﺴﻮي ﻟﻬﻢ ﻋﻤﻠ‬sz‫ﺻﻐ‬
„ …
‫ اﻋﺮاض‬ƒ… ‫ وﺧﺎﺻﺔ ﻣﺎ‬.
….
Infant mouconim aspirations management: A-Nitric oxide? B-lavage surfactant

‫ اﻟﻲ‬mecnium ‫ ﻋﺷﺎن ﻧﺳﺣب ﻟل‬trachia ‫ ﻣن‬tube ‫ ﺑﻌظﮭﺎ ﻟو ﻣﺎﻧﻔﻊ ﻧدﺧل‬، ‫ ﻟﻠﺑﯾﺑﻲ‬suction ‫ﻟﻛن اﻟﻲ ﻣﻔروض ﻧﺳوﯾﮫ اول ﺷﻲء ﻧﻌﻣل‬
vetilantion .‫ ﻧﺿﻌﮫ ﻋﻠﻰ‬، ‫ وﻟو ﻟﺳﺔ اﻟطﻔل ﻣﺎ اﺳﺗﺟﺎب‬، ‫ وﻧﻌﻛﯾﮫ ﻣﺿﺎد ﺣﯾوي‬، ‫ ﺑﻌدھﺎ ﻧﺿﻊ ﻟﮫ ﻣﺎﺳك اوﻛﺳﺟﯾن‬، ‫دﺧل ﻟﻠرﺋﺔ‬
meconium aspiration syndrome:
MAS occurs, your newborn will need immediate treatment to remove the meconium from the upper
airway. After delivery, your doctor will immediately suction the nose, mouth, and throat.
If your baby isn’t breathing or responding well, a tube may be placed in your newborn’s
windpipe (trachea) to suction the fluid containing meconium from the windpipe. The suctioning may
then continue until no meconium is seen in the material removed. oxygen therapy to make sure there is
enough oxygen in the blood. the use of a radiant warmer to help your baby maintain body temperature.
antibiotics such as ampicillin and gentamicin to prevent or treat an infection. the use of a ventilator (a
breathing machine) to help your infant breathe.
. . .. . . . .
Pediatric patient has fallen from tall building, presented to ER with SOB and right chest pain,
investigations showed multiple fractures multiple sites on his body, CXR showed: Right pneumothorax
with mediastinal shift, your management:
= its Tenstion pnumothorax = A.Thoracotomy. B.Thoracostomy C. Intubation and mechanical ventilation
Thoracotomy vs. thoracostomy
*thoracoStomy is a procedure that is used to drain the space between the lungs and chest wall of excess
fluid, blood, or air.
Thoracotomy is surgery that makes an incision to access the chest. It's often done to remove part or all of a
lung in people with lung cancer.
What is Needle Thoracostomy?
urgent needle thoracostomy (also called "needle decompression") is performed. Simply, it is the insertion
of a large-bore needle or cannula through the chest wall and into the pleural space to allow air within the
pleural cavity to escape.
‫ ﻓﻴﻬﺎ ﺣﺮف‬Ûd‫اذا اﻟﺠﻮاب ا‬
….

🌹child with erythema marginatum , knee pain , fever = what inv =ECG 🌹
💠erythema marginatum = its rash with Rheumatic fever ,we order ecg to see any muse heart
damage 💠

1Virus
💐Child presented with ulcers on mouth and gingiva erythematous based and pale in the center. Dx?
A. Coxaci ✅💐vB.EBV vC.Herpes
....... ......

Rash on the face and inner cheeck there’s whits spots: = Measles
‫ﺾ‬¹‫ﻠﻪ ﻟﻮﻧﻪ اﺑ‬ÿ‫ = ر•ﻂ =ﻣﻮز ﻟﻤﺎ ﻧﻔﺘﺤﻪ وﻧﺎ‬wight spot ‫ اﻟﻔﻬﻢ‬bd c .
.......
🌹2yrs old child Not vaccinated, had fever for 3-4 days then rash appear, on examination there was white
spot in mucosal membrane. What is dx? A. measles B. Rubella

Pediatric patient with coryza, conjunctivitis, and white spots in the mouth, what is the diagnosis?
A-Measles ✅✅ B-Rubella C-Mumps

🍓Measles should mention the 3Cs (Cough coryza conjunctivitis) mouth koplick spots

🍓Child with sore throat and coryza 2 days ago came with difficulty swallow food what is investigation
you will order? A. Chest x ray B. Ct scan✅‫ ﺣﻞ ﺟﻠﻮري‬C. Lateral neck x ray
..
🌹Child with fever conjunctivitis, coryza cough, wheezing Tachypenic what is the optimal ttt =O2 therapy
(not Steroid)

🌹 Child with acute onset fever, Coryza, sore throat, difficulty swallowing solid food. Ex showed cervical LN
2.3 cm. Next step? . CT neck
....
🌹Young girl has a recent history of fever, difficulty in swallowing solid food only. enlarged 2cm cervical
LN. investigations will you do ? Lateral neck X RAY.

🌹Fever and cough and maculopapular rash in behind ear and face and trunk
A. mubes B. measala C. rubella

🍇7 y/o unvacclnated boy presents with red erythematous irregular patches of rash that is around hls
neck and spreads down hls back. What does he have?
A. measles ✅B. Chickenpox C. Rubella D. Pertussis or something weird like that
.....
🌹A child with runny nose and fever which subsides and then rash appear allover
his body starting from the face. Dx? Rubella, after fever subside it’s roseola, concurrent fever Rubella.
....
🌹Baby with white eye reflex (Leukocoria) and murmur. Mother mentioned viral infection during
pregnancy: ? A. Rubella ✅ B. CMV C. Toxoplasmosis ‫ﻀﺎ‬¹‫ رﻋﺐ =ﻋﻴﻮﻧﻪ ﺑ‬.
....
🌹Absent red reflex caused by wich infection?Rubella

Face Rash with conjunctivitis spread later to the trunk: = Rubella
‫ واﺣد ﺟﺳﻣﮫ ﻣن وﺟﮭﮫ ل ﺟﺳﻣﮫ اﺣﻣر‬rubella ‫رﺑط = رﻋب‬
.........
🍒🍒child with rash started on face and then spread to the trunk. ? Rubella🍒🍒

🌹Newborn with absent red eye reflex and new murmur , what does his mother had when she was
pregnant ? A. Rubella B. Cmv C. Toxoplasmosis
….
🌹pic of skin lesion in child , rash was red and mother tell it start on face the go to trunk , with LN enlarge
of groin , Dx ? A- imptigo B- HSV C- candidia D_rubell

Baby presents as shown in pic, what would you do? Immediate referral to ophtha✅

🌹Kawasaki management? A.intravenous gamaglubulines✅


intravenous immunoglobulin (IVIg) can treat include: Immune deficiencies like immune thrombocytopenia.
Kawasaki disease.
.............
🌹Kawasaki sign? bilateral red eyes
….
🌹Case about kawasaki how you will assess the coronary artery disease? A. iIVIG+ASA B.ECHO ✅
..
🌹Kawasaki disease assess for heart complication? Echo

🌹kawasaki case how you will assess the coronary artery disease? by 2D echocardiography (2DE) or
coronary angiography. 🌹
......
🍓A child presented with 5 days of fever, oral mucosal lesions, cervical lymph node enlargement and limb
edema. Lab results essentially normal. Drug of treatment?
A. Acyclovir B. Cefotaxime C. Ampicilin D. Aspirin✅kawasaki case
.......
🌹The best treatment of Kawasaki disease? Aspirin and IVIG
…….
🌹kawasaki best treatment= a. aspirin b. IVIG
N.B. Aspirin for fever, continued for 2 months. High dose IVIG is ultimate treatment

Child with Conjunctivitis Rash cervical lymph nodes fever what to give as a treatment?
- Aspirin✅ - NSAID - No IVIG in the choices
….
🌹poor prognosis in treating Kawasaki with IVIg? High CRP
..
🌹A patient with kawasaki features,what is the best indicator as poor response to IVIG?
A. Neutropenia B. High CRP✅ C. Albumin

🌹A child is complaining of 5 days of fever , bilateral non-purulent conjunctivitis , rashes in palms and soles
( case of kawasaki ) =Aspirin

🍇Case of Kawasaki ask about what will reduce affect of lvlg =A. Anemia B. Neutropenia ✅

🌹Which indicated poor response to IVIG In Kawasaki = Neutrophilia, hypernatremia, high CRP

🍓Kawasaki on IVIG how to know if bad prognosis : C reactive protien ✅ A. Anemia B. Neutropenia

🌹Kawasaki case asking about which of the following is one of the criteria:
A. anterior uveitis B. myocarditis C. injection conjunctivitis with no exudate D. arthritis
.....
Fever 5 days, conjunctivitis, lymphadenopathy, high ESR and CRP. Dx? A-Rubella B- Kawasaki ✅
…….
🌹child with fever and then rash and peeling on hands and edema with peeling lips = kawasaki
....
Fever 5 days, conjunctivitis, lymphadenopathy, high ESR and CRP= Kawasaki
….
Confirmation of Kawasaki? Clinical ✅ ‫ﻠﻬﺎ ﻟﺴﺎن اﺣﻤﺮ وﻋﻴﻮن ﺣﻤﺮاء وﺧﻠﺼﻨﺎ‬È ‫ﺤﺘﺎج ﻓﺤﻮﺻﺎت‬¤‫ﻣﺎ‬
….
🌹Child with vesicle at lip and gum and proximal tongue and hard palate. = gingivostomatitis ‫ﻣﻦ اﺳﻤﻬﺎ‬

🌹Pedia with Egg allergy contraindicantio A. Yellow fever B. Influenza
...
🌹4yrs old fully vaccinated child, came to the ER with on day history of fever and sore throat which started
on the same day. What is dx? A. scarlet fever B. Kawasaki C. Measles

🍓Pt with pharyngitis for 2 days , what’s the possible complication : sclar fever infectious
Pharyngitis/Tonsillitis;
Definition = inflammation of the pharynx, especially the tonsils if present, causing a sore throat =

🍓peritonsillar abscess = Quinzi
‫ﻂ =ﺑﺮ‬i‫ ر‬peri = ‫ﻚ ﻗ™ﻞ اﻻﺧﺘ™ﺎر‬v‫ = ﺑﺮ واﻟﺪ‬Quiz
........

Complications
= immune-mediated complications =
🍏scarlet fever🍏 🍅acute rheumatic fever🍅. 🍏 post-streptococcal GN 🍏. 🍅 reactive arthritis, 🍅
1_SCARLET FEVER
acute onset of fever, sore throat, strawberry tongue
• 24-48 h after pharyngitis, rash begins in the groin, axillae, neck, antecubital fossa; Pastia’s lines + may
be accentuated in flexural areas
• within 24 h, sandpaper rash becomes generalized with perioral sparing, non-pruritic, non- painful,
blanchable
• treatment is penicillin, amoxicillin, or erythromycin x 10 d
……
A 12-year-old girl presents to her pediatrician for a sore throat. Her symptoms began approximately
1 week ago after she attended a sleepover.
Since then she experienced a sore throat and noted a temperature of 101°F (38.3°C). She denies a cough
but has noticed increasing fatigue and difficulty swallowing due to pain. On physical exam, she has
anteriorcervical lymphadenopathyandpatchy tonsillar exudates and swelling. What complication
could he develop? -Scarlet fever✅
Scarlet fever is a disease which can occur as a result of a group A streptococcus (group A strep) infection,
also known as Streptococcus pyogenes. The signs and symptoms include a sore throat, fever, headaches,
swollen lymph nodes, and a characteristic rash.
‫ اول ﻧﺮﻛﺰ اﻧﻪ ﻗﺎل‬patchy tonsillar exudate ‫ ﻣﻦ‬، ‫ﺎس‬-‫ﺘﻮا¸ﻮ‬ÍÌst‫ﺎ ﺳ‬Ìsz‫ﻜﻮن |ﻜﺘ‬v ‫ﺬا ﻏﺎﻟ™ﺎ‬P‫ي و‬sz‫ اﻟﺘﻬﺎب |ﺎﻟﺤﻠﻖ |ﻜﺘ‬È… „ ‫ﻌ‬v
‫ اﺣﺪ ﻣﻀﺎﻋﻔﺎت‬group A strep ‫ﺤﺼﻞ‬¤ ‫ اﻧﻪ‬Scarlet fever i ‫ﻠﻤﺔ وﻣﻦ‬È ‫ﺼﺎب ب ((ﺣﺮارة ))ﻫﺬە اﻫﻢ‬¤ ‫„ ان اﻟﺸﺨﺺ‬d c ‫ﻠﻤﺔ ﻫﺬە ﺗﻌ‬Ó‫اﻟ‬

‫ﻠﻤﺔ‬È ‫ﺎ‬¹‫ اﺳﻤﻬﺎ ﺛﺎﻧ‬Scarlet È… „ ‫ﻌ‬v Rash ‫ﺔ‬Ì‫ اﻟﻐﺪد اﻟﻠﻤﻔﺎو‬ƒ… ‫ﺸﻤﻞ اﻧﺘﻔﺎخ‬Ð ‫ﻤﺎن‬-‫ و‬lymphoadenopathy .
‫ﻤﺎن‬È‫ﺎﻟﻐﺪة و‬- ‫ﻌﺪﻫﺎ ﺣﺮارة ﻣﻊ اﻧﺘﻔﺎاااخ‬- ‫ﺎﻟﺤﻠﻖ وﺟﺎء‬- ‫ﺪي‬¤‫ ﻓﺎذاا ﺷﻮﻓﻨﺎ واﺣﺪ ﻋﻨﺪە اﻟﺘﻬﺎب ﺻﺪ‬rash ‫ ب‬ç‫ﺴ‬± ‫ ﻧﻔﻜﺮ ان ﻫﺬا‬scler fever
‫’ي ﻟﻠﺤﻠﻖ‬ª‫ﻜﺘ‬œ‫ﺐ اﻟﻬﺠﻮم اﻟ‬é‫ﺴ‬ø .
Scarlet fever :

.... . . . . .. . .
pt with fever ,cervical lymphadenopathy swollen tonsils (without?) patch Possible complication = -
Pharyngitis. - Scarlet fever✅✅
Scarlet fever is a disease which can occur as a result of a group A streptococcus (group A strep) infection,
also known as Streptococcus pyogenes. The signs and symptoms include a sore throat, fever, headaches,
swollen lymph nodes, and a characteristic rash.
….
🌹 pediatric patient with fever, on examination, there is a white membrane covering the tonsils, the most
likely complication to be happened is: A. Scarlet fever ✅. B. Pharyngitis C.Glomerulonephritis

🍓Complications of tonsillitis= A. Scarlet fever✅ B. Pharyngitis. C. Glomerulonephritis

🌹14-yrs had fever, pharyngeal exidate, enlaged LN, most common complication = Scarlet fever

🍓Case of infant have cough and low grade fever ,rash ,runny nose :
A. RSV✅ B. rubella Cough + Running nose + Fever
….
🍓Child with fever and vomiting and rash on 2nd day rash become over All body
A. Meningococcemia. B. Rocky mountain fever✅. C. Kawasaki. D. Measles
…….
🌹Prophylaxis for contact with pertussis:
The macrolide antibiotic = erythromycin, clarithromycin, and azithromycin 🌹
pertussis = ‫ﻌﻤﻞ‬v ‫ = ﺟﺪاا ﻣﻌﺪي‬whooping cagh
‫ﻂ‬i‫ = ر‬per = ‫ﺠﺔ‬nÔ‫ﺎﻟﻚ اﻟﻨ‬n‫ﺸﻮﻓﻪ |ﻌ‬Ð Èt‫ﻚ =ﺣ‬v‫ = ﺑﺮ واﻟﺪ‬My son = mycin
.............
🍓pedia pt not vaccinated, present with sore throat and cervical lymphadenopathy what’s your dx:
A. diphtheria✅ B. streptococcus = not related to vaccin . C. pertussis = croup

🍒 How many years the pertussis vaccine last? 10 years🍒

🍓1-year-old, never vaccinated, presented with "hacking" cough and inspiratory something, What's the
organism? Pertussis
….
🌹Pertussis with severe vomiting, most complications? Pneumonia ( not Pneumothoroax)

🌹4 months old with proven pertussis infection on macrolide. His 3 and 5 years old siblings are vaccinated
up to date. What is the proper action to prevent the siblings from getting the infection?
A. prophylactic macrolide. B. booster vaccination against pertussis
C. observe them for the possibility of developing the infection.
….
🌹3 month years old present with paroxysmal cough with deep inspiration between
the cough, conjunctivitis, diarrhea, he is up to date with his vaccination.. What is the causative organisms?
A. Chlamydia B. Adenovirus C. Pertussis

🌹 Pertussis case “whooping cough”Ask about diagnosis and Investigation:
A. Nasopharyngeal swab B. Blood culture
.. ‫ ﻓﺗﺣﺗﯾن اﻻﻧف‬nose =OO‫ ﻛﺎﻧﮭﺎ‬whooping ‫ ﻣن ﻛﻠﻣﺔ‬OO ‫ﺣرف‬
……
Patient 50 yrs. Old. Came for. Routine check. Up visit, by CXR you. Find.Solitary Cheast nodule. About 2*3
patient is asymptomatic ,what is most useful thing to ask. -Ask about old CXR. ✅

🌹diffuse ST elevation (ECG pic) ttt? aspirin ✅✅ Pericarditis 🌹
….
🌹Child with Sx of varicella. Has immunodeficient brother. Action with the immunodeficient child?
A. Avoid skin contact B. Immunoglobulins ✅ C. antiviral meds

1Bacteria infection
Meningitis with 1 month or less choice (.ampicillin + gentamicin).
Meningitis with more than 1 month age choice ( ceftriaxone and vancomycin).
Meningitis with Bacilli catalase choice (Ampicillin) .
….
🍓Neonatal lumbar puncture + diplococci Management?? B.ampicillin + gentamicin✅
🎃….‫ ☠🏴ﺑﺳﺔ)ﺑﯾﺳﯾﻠﯾن( = اﻟﺟن ﯾﺗﻛﺷل ﻋﻠﻰ ﺑﺳﺔ ﻋﺷﺎن ﯾﺧرب اﻟﺧطﺑﺔ‬+ ‫ 💍ﺟن‬+ ‫ دﺑﻠﺔ‬:‫…رﺑط‬.
…..
3 days old , csf culture showed gram + bacilli catalase + beta hemolytic , how to treat? ampicillin ✅✅

🍓15 months with meningitis, Gram stain G+ double coccus tt=
A. vancomycin and gentamicin B. ceftriaxone and vancomycin✅ C. ampicillin
….
🌹3 day neonate with B hemolytic and catalse +ve what antibiotic give:
I )ampicillin 2)gentamicin 3)ceftriaxone
….
🌹 🌹Child 15 month with meningitis lp show gram+.cocci mange?A. Vanco+ceftr B. ampicillin +gentamicin
….
🌹 15 m old baby with sign of meningitis on csf analysis you found gram positive diplococcai what is the
treatment ? A.ampocilib B.voncomycin C. ampicilibe with guntamycin D. vancomycin with ceftraixon

🍒 8 years old child came with fever ,neck stiffness ,irritability, normal glucose normal protein ,increases
WBC what's abx used ? A- Ampicillin. B- Ceftriaxone. C- Vancomycin. D- Ceftriaxone and vancomycin. ✅
….
🌹7Y with meningitis , Gram stain G postive diplococci tt: = ceftriaxone and vancomyci ‫ﻌﺾ‬- ‫ ﻣﻊ‬Ý c ª ‫ ﻻزم اﻻﺛﻨ‬.
….
🍓 Child with pain that last for 10-15 mints (intermitted pain) crying , postive kering sign ? What is the
best treatment : Ceftrixon + Vanco 🍒Kernig's sign = meningitis 🍒

7 y/o with meningeal irritation, headache, and fever. CSF (normal protein and normal glucose and
lymphocytosis). What you will give the child?
A.Ceftriaxone and vancomycin and steroids B.ceftriaxone and steroid
C. antiviral ✅✅ Bez( normal glucose )
.......
🍇Pt 7 yrs old with nausea and vomiting dehydrated comatose acidotic CSF : high protein , normal
glucose = viral meningitis ✅

🍒5 y/o with fever, lethargy, positive Brudzinski sign. CSF showed lower limit of glucose with high protein.
Gram stain revealed gram positive cocci in chains. Your management?
A. Ceftriaxone, vancomycin, and steroid* ✅ B.ceftriaxone and steroid C.antiviral
......... .....
7 y/o with meningeal signs, headache, and fever. He and his family came from Africa recently. He also has
sore throat and lymphadenopathy. CSF (normal protein and normal glucose and lymphocytosis). What you
will give the child? A.Coronavirus B.CMV C. EBV ✅
.......
🌹Child after returning from Africa. Presented with fever, sore throat followed by meningitis symptoms.
CSF shows normal protein and glucose but high lymphocytes = organism? EBV (not CMV)
….
Fever for 6days and tender splenomegaly, which culture is most importantly needed?
-urine and stool culture -repeated blood cultures ✅ -bone marrow smear culture
‫ﺪ‬ñ&‫ﻔ‬¤‫ اﻟﺘﺎ‬bd c ‫ ﻧﻔﻜﺮ‬، Ûd‫ﺠﺎ‬¹‫ﻠﻴﻨﻮﻣ‬œ‫ﻪ ﺳ‬¹‫ﻜﻮن ﻓ‬¤ ‫ﺎم و‬¤‫ ا‬٦ ‫ﺪ ﻋﻦ‬ñ$‫ ﻟﻤﺎ اﻟﺤﺮارة ﺗ‬.
Baby mele fever wet his diapers what is suggests uti in that baby wetting dippers boy == fever✅
‫ ﺧﺎﺻﺔ ﻓﻲ‬urinary tract infection ‫ ﻻن ﻣن اﻋراض‬، ‫ وھو وﺟود اﻟﺣرارة‬UTI ‫اﻟﺳؤوال ﻗﺻده اﯾش اﻟﻲ ﯾﺧﻠﯾﻧﻲ أﺷك اﻛﺛر ان ﺳﺑب ﻛﺛرة اﻟﺗﺑول ھو ﺑﺳﺑب‬
.‫اﻻطﻔﺎل ھو اﻟﺣراااارة ﻣﻌﮭﺎ ﻛﺛرة اﻟﺗﺑول‬
......

🍇 boy 15 years old fever, abdominal pain, splenomegaly? multiple blood cultures✅

🌹Girl 7 years old with suprapublc pain No rebound no guarding. Tx?
A-observ B-admits iv abx C- discharge with oral Abx ✅
,,,
🍓7 year old child, presented to the emergency by his partners with 2 days history of fever and vomiting,
child is comatose dehydrated with acidosis. CSF report: Cells 20 (above normal) , Protein 0.45 (above
normal) ,Glucose (Normal)
A. Tubercular meningitis. B. Salicylate toxicity. C. Diabetic coma. D. Viral meningitis. ✅
…..
🌹child with chill ,fever ,irritability , on exam there was neck rigidity and positive Kernig’s,CSF showed
clear color, lymph and neutrophils high, protin and glucose normal, dx?
A. pneumococcal meningitis B. Meningococcal meningitis ‫ ﻧﻮرﻣﺎل‬Ý c ª ‫ﻻ ﻣﺎﻧﺨﺘﺎرە ﻻن اﻟ…’وﺗ‬
C. aseptic meningitis ✅ D. pyogenic meningitis
….
🌹Girl confused with fever + sign of meningitis = encephalitiis 🌹(Bez confustion somthing happen to
brain , bactiria goto brain)

🍓child meningitis, what is the complication ? A. blindness B. hearing loss✅
….
🌹Chime with meningitis came with his parents and has papilldema , parents are afarid of ?
A-Hearing loss ✅ B-Vision loss
….

child with Irritability ,headache ,nausea, lethargy and rash all over the body what is dx : A.
meningococcemia ✅ B. kawasaki
🌹 3 mo old boy with pic of bacterial meningitis What’s most common pathogen?
A. Moraxella catarrhalis. B. Streptococcus pneumonia. C. Streptococcus pyogen
🌹pediatric patient have meningitis, with close contact to his brother recently, Asking for what to give to
his brother: A. Rifampicin B. IVIG
….
🌹 3 y/o boy with maculopapular rash on this limbs and buttocks is brought to the ER by his partners
saying he is lethargic and irritable. On examination, neck stiffness is noted. LP shows diplococci parents
are concerned about his 6 y/o brother. What prophylaxis will give= Oral rifampicin

🌹Most common cause of ear infection or otitis media in children ? A. Bacteria B. Virus C. Fungal

🌹 Child came from africa. complaining of weakness, he couldn’t move his head and legs especially when
he is prone. What is the dx? A. Polio B. Cmv
….
🌹At school age what we tend to prevent?
A. Japanes encephalitis B. Hemophilus influenza encephalitis✅ C. Herpes encephalitis
….
🌹Children with 2 days history of ear pain, exam reveals perforated ear drum with fluid passing through
it= A) Chronic otitis media B)Acute suppurative otitis media ✅ C) Otitis externa
….
🌹Most common virus cause of acute otitis media in pedia =Rhinovirus
‫ = ر•ﻂ‬rhino = ‫„ اﻧﻒ‬d c ‫ﻌ‬¤ = ottits ‫ﻌﺾ‬œ‫ ﺑ‬Ýc ª ‫ﻄ‬œ‫ ﻣﺮﺗ‬Ý
c ª ‫اذن اﻻﺛﻨ‬
...
🌹Child hasfever with perforated tympanic member and pus in the external canal = Acute OM ( Not
Otitis media with effusion )

🌹 Pediatric with fever, ear pain ruptured tymp
A- Acute otitis media. B- Otitis media with effusion. C- Otitis externa

child came with limping and non-weight bearing, not allowing anyone to touch his leg, most common
organism is: staph. Aureus ✅+(septic arthritis)
.‫ ﻻ ﺣد ﯾﻠﻣﺳﮫ‬، ‫اوه ﺑس ﯾﺄﻟم‬aureus = ، ) ‫رﺑط = ﺳﺗﺎف = )ﻋﻠﻰ اﻟﺣﺎﻓﺔ ﻣﺎﯾﻘدر اﺣد ﻣﺟرد ﺑس ﯾﻠﻣﺳﮫ‬
….
🍓child with fever and left knee pain and swelling. Most important single investigation?
A. Blood culture. B. FBC. C. Joint aspirate✅✅. D. Xray

🌷 Conductive hearing loss could be because of recurrent OM.🌷
….
🌹honey crust infection= A) staph aureus ✅. B) GA streptococcus. C) herpes
…..
🌹Pediatric has unilateral hip joint pain Flexing Refuse to be touched Mostly causative organism?
A. Staph aurus ✅✅vB.Group b strep vvvC.H influnza
…..
child with limping for 2 day and abnormality in hip what's most likely organism=A. staph ✅. B. GBS C. Hib
......
🍓Child with pneumonia, indication of hospitalization?
A. Vital signs( hypotension and tachycardia) B. not able to take orally ✅

4 years old with pneumonia, with vomiting for any oral food .. vitaly stable; reason for admission:
A. pneumonia B. unable to tolerate orally
….
🌹headache, stiff neck, and vomiting, coughing and breathing with breathing difficulty , causative
organism ? strep. Pneumonia

🌹Pediatric Pneumonia = A. Iv B_ oral Abx
…..
🌹13 months old girl present with fever 38 , bilateral lung infiltrate , she looks mildly ill , what is the likely
organism==A. Moraxella catarrhalis B. Strep pneumoniae C. Hib influenza
….
🌹 Baby with tachypnea, cough, hemoptysis and bilateral lung infiltrates. What is the treatment ?
A. Steroid. B. Antibiotic✅. C. Surgery. D. Betablocker
Hemoptysis is the coughing up of blood or blood-stained mucus from the bronchi, larynx, trachea, or lungs.
In other words, it is the airway bleeding. This can occur with lung cancer, infections such as tuberculosis,
bronchitis, or pneumonia, and certain cardiovascular conditions

🌹Pneumonia ask about treatment

🌹 child with flu like symptoms+ fever + has middle lobe crackles+ stony dullness, chest x.ray will show =
pleural effusion bez dulness not pnimothorax

🌹Staccato cough : . chlamydia pneumonia
….
🌹Children with cough, fatigue, 2 time bloody vomiting, low grade fever, with dullness in percussion dx?
A. parapneumonic effusion B. pleural effusion C. TB
….
🌹Absolutely contraindicantion in penicillin allergy? Pipracillin/tazobactam
....
🌹Child with poor feeding since 2 days have oral thrush and dipper dermatitis what you will give
A. Topical antifungal. B. Oral antifungal. C. Topical and oral anti fungal D. Systemic antifungal
….
🌹Pediatric pt with Rash in cheeks trunk and upper limb:
A. dermatitis herpetiformis (location typic on extensor surfaces like elbow)
B. Impetigo C. Candida D. herpes simplex

Cystourethrogram of a pediatric with recurrent UTI =
VesicoUrethral reflux is a condition in which urine flows backward from the . bladder to one or both
ureters and sometimes to the kidneys

.. . . ...... ...... .....


case of febrile neutropenia what next? cluture form sputum,urine,blood and Iv antibiotics ‫ﻞ ﻣ•ﺎن ﻧﺎﺧﺬ‬È ‫ﻣﻦ‬
‫ﺸﺎر‬è‫ﺎﻟ‬È .
Patient presented with typical picture of malaria infection, blood smear showed no
parasite, what’s your next step?
A. Repeat thin blood smear. B. Repeat thick blood smear C. Repeat every 8 hours for two days ✅
….
🌹Tonslitis = amoxacillin / clavi .🌹
..
🌹Neonate with sign of sepsis what is empirical antibiotic ?. Ampicillin🌹
..

🌹Q about treatment of uncomplicated cytits in child ?


A.iv ceftrixone B.im ceftrixone C. oral amoxicilln ✅✅. D.oral carbonate
....
🌹Pediatric prophylaxis for maningitis? rifampicine 🌹
….
…..
Baby with septic arthritis (scenario not directly saying the diagnosis) what is the best study:
A) fluid aspiration from the joint✅ B) blood culture C) CRP
........
🌹Young girl with diarrhea came with left knee swelling, right elbow, left Achilles tendon. Stool analysis
shows +ve clostridium toxins. What is the dx?
A. JIA B. Reiter's syndrome C. septic arthritis D. Reactive arthritis
....N.B: Reactive typical sites > knee and ankle..
🌹Baby has swollen tender joint, limited passive movement, the most important test: Examination of
synovial fluid
‫ﻪ‬¹‫ ﻧﻔﺤﺺ اﻟﺴﺎﯨﻞ ﻋﺸﺎن *ﺸﻮف اذا ﻓ‬wbc ‫ﻌﺪ‬œè‫”ء *ﺴ‬ • c
d ‫„ اﻫﻢ‬d ‫ﻌ‬¤ ، ‫ﺔ او ﻻ‬¹‫ ﻋﺎﻟ‬septic arthritis . ‫ﻞ اﻟﺴﺎﺋﻞ‬¹‫ﻖ ﺗﺤﻠ‬ñÂ‫ﻜﻮن ﻋﻦ ﻃ‬¹‫ ﻓ‬.
......

When resuscitating a child with septic shock, which of the following has the least evidence of benefit in
treatment? A. ABX B. inotropes C. steroids ✅✅

Ped URTI lethargy , confuse , Tem39, rr35 , hr>100 = A.Sepsis ✅ B. Septic shock
If hypotension and not respond to ivf = septic shock
Severe sepsis = organ damage
.....
🍓Commonest cause of shock in children? .sepsis✅
🌹baby with septic shock, what is less indicated? steroid
..
🌹Neonate with High fever, developed petechial rash and was hypotensive 70/55, with cold extremities
and poor feeding. What is the dx: A. septic shock B. ITP
..
🌹Case of child with leukemia , after 17 days of chemo coming with fever , neutrophil is normal, tx? blood
culture , urine culture , broad spectrum iv abx 🌹

🌹Child with fever, vomiting and diarrhea. ABG: Normal pco2 + Normal bicarb Base access -4 “normal
range from -2 to 2” Ph 7.3 What type of shock does he have?
early compensated shock ✅ Not late but early = bez evry thing normal .

+ve sexual history 6 years ago , labs WBC 0.3 Serology: cryptococcosis positive What is the best diagnostic
test = HIV serology
............ .......
🌹Pedia pt under went tonsillectomy, the father notes that surgery take more time than usual but pt
recurved well, thre was bleeding during surgery they ask what you tell the father? Explain to the father
what happened 🌹
….
🍓Child with flu symptoms on examination he has membranous exodus in tonsils, causative organism
A. streptococcal✅. B. EBV ‫ﻓﺎﻳﺮس‬

🍓child present to the clinic with pain in micturition and lower abdominal pain
all symptoms of UTI and his parents observe in his urine foul smell, which organis can cause this
condition? A. E coli gram negative✅. B. E coli rod in shape. c. Proteus gram negative bacilli
70 ‫ﺎ‬ñ’ª‫ﻜﺘ‬- , d ‫ﺔ ﻣﻦ ﺣﺎﻻت اﻟﺘﻬﺎب اﻟﻤﺴﺎﻟﻚ‬¹‫ﺎﻟﻤ‬- e.coli

🍓Child Foul smelling (urine) which organism?E. Coli.

..... ......
🍒🍒 10y boy with hx of 2wk of bloody diarrhea and abdominal pain, tenesmus: Amebiasis🍒🍒

Child c/o fever, bloody stool, and tenesmus, abdominal exam showed abdominal distention, Dx?
A. Ascaris. B. Amebiasis✅. C. Giardiasis ( watry diarrhia ) D. Rotavirus
🌹bloody stool, and tenesmus, fever = Amebiasis🌹

🌹child has bloody diarrhea and oliguria, vomiting, nausea, abdominal pain, fever for 7 days before that
family think this from restaurant What's ,treat= A.- antibiotics✅ B.- supportive C.- steroid
….
Child with bloody diarrhea fever 39c dx : A. Salmonella B. Shigella C. Campylobacter ✅
Sheigella = neurolgical Sx = ‫ﺔ‬¹-‫ﺐ ﻋﺼ‬é‫ﺴ‬±= ‫ﺷﻘﺎوة‬

🌹6 years with sore throat + difficulty swallowing + painful cervical lymph nodes. the organism =
streptococcus pyogenes
….
🌹 Child with group A strep pharyngitis. What will you do with his brother?
A. Observation ✅ B. Throat culture C. Throat swab for rapid antigen test D. Antibiotics
..
🌹Child with chronic diarrhea and labs indicative of macrocytic anemia asks which of the following is
important in past giardiasis infection

🍇Baby with mass in umbilical and developed veslculopustular rash grape like organism
A. coxsackie B. Hsv C. Group A strep✅

1Neuro
🍇 child took hyoscine butylbromide and metoclopramide for gastroenteritis and fulud the he develop
jerky movement (not sure about the presentation it was wired to me ) what to give ? domperidone
Thid drug improves symptoms of nausea, vomiting, bloating, and feeling of fullness.

🌹child starting brief sizer (less than 30 seconds) , EEG (generalized 3-Hz spike-and-wave activity.)
Treatment = ethosuximide ( Absence seizure)
….
🌹Child fall from his bed complain of headache and 2time vomit everything normal like physical exam
neurological and he is conscious next? A. close observation B. CT head

🌹 An 8 year old boy’s parents complain that he has episodes where he blinks multiple times and becomes
okay after that.. he is conscious and responsive during those episodes. The most likely diagnosis is ,,
A. Tics B. Tourette syndrome. C. Blinking disorder

Addictive found in deep coma and cyanosed what do to?
a-Mechanical ventilation ✅ b-History details from family c-Lavage gastric ‫ﻞ ﻣﻌﺪة‬¹‫„ ﻏﺴ‬d c ‫ﻌ‬¤
‫ اول ﺷﻲء ھل راح ﻧﺧﻠﻲ اﻟﻣرﯾض وﻧﻣﺳك اﻟﻌﺎﺋﻠﺔ وﻧﺎﺧذ اﻟﮭﺳﺗوري ﺑﺎﻟﺗﻔﺻﯾﯾل واظل اﺳﺎل‬coma ‫ ﻣرﯾض ﺟﺎﻧﺎ ﻣدﻣن وﺷوﻓﻧﺎ وﺟﮭﮫ ازرق و ﻓﻲ‬: ‫ﻟو ﺗﺧﯾﻠﻧﺎ اﻟﻣوﻗف‬
.Mechanical ventilation ‫واﺳﺎل واﺗرك اﻟﻣرﯾض ؟ ﻣﻧطﻘﻲ اول ﺷﻲء اﺳوي اﻧﻘذه واﺳوي ﻟﮫ‬
….
🌹3 y old boy fall down from his bed No LOC But vomiting 3 times + headache Neurological ex > normal
What next ? A-Brain MRI B-Brain Ct C-Close observation✅ D-Neurologist consult

🌹Fracture of the left stylomastoid foramen during delivery of a baby:
A. Decreased blood supply to the left ear B. Loss of sensation of the left side of face
C. Loss of anterior 2/3 sensation of tongue

🌹child unable to feed herself with a spoon , hx of head trauma 10 days ago where is the lesions?
cerebellum
‫ﻘﺪر ﻳﻮازن اﻟﻤﻌﻠﻘﺔ‬v‫ ﻣﺎ‬. .....
pt BMI 18 but think of oneself as obese thus dieting, Dx? A. bulimia‫ﺎﻟﻌﻤﺪ‬- ‫ﻞ‬ÿ‫غ اﻻ‬Â‫ﺴﺘﻔ‬î ‫ﻌﺪﻳﻦ‬- ’ª‫ﻞ ﻛﺜ‬ÿ‫ﺗﺎ‬
B. anorexia✅
….

1-2months old , full term , diabteic mother birth weight is 4.8kg and no other complications during
delivery absent moroʼs reflex on right side what is the cause: Erbʼs palsy ✅
‫ﻠﻤﺔ‬È ‫ﺲ *ﺸﻮف‬ø: ((absent hand motor reflex )) bd c ‫ ﻃﻮل‬Š‫ ﻧﻔﻜﺮ ﻋ‬Erbs palsy
absent hand motor reflex === Erbs palsy
......
🌷 Earb’s palsy, happened to macrosomia baby Wight more than 4500🌷

🌹unilateral moro's reflex = Erb's palsy
….
🌹Child has pneumonia+ fever then developed seizures. Treatment:
A- Amoxicillin B- paracetamol C- diazepam
….
Child with febrile illness and seizur
A. IV lorazepam B. Nasal lorazepam. C. Rectal diazepam✅✅
‫ ﻻن ﻋﻠﻰ ﻛﻼم‬، ‫ وﻟﻛن طﻠﻊ ﺧطﺎ‬، ‫ھذا ااﺳؤوال ﺣﻠﮫ ﺟﻠوري ﺳﺎﺑﻘﺎ اﻧﮫ ﻧﻌطﯾﮭم ﺑﺎراﺳﯾﺗﺎﻣول = ﻋﻠﻰ اﺳﺎس ﯾﻧزل اﻟﺣرارة وﯾﻌﺎﻟﺞ اﻟﺗﺷﺟﻧﺎت‬
‫ وﻟﯾس‬anticonvultion ‫ ﻓﺎﻟﻌﻼج ﻧﻌطﯾﮭم‬، ‫اﻻﺑﺣﺎث ﺣﺗر ﻟو ﻋطﯾﻧﺎھم ﻋﻼج ﻟﻠﺣرارة اﻟﻲ ﻋﻣﻠت اﻟﺗﺷﺟﻧﺎت ﻣﺎ راح ﯾﺟﯾب ﻓﺎﯾدة‬
Diazepam‫طﯾب اﻟﺳؤوال اﻟﺛﺎﻧﻲ اﯾش اول ﺷﻲء اﺑدءه ﻣﻌﮭم ﺑﺎﻟﻌﻼج ؟‬paracetamol ,
here is no evidence that antipyretics reduce the risk of subsequent febrile convulsions in at risk children.
Prescription of paracetamol following febrile seizures may provide comfort and symptomatic relief, but
should not be recommended to prevent further febrile convulsions.
plus taking a dose of diazepam (Valium) gel that’s administered rectally. You can be taught to give the
treatment at home if your child has recurrent febrile seizures.
.......
🌹Continuing >35 minutes Seizure epilepticus Given lorazepam iv what's next? Iv phenytoin
phenytion ‫ ﺛم ﻧﺑدء ب دواء‬lorazepam ‫ ﻣﺛل دواء‬diazepam ‫ول ﺷﻲء ﻧﺑدء ب‬
‫ ﻻن‬. ‫ ﻋﻧدﻧﺎ وﻣﺎ ﻛﻔﻰ اﻟوﻗت ﻧﻌﻣل ﻟﮭم اﻛل ﯾﺳﻌﻔﻧﺎ اﻟﻔﯾﻧوا‬zepam ‫ ﺟﻣﯾل ﻋﺷﺎن ﻟﻣﺎ ﯾﺟﻲ زﺑﺎﯾن‬pheno 🥖🥖 ‫رﺑط = ﺧﺑز اﻟﻔﯾﻧوا‬
😂‫ﺳﺎﻧدوﯾﺗﺎش ﺳﮭل‬
.....
Child with fever and active seizure now. TTT? A- Phenytoln B- Paracetamol C- Diazepam ✅

🌹Child k/c of grand mal seizure on Depakine comes with breakthrough seizure
what to give initially after intubation on ER presentation: Diazepam

🌹Pt status epilepticus for 5 min, with iv access, what is the first line:
A. lorazepam B. diazepam rectal C. buccal midazolam

🌹kid with seizure for more than 5 mins, iv line secured = IV lorazepam

🌹pediatric pt with tonic clonic seizure before 5 min resolve ,what Rx. to give ( availability of IV access) all
options was anti-epileptic.
🍒Status epilepticus (more than 5m) > Lorazepam
Generalized tonic clonic > Carbamazepine
Partial tonic clonic > Phenytoin
Absent seizure > Ethosuximide

🌹Cerebral palsy in kernicterus?
Answer is: Athetoid cerebral palsy or dyskinetic cerebral palsy.🌹
Kernicterus is a type of brain damage most often seen in babies. It's caused by an extreme buildup of
bilirubin in the brain. Bilirubin is a waste product that's produced when your liver breaks down old red
blood cells so your body can remove them.
.....
A case of icterus ‫ﺎ ﺟﻮﻧﺪ×ﺲ‬P‫ >=ﻣﻌﻨﺎ‬Kernicterus
1-ABO incompatibility 2-RH incompatibility

🍓Fist hand +feet crossed seen in : A. down syndrome B. cerebral palsy✅

6w or month old with unilateral absence of red reflex what to do next?
A-Mri brain and optic. B-Funduscopy
🌹best time to do red reflex examinations =at birth and age of 6 weeks 🌹
….
15 months old baby presented to the clinic with developmental delay, on examination patient was having
spasticity, crossed leg, lower limb were involved more than the upper limb but both were affected, what’s
the type of cerebral palsy he has: A. Quadriplegia ✅. B. Hemiplegia. C. Diplegia. D. Athetoid

🍓girl with bruises and fecal incontinence after being fully toilet trained: dx sexual abuse✅

🍓Parents came with there child complaining of that there child always blinking on rest and activity on
examination there is no pain and tearing what is the most likely diagnosis? ‫ﻘﻮة‬- ‫ﻐﻠﻖ ﻋﻴﻮﻧﻪ‬¤ „d c ‫ﻌ‬¤
A.conjunctivitis B.dry eye syndrom C.tourette syndrome D. : tics disease ✅
.‫ ﯾﻐﻠق ﻋﯾﻧﮫ ﺑﻘوة‬، ‫ ﺑﻘوة‬tight ‫ = ﻣن‬tics = ‫رﺑط‬
.......
🍓child came with continued seizure for 5 minutes ,intravenous line is secured, first line management?
A. rectal diazepam B. IV lorazepam ✅

🌹Child with rapid eye, he is active with parents during the attack= Tic disorder
….
🍓Child with repetitive eye movements. Otherwise normal no pain nothing. On exam no redness or
tenderness or edema. Dx? Tics syndrome
. 🚕🚖🚕🚖. ‫ = ﺟﺎﻟس ﯾﻧظر ﺑﻛل ﻣﻛﺎن ﯾدور ﻋﻠﻰ ﺗﺎﻛﺳﻲ‬repetitive eye movements ‫ = ﺗﺎﻛﺳﻲ‬Tics = ‫رﺑط‬
…..
5-7years old presents with inability to stand or sit unsupported and clumpsy gait and resistance to neck
flexion after chicken pox infection weeks ago:dx: Acute cerebellar ataxia ✅
‫ ﻫﻮ‬Ûd‫ اﻟﺘﻮازن ﻟﻠﻤﺨﻴﺦ ا‬bd c ‫ﻌﺪوى ﺣﺼﻞ ﻟﻪ ﺧﻠﻞ‬- ‫ﺔ‬-‫ﻌﺪ اﺻﺎ‬- ‫ﻘﻮﻟﻚ‬¤ ‫ﺎﻟﺴﺆوال‬- ‫ رﻛﺰ‬celebrum ‫ﻌﺪوى‬- ‫ﺐ اﺻﺎﺑﺘﻪ‬é‫ﺴ‬ø ‫وﻫﻮ‬
Acute cerebellar ataxia of childhood is a childhood condition characterized by an unsteady gait, most likely
secondary to an autoimmune of postinfectious cause, drug induced or paraneoplastic
....
Case of abuse, subdural hematoma and retinal hemorrhage , the childʼs mother is mentally retarded but
father is normal = shaken baby syndrome ✅ ‫ﺔ‬ñ&‫„ واﺿﺢ اﻧﻪ ﻫﺰة ﻗ‬d c ‫ﻌ‬¤= Ý c ª ‫ﺎﻟﻤﺦ و اﻟﻌ‬- ‫ﻒ‬ñ$‫ﻻن ﻗﺎل ﻟﻚ ﻧ‬
....
fetus delivered by Vacuum instrument, has swelling that doesn’t cross the sutures? Cephalohematoma

🍒 Child Head trauma subarachnoid, hge, periorbital edema, bruises and LOC Father said he found her like
this, what to do? Call child protection🍒

🍒🍒A fetus delivered by Vacuum instrument, has swelling that doesn’t cross the sutures?
Cephalohematoma 🍒🍒
.......

c ª ‫ﻣﻦ اﺳﻤﻬﺎ ﻋﺼﺐ اﻟﻮﺟﻪ =ﻋ‬


Can’t close his eye inone sidewhat nerve affected? -Facial Nerve ✅ Ý
......
Duchenne muscular dystrophy sign? Gowers maneuver ✅ ✔


child with febrile seizure since 3 days, tonic clonic lasting an hour, he still have fever. what to do? A-give
phenytoin. B diazepam Ö C-Abx D-Paracetamol
N.B: Febrile seizure main treatment is to treat underlying cause, but if patient came to you in ER seizuring
then :- If 5 minutes or more --> Diazepam. If < 5 minutes--> Paracetamol.
If non seizuring in ER --> Paracetamol, whatever the duration he seized at home.

Child afraid of going to school, how can the mother deal with the case =
Talk to him about how his favorite super hero would deal with the situation.

🌹1x2 cm on the forearm since birth, no symptoms =follow up
...
🌹What improved to decrease the premature baby mortality rate? hypothermia
1Bronchiolitis
Case of bronchiolitis (severe) with chest recessions what is the management? -Admit for fluids hydration
and oxygen✅
‫ﻪ ﺟﻔﺎف‬- ‫ﺤﺼﻞ‬¤ ‫„ ﻻ‬i‫ﺔ ﻟﻠﺠﺴﻢ واﻟﺴﻮاﺋﻞ ﻻن ﻃﻔﻞ ﺣ‬ñ‫’و‬i‫„ ﻳﺰود اﻟ‬i‫ ﻋﺸﺎن اﻻ*ﺴﺠﺔ ﻣﺎ ﺗﻤﻮت ﺣ‬Ý
c ª ‫اﻻوﻛﺴﺠ‬
Oral corticosteroid medications and pounding on the chest to loosen mucus (chest physiotherapy) have
not been shown to be effective treatments for bronchiolitis and are not recommended.
Hospital care
A tiny percentage of children need hospital care to manage their condition. At the hospital, your child may
receive humidified oxygen to maintain sufficient oxygen in the blood, and perhaps fluids through a vein
(intravenously) to prevent dehydration. In severe cases, a tube may be inserted into the windpipe (trachea)
to help the child's breathing.
.........
🍇Portia pt have tachypnea, runny nose, cough,slightly elevated fever, audible wheezing sound whats is
tho definitive diagnosis: = Nasopharyngeal swabs (case of Bronchiolitis caused by RSV) ✅
...
🌹18 months with picture of bronchiolitis, developed several episodes of apnea. What’s the appropriate
management? a. Supportive b. Ventilatory management c. IV methylprednisolone
...
🍓two cases about bronchiolitis: cause and treatment= RSV. Ttt: supportive rehydration ✅
.....
🌹 pt have arthralgia after viral infection" watery diarrhoea " what is the type of arthritis?! Reactive
..
🌹Case descripe typlcl symptos of bronchitis / asking about the diagnisis (Chronic productive cough , SOB
Bronchiolitis ) RSV = supportive
,…
🌷Bronchiolitis:---------RSV🌷respiratory syncytial virus.
.....
🌷Child with Small VSD, asymptotic, Mx: observation and watchful waiting.✅

🌹Pediatric with intact radial and reduced femoral, with fhx of htn: Coarctation of the aorta

🌹 best diagnostic for CoA - Coarctation of the aorta for neonate ? A. Us B. Ct C. Echo

🍓Child need non invasive M.V , Where you will manage him?
A. Emergent room✅. B. Child ward. ward (not sure of name). C. Infant. D. Intensive care unit
N.B:8 months age might be the age. Baby in icu with multiple cong.

🌹 Child with mild fever and bilateral conjunctivitis and abdominal pain subsided 2 weeks ago now on x-
ray follow up you found bilateral lung infiltration, most common pathogen?
A. Moraxella catarrhalis B. Streptococcus pyogen C. Adenovirus

1Pediatric surgery
🌹2 year old boy with pain over anterior tibial tubercle, diagnosis? A- Osgood Schlatter Syndrome
..
🌹RTA and you find aortic thoracic injuries and splenic abrasion with hypotension what u do ?
A. Thoracic surgery B. Abdominal surgery C. Call center for vascular surgery D. laparotomy

🍇Young boy Just had a growth spurt came with pain in hls leg , dx ?
Ankle sprint = Osgood schlatter if below the knee

🌹 positive rebound tenderness in Macc Barney point (case of appendicitis) the pathophysiology =
peripheral vasoconstriction

🌹Peds 8 yrs old with RLQ pain and rebound tenderness what's confirmatorytest?
A. US abdomen B. MRI abdomen C. CT abdomen

child having sudden pain at thigh pic of spiral fracture with labs of (PTH Ca) high
A. primary hyperparathyroid ✅. B. secondary. C. parathyroid carcinoma
......
🌹9y old child , PTH high , ca high , came with bone pain tt ?
B.rehydration + diuretics + bisphosphonate ✅
C.rehydration + bisphosphonate + diuretics + statin
......
🌹Child with bowed legs Labs: calcium is high. Phosphate is normal. = familial hypophosphatemia ( not
nutritional vit D deficiency )

🌹Bowing legs, frontal bossing , management: Vit D3


..
child with rash appeared as vesicles. Some of his other classmates were having the same. He has
immunodeficient brother. Family has concern about their immunodeficient child. What is your action?
A.give oral antiviral B.don’t do anything because this disease is self-limited C. give IVIG ✅✅✅.....
.....
boy came with deep stabbed wound in the anterior right thigh 10 cm in depth. What is your next step?
A.tourniquet B.call vascular surgeon C. direct pressure apply on the wound* ✅
D. directly clamp the femoral artery
….
Child abdominal trauma, investigators show splenic lacerations 2cm w peri-spleen
fluid most appropriate management A. Non OPrative ✅ ‫ﺔ‬¹‫ﺤﺘﺎج ﻋﻤﻠ‬¤‫„ ﻣﺎ‬d c ‫ﻌ‬¤

- child fall dawn on his hand ( radial&ulna) fracture,1cm open wound


A.Closed reduction with cast above elbow. B.Closed reduction with cast bellow elbow
C. Innernal fix with cast till elbow. D. surgical debridement& fixation ✅
‫ ﻧﺨﺘﺎر دي‬، ‫ﻂ ﻣﺎ ﺗﺤﺮك اﻟﻌﻈﻢ ﻣﻦ ﻣ•ﺎﻧﺔ‬¹‫ﺴ‬ø ²‫ او ﻛ‬²‫ﻪ ﻛ‬¹‫ﺲ ﻟﻮ ﻣﺎﻓ‬ø ، ‫ ﺣﺴﺐ اﻻﺷﻌﺔ‬Š‫ﻌﺎ ﻋ‬œ‫ﻫﻮ ﻃ‬
…..

🌹Child with humural & ulnar & and un able to move extensors muscle of forearme and hand ? median
nerve in cubtal fossa 🌹
….
Child with vascular malformation of lower limb , when to interfere : A- pain✅✅ B- Claudication

🌹 Newborn circumcision , chordee and hooded foreskin + hypospadias ,how to manage?
A.open circumcision B. plastibell circumcision ✅ C.circumcision with *** D.referral to pedia surgeon

🍒An infant with hypospadias and for circumcision, what’s the procedure?
The surgeon will use a small piece of foreskin to create a tube that
increases the length of the urethra ✅✅
Boys who are born with hypospadias should not be circumcised at birth. The extra tissue of the foreskin
may be needed to repair the hypospadias during surgery. ... During surgery, the surgeon may place a
catheter (tube) in the urethra to make it hold its new shape
….
🍒Bleeding Circumcision which factor ? 8🍒 ‫ = ر•ﻂ‬Circumcis= Circumcision= 8 ‫ﻋﺪد ﺣﺮوﻓﻬﺎ‬
......
🍓Neonate+ prolonged bleeding after circumcision, aptt high, pt, bleeding time and platelet are narmal,
condition is most likely due to deficiency in
A. V. B, Vii. C. Vii ✅factor 8. D. X
Bleeding after ciraumcision factor 8
Bleeding after umbilical stump> factor 13
Bleeding after home delivary - Vit K (factor 10)
.........
home delivery baby with umbilical bleeding after day 5, dx:
A- vit k deficiency B- factor X deficiency ✅. C- hemophilia ((umbilical = ❌ fsctor))
.......... .......
infant had bruises on his thigh after delivery at home what is the cause? vit k def

🌹boy come with gingival hyperatrophy with enlarge and bluish purple friable
gum . What’s vitamin deficiency? A. vitamin A B. vitamin C C. vitamin D D. vitamin E

(Due to vitamin K deficiency bleeding in the newborn) B. factor x deficiency
PT high , PTT ==prolonged PT means that the blood is taking too long to form a clot.caused by conditions
such as liver disease, vitamin K deficiency, or a coagulation factor deficiency (e.g., factor VII deficiency)

🌹 Boy bleeds out after tooth extraction Then after develop bruises they mention factors Vlll= +ve , Plat=
normal Only slight increase in pt Dx?. von willebrand disease
V = III (VON) ‫ﺛﻼث ﺣﺮوف ﻣﻊ ﺛﻼث اي‬

🌹Baby for circumcision u found urethra midshaft what procedure will u do
A. gomco clamp B. plastibell C. other name can’t recall D. inform surgeon
…..
🌹Baby for circumcision u found urethra mid shaft what procedure will u do
a. gomco clamp b. plastibell d. inform surgeon✅
….
🌹 6 months uncircumcised pediatric UTI how to treat: A. IV ceftriaxone B. IV cefepime C. Oral Ceftriaxone
….
🌹Months boy Uncircumcised with fever , labs showed UTI = IV Ceftriaxone✅ cut = ‫ﺳﯿﻒ‬
….
..
🌷5 y/o boy uncircumcised presented to ER with fever and abdmonial pain, suspected
to have UTI, urine analysis showed: Nitrate positive and high WBC, Which of the following will most likely
indicates he has UTI: A. Leukocytosis. B. Nitrate ✅✅. C. Urine gravity D. Urine RBC
🌷most likely indicates he has UTI: Nitrate🌷
….
🌹 4 months Child with mid-shaft hypospadias, came for circumcision. What u will do?
A- not possible since they will use it for the repair.✅ B- should be delayed.
#Babies who have hypospadias that requires surgery shouldn't be circumcised, because the foreskin may
be for tissue grafts during the operation#

🌹Child circumcised has UTI treated with TMP/SMX and improved, which further should be done?
A- renal ultrasound B- cystourethrography C- reassuring

Pediatric patient presented with abdominal pain and jelly like stools, right quadrant mass (case of
intussusception) what’s the best diagnostic test?
A. Abdominal x-ray. B. Abdominal CT. C. Abdominal U/S. D. Barium enema ‫✅ ﻣﻜﺮر‬
🌷jelly like stools, = the best diagnostic test = Barium enema🌷
...
two cases of Intussusception, one about next step after stabilizing pt. :
barium enema / radiological reduction✅
...
- intussusception case, what would you tell the mother?
A- Shock is most common complication B- Require surgery immediately
C- Recurrence common after surgery✅ D- Enema carried out in case of peritonitis
the probability of recurrence was 100% after the fourth episode of intussusception. After the third
episode, the probability of recurrence and eventual surgery are 68% and 70%,
respectively. Surgical intervention should be considered at the third episode of intussusception.
….
Picture of intussusception : nausea and vomiting Which statement regarding
Diagnosis A. presence of sausage shape in palpation ✅✅ B. Passage of current jelly stool confirm the
diagnosis C. Present in 2% of population
….
🌹Case of baby with intermitted cries that is sever to the point where he is rise his legs and screams for
hours what best nexst step = abdominal ultrasoud 🌹(intussusception )
‫ ﻓﮭو‬، ‫ وﻻ ﺳﻲ ﺗﻲ ﺳﻛﺎن ﻟﻠﺑطن‬، ‫ ﺑﺎﻗﻲ اﻻﺧﺗﯾﺎرات اﻛس راي ﻟﻠﺑطن‬، ‫ ﻟﻔﺗرات ﻣﺗﻘطﻌﺔ‬، ‫ وﯾرﻓﻊ ﻗدﻣﮫ ﻣن ﻗوة اﻻﻟم‬، ‫طﻔل ﯾﺻرخ ﻣن اﻻﻟم‬
US‫ ﻧﺧﺗﺎر طﺑﻌﺎ اﻻراﺳﺎوﻧد‬nexst step ‫ﻗﺎل‬
. . . .. .
🌹Tt of intussusception in pediatric = hydrostatic enema 🌹 ( Dont choice reduction by ultrasound or
fluroscopic)
….
🌹Abdominal exam sausage shaped mass = Intussusception 🌹

…….
🌹intussusception case how you confirm it clinically: A- Sausage mass B- Red currant jelly stool

🌹Intussusception case stable Best initial radiological reduction ( not I.v fluid bez he is stable)
….
Child cry when left her hip with mass in upper abdomen what’s Dx: intussusception

🌹Intussusception child first thing to do = Iv fluid & analgesia✅

🌹 13 month old with Abdominal tenderness,vomiting,bloody stool,leukocytosis and US doughnut shape.
intussusception

🍒 A 3 year old girl with bloody diapers. She has no pain or constipation. diagnosis?
A.Meckel's Diverticulum 🍒🍒 B. Intussusception C. Colon
.......
🌹 3 year old girl with bloody diapers. She has no pain or constipation diagnosis
= meckel diverticulum 🌹

🌹target sign by us= Intussusception 🌹
............
🌹Case of intussuption child very dehydrated what is the next immediate action? Iv fluid 🌹

🌹11 months presented with bloody smelling stool how yo confirm dx A. Us B. Barium enema

Intussception presentation and q is whats is the initial investigation: = ultrasound abdomin
‫ﺺ ﻫﺬا اﻟﻤﺮض‬¹‫ﺸﺨ‬î bd c ‫ﻘﺔ ﺟﺪااا‬¹‫ع ودﻗ‬ú‫ﻪ ؟ ﻻﻧﻬﺎ اﺳﻬﻞ وا‬¹‫ اﻟﺴﻮﻧﺎر ﻟ‬,
d ‫ ف اول اول اﺷﻌﺔ‬، ‫ﻈﻬﺮ ﻣﺜﻞ اﻟﺪوﻧﺎت‬¤
….
🌹case of intussuception came with clicky pain + doughnuat sign on ultrasound + bloody stool what is
most important step to manage rhis case ?
Ÿ Urgent surgery
Ÿ Nasogastric decomprestion
Ÿ I.v fluid resuction ✅
Ÿ Bariam enema
.....

🍓Case of baby 6 days passed stool after birth w constipation wts dx:
A. Meconium ileus B. Hirshpurge C. Volvolus✅
….
Abdominal distended, vomiting, picture of obstruction. Dx?
A- volvulus ‫ ﺻﺢ ﻻن اﻟﺴﺆوال ﻗﺎل ا*ﺴﺪادد‬B- ischemia. C- UC. D- Crohn
....
🌹Neonate with bilios vomiting He pass meconiun (Then he pass yellow stool)=>I think Dx?
A-Hirschberg dis B-Mid gut volvulus C-Meconium colitis
...
A child passed meconium within 24 hrs after birth. Two weeks later, child developed bilious vomiting,
abdominal distention and passage of pellet stool. What's the diagnosis?
A. Meconium plug. B. Midgut volvulus✅✅. C. Hirschsprung's disease. D. Intussusception
..... ...... ......
Child with bilious vomiting and pass limited amount stool. 5 days old
A.meconium ileus B. Volvlus ✅. C.Hirshsprung
...............
🍓baby 3-7 days presented with bilious vomiting , decrease oral intake , this happened after introducing
milk formula He passed meconium after birth and after that yellow stool :
A. Hirshbrung (no bez said pass mecnium at birth so exclud ) B. milk allergy C. Mid gut volvulus✅
….
🌹I got same picture exactly sigmoid volvulus

…..
Coffee bean sign and want the diagnosis? Sigmoid vulvulus
.....
Thumb print sign in abdomen and eant the daignosis ? Bowel ischemia

Abdominal distrnsion, constipation, vomiting. No Xray in choice .Investigation? A- CT
B- US✅ ‫”ء ﻫﻮ ا‘ﺲ راي‬ •
d ‫ﺎﻟﻄﻮارء اول اول‬- ‫ﻄﻦ ﻓﻌﻼ‬- ‫ﻻن‬
...... ....... ......
features of Primary sclerosing cholangitis What to do?
Colonoscopy = Primary sclerosing cholangitis is a chronic cholestatic liver disease. The majority of
patients with PSC have underlying inflammatory bowel disease . Patients with concurrent PSC and IBD have
an increased risk of colorectal cancer .
‫ﺎﻟﺘﻬﺎب اﻻﻣﻌﺎء‬- ‫ﻂ‬œ‫ﻢ ﻣﺮﺗ‬¤‫ﺪ دا‬œõ‫ اﻟ‬bd c ‫ ﻫﻮ‬Ûd‫ﺸﻔﻮا ﻫﺬا اﻟﻤﺮض ا‬è‘‫ ﻣﻨﻈﺎر اﻟﻘﻮﻟﻮن واﻟﺠﻮاب ﻫﻮ ا‬bd c ‫’ول‬i‫ﻠﺴ‬Ó‫ﺪ اﻟ‬œõ‫ﺶ دﺧﻞ ﻣﺮض اﻟ‬±‫ﺐ ا‬¹‫ﻃ‬
....... ....... ........
Child tripped on a toy and the right leg was trapped within the toy ,and fell on the leg child
complaining of pain ,what type of fracture do you expect:-
-Spiral facture of femur -spiral fracture of tibial ✅ -hairline fracture(not sure)
... ...... .. .....
🍓A child was brought by his parents after he refused to walk and insisted on being
carried always, the parents reported this happened after he was playing and
stepped on a toy and his leg was twisted and fell down = Toddlers =spiral fracture of distal tibia.
….
🌹Child with X ray of distal radial and ulnar bone fractures = Cast below elbow
‫ﻂ ﻣﻦ اﺳﻤﻬﺎ‬i‫ ر‬distal ‫ ﺗﺤﺖ‬È… „ ‫ﻌ‬v ‫ﺪ‬n‫ |ﻌ‬È… „ ‫ﻌ‬v elbow ‫ﺤﺘﺎج ارﻓﻌﻬﺎ ﻓﻮق‬v‫ة ﻣﺎ‬sz‫ﺪ ﻓﺎﻟﺠﺒ‬n‫ |ﻌ‬Ûr‫ ﻻن اﻟ‬elbow .
....
🌹 6 y.o child with fracture of thigh and 30% angulation. TTT?Hip spica with traction

🌹Child with forarm fracture , mx ? closed reduction and cast 🌹
..
🍓Distal radial fracture in peds patient (xray shown), partially penetrated the skin (picture). management?
A. internal fixation B. external fixation C. internal fixation with casting above elbow D. internal fixation
with casting below elbow ✅

Child fall from hight presented to you in ER crying, Bleeding from the ear, tympanic membrane bulging
and bleed , imaging confirmed basal skull fracture, The nerve which pass through foramen ovalea injured
whatʼs the function which will be affected ? Mandible nerve , mastication
‫ = ر•ﻂ‬Mandible = man
foramen = men = man = (man with men)
...... ..... ....
child with supracondylar fracture, distal pulse not palpable, your management:
A- K-wires. B- exploratory operation ✅✅. C- hand elevatio
..... ..... ...
xray of both bone distal forearm fracture, greenstick, your management:
A- closed reduction and cast )*+(exact pic

🍇A 3 y/o fell off his bunk bod and immediately cried, but ho didn't have an apparent loss of
consciousness. Brought to tho hospital and further Investigations show that there l$ no skull fracture.
What is your next action? . Watchful waiting🍏

Infant with absent red reflex ( retinoblastoma most likely) What to do:
A-Immediate referral to ophthalmology✅ B-Brain MRI
..
🌹what is diagnostic for retinoblastoma ? CT

🍒Retinoblastoma on slit lamp examination = urgent referral to ophthalmologist✅✅🍒

1inheritance
🌹 17 alpha hydroxylase deficiency= autosomal recessive
...
🌹child with 17-hydroxylase what type of inheritance?
A. multifactorial B. autosomal recessive C. autosomal dominant D. x linked
...
🍓Baby with ambiguous genitalia what deficiency: A. 17 hydroxylase B. 21 hydroxylase✅
...‫ ﷼‬21 ‫ = ھﻣﺑرﺟر 🍔🍔 ب‬ambig = ‫اﻣﺑﯾﺞ‬
🌹 inheritance of an ambiguous genitalia= AR (21 hydroxylase deficiency)

🌹 Pt carry risk of 25% to have genetic dis , What is the type of genetic abnormality ?
A. autosomal recessive B. autosomal dominant
......
🌹 Parents carrier 25% chance of having affected child with "Cystic fibrosis"= autosomal recessive
‫ﻂ‬i‫ =ر‬cystic = ‫ﺲ‬Ý‫ = ﻛ‬receive ‫ﺲ‬Ýr‫ اﺳﺘﻠﻤﺖ اﻟ™ﻀﺎﻋﺔ وﺣﻄﻴﺘﻬﺎ |ﺎﻟ‬:)
.....
🌹Child with cough,wheezing , recurrent infection, poor feeding and poor weight gain and murmur =Cystic
fibrosis
....

🌹Inherited of 21 hydroxylase deficiency autosomal recessive. R=2


....
🍓Congenital adrenal hyperplasia mode of inheritance? AR 21 hydroxylase deficiency
….
🌹case of Neurofibromatosis type 1 (7 cafe au liat spots, axillary freckles), ask mode of inheritance
:autosomal dominant
‫دوﻣﻴﻨﺎت‬. = ’c c ‫ ﻣﻦ دوﻣﻴ‬bd c ‫ﺎ‬È ‫’ي‬i‫ *ﺸ‬.
.......
🌹What is the type of genetics in pt presented with cafe aule spots dx neurofibromatosis?
A-AD✅ B-AR C-Xlinked AD
.......
🌹Child with multiple cafe au let spot on his body, his mother mentioned that his relatives also have the
same spots= consul about NF1
....
🌹4 year old boy brought by his mother examination reveals multiple Café au lait spots. The mother says
that “it’s a common birthmark in our family” what is your test action
A. Send her for genetic counseling B. Educate her on NF type 1 C. Confirm diagnosis with genetic studies D.
Reassurance
..
🌹case about wiskott-aldrich( recurrent infections, eczema, 2 healthy sisters, 1 died before the age of 10
months ask about mode of inheritance? x linke
‫ر•ﻂ‬
Died = X wiskot = ‫ = اﺳﻜﺖ‬X reccurnt infection = ‫ﻞ ﺷﻮي ﻋﺪوى ﻋﺪوى‬- = X
…..
🌹Case of 14m boy with a history of 4 lung infections, he has to healthy sisters. X-linked
agammaglobulinemia
….
🌹 Parent come with their child who have recurrent chest infection and they have another child who died
from one attack of chest infection ask about Dx ? -X-linked gammaalbuniemia
X-linked agammaglobulinemia (XLA) is a rare genetic disorder that affects the body's ability to fight
infection.
‫ ﻣﺎﻓﻲ اﺣد ﯾﻣوت ﻣﻧﮭﺎ اﻻ اذا اﻟﻣﻧﺎﻋﺔ ﺟدا ﺿﻌﯾﻔﺔ ﻣﺛل ھذه‬chest infection ‫ ﺛﺎﻧﯾﺎ‬x.linked ‫ ﻟذﻟك‬، ‫ﻓﯾﮫ ﻛﻠﻣﺗﯾن اوﻻ اﺧوه ﺗوﻓﻰ ﺑﺳﺑب اﻧﻔﻛﺷن ﯾﻌﻧﻲ وراﺛﺔ‬
........ ....‫اﻟﻣﺗﺎﻻزﻣﺔ ﯾﻛون ﻋﻧدھم ﺟﺳﻣﮭم ﻣﺎﯾﻘدروا ﯾﺣﺎرب اﻟﻌدوى‬
....
🌹-10 m old infant with pneumococcal infection and repeated infections. His brother died from severe
sepsis. on studies he has few B cells but normal T cells diagnosis=xlinked agammaglobinemia
….
🌹Case about a male with immunodeficiency has two normal sister andhx of one brother died due to
pneumonia.? X linked immunoglobulin‫🌹ﻣﻜﺮر‬
....
#Wiskott–Aldrich syndrome has an X-linked recessive pattern characterized by 3 thing : eczema +
thrombocytopenia + immune deficiency or upper resp infection
‫ﺴﻜﻮت‬8‫ ﻣﺘﻼزﻣﺔ و‬bd c ‫ﻌﺾ ﻧﻔﻜﺮ‬- ‫ﻠﻤﺎت ﻣﻊ‬È ‫ اذا ﺷﻮﻓﻨﺎ ﺛﻼث‬:
eczema + thrombocytopenia +immune deficiency or upper resp infection
‫ ﻣﻊ اﻛزﯾﻣﯾﺎ‬S ‫ = ﺣرف‬wisskott = .‫ ﻣﻊ اﻟﻘﺻﺑﺔ اﻟﮭواﺋﻲ‬o ‫ ﺣرف‬thrombo‫ ﻣﻊ‬T ‫ﺣرف‬
...
🍒Child with recurrent URTIs , eczema and thrombocytopenia both brother and uncle have the same
condition? Wiskott aldrich🌹
...... ....
🌹Wiskot aldreith syndrome= Dermatitis, thrombocytopenia, immunodeficiency
…..
🍓Child with tuberous sclerosis? A. Single gene testing B. Multiple panel gene testing ✅

🌹Tuberous sclerosis in child also his mother have it with TCH1 mutation what to order to child I think?
multiple gene screening

1Endocrine
🌹What is the recommended screening age for hemoglobin? A. 8 mo B. 12 mo C. 15 mo D. 24 m
..
🍇 eye pigment with green brown =Wilson's disease

🌹Mother is concerned about her 5 month old baby that he’s delayed developmentally how would you
relieve her concern? A. baby is waving hi B. baby is setting independently C. baby is holding objects

🌹congintal adrenal hyperplasia , now he is dehydrantion + slight low glucose what will give ? 3 thing (
normal salin + steroid + glucose ) 🌹

🌹High levels of 17-OH progesterone can indicate acongenital adrenal hyperplasia (CAH) =How to manage?
daily hydrocortisone orally
…..
🌹 Female child has several episodes of vomiting and enlarged clitoris on examination. studies are given
and show sodium 120 with other electrolyte
imbalance treatment = corticosteroids
( congenital adrenal hyperplasia case)
CAH crisis > the definitive ttt is hormonal replacement corticosteroids + mineralocortecoids

What is the classic cause of ambiguous genitalia on the Step 2 exam?
Adrenogenital syndrome, also known as congenital adrenal hyperplasia. Ninety percent of cases are
caused by 21-hydroxylase deficiency. Girls present as neonates with ambiguous genitalia. Boys present as
neonates with salt-losing adrenal crisis or as toddlers with precocious sexual development. Patients
with 21-hydroxylase deficiency have salt-wasting (low sodium), hyperkalemia, hypotension, and elevated
17-hydroxyprogesterone. Treat with steroids and intravenous fluids immediately to prevent death.

🍓 2 months old develop diarrhea (did not mention how many times or any other information) without
vomiting or any other symptoms ( did not mention the vital signs or the health status of the baby) The
mother concerns about dehydration management ? A. oral rehydration solution ✅ B. IV fluid

🌹Child was taking oral rehydration solution, present with mild dehydration, what is the reason of his
symptoms? A.lactose intolerance B.fructose intolerance C. glucose intolerance ✅

🌹Mother came with baby 12 months suffering from recurrent gastritis after introduce normal diet again
according to previous pediatrician give him oral rehydration Now baby came with same feature with mild
dehydration also =Oral rehydration for 24 day then give normal diet ( not for 5 days)

🍓pedia pt with polyuria , high glucose , what’s next investigation:=. Hb A1C

🌹Pediatric patient with classical symptoms of DKA + elevated blood glucose . What will you do next?
Urinedipstick ( ‫ﻴﺘﻮن‬õ‫ = ) ﻋﺸﺎن *ﺸﻮف اﻟ‬Dont choice Hba1c bez Q next step🌹

Child lossing 1kg despite he eating and drinking a lot , came dehydrated and irritable What's the
diagnostic test to the reach the diagnosis
A) glycosylated hemoglobin B) HLA-DR3 C) urine dipstick ✅✅. D) sorry forgot it the choice Child

🌹Pediatric presented with hyperglycemia and sx of diabetes In vs RR:60 What is the most important test
to do: Hba1c == Urinalysis 🌹 ( to detect the keton).

Diabetic mother, how to know if baby will be normal?
A- FBS B- post prandial C- HgA1C ✅

🌹Child diagnosed with T1DM screen eyes how often =after 5 years then annually 🌹

🌹Pediatric pt recently diagnosed with DM type 1 1 month ago compline on medication when to do
Opthamology screening Immediately =6 yrs

🌹When do screening for uveitis in SLE patients with (-) ANA. = 6m

child with type 1 DM, what he will use to control the disease? Regular insulin ✅ ✔

🌹6 years old K/c DM type 1 complain of hypoglycemia best TTT ? decrease Mixture insulin
Rapid ACTING INSULIN ANALOG before meal , no long acting NPH
/….
🌹Child didn’t eat anything last 3 h came with severe thirst . Fasting blood glucose 6.3 . Why he is increase
of glucose ? decrease insulin
…..
🌹child on glargine + aspart complain of fasting + postprandial hypoglycemia :reduce both

🍒🍒DKA During the management most important thing to be monitored?cerebral edema🍒🍒
….
🌹DKA treated but still have hypokalemia why ? ( vomting ) ( Dont choice insulin therapy or i.v fluid )

🍓7 y. Newly diagnosed DKA
A. go to DM group. B. write management plan in a paper. C. hear what the child said and its concern ✅
......
🌹child with DKA, PH 7.1 and glucose 20mmol. What is the initial mx step? Iv fluids
.....

child with Sx of dehydration and lethargy He also has fever. Rapid infusion of normal saline has been
started; after that, the boy start some abnormal movement and went into deep coma which lead the
doctors to intubate and transferred to PICU. Hypotensive, Tachycardic, Na 165, K 3.2, Cl 115. What is the
cause behind it?
A.delay in Abx treatment
B. rapid infusion led to cerebral edema ✅✅
C.intraventricular hemorrhage due to the high sodium.*
….
Child with dehydration, depressed anterior fontanel, and decreased skin turgor. What is the percentage of
dehydration? А. 5. В. 10✅. С. 15. D. 20
if asymptomatic> mild> 1-5%
if more > moderate> 6-10 %
if severe (hypotension)> 15 %

🌹 in children of diabetic mothers, glucose 12.5% is given in? A. central line B. peripheral line C. NGT
....

🌹Newborn with hypoglycemia what is the route of 20% dexterous? Central line ( 20 % tacke by central ,
if say 10% priphral =
I =10
.......
🌹Child came with hypoglycemia what is the infusion rate ? . 10% dextrose + 10ml/hr✅

🌷Treatment of hypoglycemia 🌷:
• Dextrose given peripherally in D5 and D10 but centrally in D12.5 and D15 and D20.
‫( اﻻﺑرة اﻟﻛﺎﻧﯾوﻻ ﻧرﻛﺑﮭﺎ وﯾن ؟ ھل ﻓﻲ اﻟﯾد وﻻ ﻧرﻛﺑﮭﺎ‬dexstros) ‫ھذا ﻣﻌﻧﺎھﺎ ﻟﻣل ﯾﻛون اﻟرﺿﯾﻊ ﻋﻧده ﻧﻘص ﻣﻣﯾت ﻓﻲ اﻟﺳﻛر وﻧﺑﻐﻰ ﻧﻌطﯾﮫ ﺟﻠوﻛوز‬
‫ ف ﻋﻠﻰ ﺣﺳب اﻟﺟرﻋﺔ اﻟﺟﻠوﻛوز اﻟﻲ ﺑﻧﻌطﯾﮫ ﻟﮭم اذا‬. central line ‫ اﺳﻣﮫ‬femoral ‫( ﯾﻌﻧﻲ ﻋﻧد‬:‫ﻋﻧد ورﯾد ﻛﺑﯾر ﻣﺛل اﻟﻲ ﯾرﻛﺑوﻧﮭﺎ ﺗﺣت اﻟرﻗﺑﺔ او اﻟﻔﺧذ‬
. ‫ اﻻرﻗﺎم ﻣﮭﻣﻣﺔ ﻧﺣﻔظﮭﺎ ﻋﺷﺎن ﻧﺣدد اي ورﯾد‬. ‫ ﻧرﻛب ﻓﻲ ورﯾد اﻟﯾد‬١٠ ‫ او‬٥ ‫ اذا اﻗل‬، ‫ ﻓﻣﺎ ﻓوق ﻧرﻛب ﻟﮭﺎ ﻋﻠﻰ ورﯾد ﻛﺑﯾر‬١٢

• We start ttt of neonatal hypoglycemia by inserting 2ml/kg of D10 *peripherally*


• If no response, persisted hypoglycemia so consider 12.5D through *central line*

• So initially it is peripheral line, not direct central line


....
🌷Dextrose given peripherally in ===D5+D10 but centrally in ===D12.5 and D15 and D20.🌷
‫ﻪ‬¹‫ ﻧﻌﻄ‬٢٠ ‫ او‬١٥ ‫ او‬١٢ ‫ ف ﻣﺜﻞ ﻣﺎ اﺗﻔﻘﻨﺎ اذا‬centrall ‫ ف ﻧﺤﺎول ﻧﺤﻔﻆ اﻻرﻗﺎم‬.
central .‫ داﯾﻣﺎ‬١٢ ‫ داﯾم ﯾﻛون ﻓﻲ وﺳط اﻟﺳﺎﻋﺔ وﺑﻌد‬١٢ ‫ = وﺳط واﻟﺳﺎﻋﺔ‬central ‫رﺑط‬
.... ..... .......
🍇Diabetic kid type I diagnosed since 3 years he is follow g growth with same sex
and age what an annually screening should be done ophthalmology

Diabetic child with growth delay .. what check annually:
A- Ophthalmology ✅ ( dispetic retinopathy) B- Growth hormone
Everyone living with diabetes over the age of 12 will get an invite to a regular eye screening. At first the
screening will be every year.

🌹When to do ophthalmology screening for child with DM type I
A-lmmediatly B-After one year C-After 3 years D-After 5 years ✅
....
🍓newborn with one umbilical artery , what’s the cause: mother with DM🍓

7 years old child brought to ER with DKA. What is the best to do after ER treatment?
A- Listen to his conserns about DKA and its management B- Supply child and family with a written plan of
care ✅ C- Send the Child to special care services
N.B: They didn't mention history of DM I think It's first diagnosis In this case ) B sure.
….
🌹18 months girl with asymmetrical breast enlargement other examination normal =
Abdominal US ( Dont choice C.T pelvic or Brain MRI) (if there is GN Rh stimulation test choice it)
….
🌹mother complains her daughter is less than her age +parent or one of then short. all lab results normal
except insulin growth hormone is low = growth hormone defincy

Child 9yrs old came with his mother because she thinks he is short stature. mother is short. When hand
bone examiend revealed age of 7 years Investigation All normal including growth hormone. Except insulin
like growth hormone was 18 low, What is diagnosis?
A. Chronological B. Failure to thrive C. Constitutional D. Growth hormone deficiency✅
.........
🌹9 y/o child brought by his mom who’s concerned about his short stature. His friends make fun of Him,
Investigations show bone age of 7 years and low insulin like growth factor (IGF-1). How would you treat
him? GH (children with GH deficiency , we recommend treating with recombinant GH rather than no
treatment
.....
🌹 Pt 6 years old with very low weight (25kg), everything is normal in examination and lab, except for low
IGF-1, what you will do: A. give growth hormone
..
Child with short stature...parents concerned whether he will remain short in the future as well !?
What is the most important thing in history that would determine your answer
A-sibling hight B- parents hight✅✅ C- wt in relation to his hight and age
...
🌹 Asymmetrical kidneys size on us? A- PCKD .polycystic kidney disease
….
girl 7 years old has pubic hair, developed breast ,ask about which kind of puberty?
• central precocious puberty ✅✅ • ovarian tumor • central lesion • gonadotropin adenoma
Precocious puberty is when the signs of puberty start: before age 7 or 8 in girls. before age 9 in boys.
.... ..... ....
5 yrs female with pubic hair , no clitoromegaly obese, hight above 90 centile ?
A- Testosterone. B- Dehydroepiandrosterone Sulfatedehy ✅. C- 17 hydroxylase. D- Lh ? Or fsh ?
Dehydroepiandrosterone Sulfatedehy also known as androstenolone, is a male sex hormone (androgen)
that is present in both men and women,
When To Get Tested ?‫ﺎس ﻟﻠﻬﺮﻣﻮن‬¹‫ﻜﺮ ﻧﻌﻤﻞ ﻟﻪ ﻗ‬œ‫ﻠ&غ اﻟﻤ‬œ‫ﺾ ﻋﻨﺪە ﻋﻼﻣﺎت اﻟ‬ñÂ‫ﻜﻮن ﻣ‬¤ ‫ﺔ ﻫﺬا اﻟﻬﺮﻣﻮن ؟ ﻟﻤﺎ‬œ‫ﺴ‬ö‫ﺎس ﻟ‬¹‫„ ﻧﻌﻤﻞ ﻗ‬i‫ﻣ‬
When a girl or woman has excess facial and body hair (hirsutism), or when shows signs of very early
(precocious) puberty such as deeper voice, pubic hair, or muscle development
….
7 y old female started breast development, and pubic hair start to appear and acne >> dx.
A- ovarian tumor, B- central ✅ C- gonadotropin
Precocious puberty is when the signs of puberty start: before age 7 or 8 in girls. before age 9 in boys
The onset of puberty is normally triggered by the hypothalamus. This area of the brain signals the pituitary
gland،Most commonly, especially in girls, precocious puberty is due to the brain sending signals earlier
than it should
‫ وﺑﻌدﯾن ھﻲ طﻔﻠﺔ‬ovarian tumor ‫ وﻟﻛن اﺑدا ﻣﺎﻟﮫ ﻋﻼﻗﺔ ب‬pitotary ‫ ﻣﻣﻛن ورم ﻓﻲ ﻏدة‬central ‫ ﯾﻌﻧﻲ ﺷﻲء‬brain ‫ﻓﺎﻟﻣﺷﻛﻠﺔ اﻟﺑﻠوغ اﻟﻣﺑﻛر ﯾﻛون ﻣﺷﻛﻠﺔ ﻓﻲ‬
.‫ ﻣﺎﯾﺟﻲ اﻻ ﻣﺗﺎﺧر ﻣﻊ اﻟﻛﺑﺎر ﺑﺎﻟﻌﻣر‬ovarian ‫ﺳرطﺎن‬

..... ..... ....


🌹2 y/o developing breast which case ? premature thelarche 🌹

🌹Turnner stage 5 breast and pupic hair = Delay causes = consitutional delay🌹

🌹 7Y tanner stage 5 (breast, pubic hair,acne) type of puberty? Precocious puberty 🌹
….
🌹7 year old with pubic hair, no axillary hair, no breast or mensis? Precocious puberty ( Not adrenarche
Bez adrena means > axillary and pubic hair and this case no axilla hair ) 🌹

🌹9 y/o boy, her mother concern about short stature, the investigation result the bone density for age 7
years= constitutional ( not Growth hormone)
.....
.. . . .. . .
🌹male with type hair and dark secrtum = tanner stage 4 🌹

🌹Boy with pubic hair towards adult distribution and darkening of scrotal skin. Tanner stage? A.II B.IV
C. V
..
🌹13 years old brought by her mother concerned about her stature, patient is normal, on examination, no
signs of breast development and no pubic hair, what is the cause? Constitutional
....
🍓Mother came with her 9 years old complaining about his height. She's short (didn't specify how short),
Patient is 25kg and 120cm, His bone exams revealed that his actual bone age is 7 years, Diagnosis:
A. Constitutional✅. B. Failure to thrive
....
🍇Case of gonadal agenesls she was 17 years no period mInImal development of breast with axlllary and
pubic hair Outflow obstruction or mullarian agenesis = pelvic US
....
🍇17 years old boy with unilateral gynecomastia: Reassure, it will disappear later
....
🌹 9 days newborn come with jaundice only in the face not not extended to the rest of the body..
otherwise he is healthy was delivered by NVD with no completing.. and he was breastfeed immediately.
what is the cause of his jaundice? breastfeeding jaundice
....
🌹44-year-old male married for three years complaining of decreased libido painful tender breasts by
lottery bilaterally breastmilk can be expressed manually he came in for an evaluation of his complaints
physical examination is otherwise normal prolactin level is very high what investigation would you like to
do? A. brain MRI(tumor pitotary) B. Ct scan C. Adrenal levels D. Abdominal us
....
🍓2 years old her mother noticed development of the breast no other sign of puberty
A. precocious puberty B. premature puberty C. premature breast development ✅
No premature thelarche in the choices
Premature thelarche is the term we use for girls who develop small breasts (often an inch or less across),
typically before the age of 3 years. Girls with premature thelarche do not have other signs of puberty.
.........
4 months on breastfeeding, This is her first baby came with 2 days hx of lethargy constipation, fever,
response weak when light directed to his eyes , cause ?
A. Hypothyroidism B. Infantile botulism C. Guillain barre D. Poliomyelitis
‫ ﺣﺗﻰ ﯾﻛون ﻋﯾﻧﮭم ﻣطﻔﯾﺔ ﻣرة‬botulsim ‫ وھذا ﺑﺎﻟﺿﺑط ﯾﻣﺷﻲ ﻣﻊ‬pupillary paralysis ‫ ﯾﻌﻧﻲ‬light ‫ و ﻣﺎﯾﺳﺗﺟﯾب ﻟل‬fever ‫ﻗﺎﻟك ﻋﻧده‬
botlism‫ اﻟطﻔل اﻣﮫ اﻛﻠﺗﮫ ﻋﺳل ھذا ﯾﺟزم ﻟك ﻣﺋﺔ ﺑﺎﻟﻣﺋﺔ ﻣرض‬baby eat Honey) ‫ وﻟو ﺷوﻓﺗوا ﻛﻠﻣﺔ اﻧﮫ‬ptosis ‫ﯾﻌﻧﻲ‬
....
🌹25 y/o primigravida , baby flat face , no smile : A- infantile botulism B- congenital hypothyroidism
….
child has leg abnormal shape and delayed walk, ca high ,alkaline phosphatase high ,normal Pos ?
A- rickets✅✅. B- familial hypophosphatemia ricks. C- renal dystrophy , D- hypophosphatemia
….
🍓A child (can't recall the age) presented with lower leg long bones angulation. Labs show high CA and
low phosphate. X ray shows distal bone hypertrophy
A. rickets ✅. B. familial rekits. C. renal osteodystrophy
.......
🍓child (can't recall the age) presented with lower leg long bones angulation. Labs show high CA and low
phosphate. X ray shows distal bone hypertrophy: rickets✅. familial rekits. renal osteodystrophy

Baby delivered at home presented 65 days later with rt thigh bruises other exam
unremarkable ( PT high , PTT high , other normal ‫? د‬A. hemorrhagic diseases of newborn✅
….
🌹Pt 5 days old baby with thigh bruises, home delivery, lab : high pt and ptt Dx: ‫ي‬
A. hemorrhagic disease of newborn. B. abo incompatibility C. Rh incompatibility
‫ ﻣن اﺳﻣﮭﺎ واﺿﺢ‬A ‫ اﺧﺗﺎروا‬bruses ‫… ﺗﺷوﻓون ﺑﺎﻟﺳﯾﻧﺎرﯾوا طﻔﻠﮫ ﺗوه ﻣوﻟود وﺣﺻل ﻟﮫ‬

Jaundice
🌹5 day with jaundice, mom said that his brother also was same sx in last delivery, what most important
question in history: = Mother blood type.

child was delivered, and he developed jaundice on a 3rd day. He was treated for physiological ‫ز‬jaundice
but 2 weeks later jaundice became progressive with associated pale stool. Investigations
done and showed: Total bilirubin high, Direct bilirubin high. What's the diagnosis?
A. ABO incompatibility B. G6PD deficiency C. Rhesus is sensitization D. Biliary atresia✅✅
Biliary atresia is a condition in infants in which the bile ducts outside and inside the liver are scarred and
blocked. Bile can't flow into the intestine, so bile builds up in the liver and damages it. The damage leads to
scarring, loss of liver tissue and function, and cirrhosis.
….
🌹8 weeks with president jaundice not relive by photophobia , with elevate ALP = Biliary atresia ✅🌹
‫ وﻻن طﻔﻠﮫ ھذا اﻟﻣرض ﻏﺎﻟﺑﺎ‬jundice , ‫ ﻓﻲ اﻟدم ﻓﯾظﮭر اﻟﻣرﯾض اﺻﻔر ﯾﻌﻧﻲ ﻋﻧده‬billuropin ‫ ﻣﺎﯾﻘدر ﯾﻌﺑرھﺎ ﻓﯾﺗراﻛم‬bile ‫ﻟﻣﺎ ﯾﺣﺻل ﺿﯾق ﻓﻲ اﻟﻘﻧوات ال‬
‫ﯾﻛون ﻣﻌﮭم‬
. . .. ... .
Infant 2 weeks old. On examination he is jaundice, has large fontanel, cold extremities + hypotonia + large
toung to his mouth. What’s likely diagnosis?
A- Galactosemia. B- Bitouriceye. C_congenital hypothyroidism✅
hyperbilirubinemia, the raised with _congenital hypothyroidism = jundice
🌹large fontanel, cold extremities + hypotonia + large toung to his mouth + jundice
(hyperbilirubinemia)== Typical sign of congenital hypothyroidism🌹

.......
🌹2 month old mother notice bulging tongue , dry mouth, constipation , fhx of autoimmune dis , tSH high
22 , T3 low T4 low what mx: A. Give 1 month levothyroxine. B. give life long levothyroxine. C. repeat in 1
month
🌻Therapy for both acquired and congenital hypothyroidism consists of lifelong treatment with
levothyroxine (L-thyroxine) and regular check-ups to monitor disease activity.
...
🌹Baby 2 mo old with jaundice, floppy, protruding tongue there is positive family history of autoimmune,
labs show low thyroid hormone What is the treatment Thyroxine for life.

.........
🌹Newborn what is the SINGLE investigation you must to do? Thyroid function test🌹
‫ﺎ ﺗﻪ‬n‫ﻄﻠﻊ ﻣﺘﺨﻠﻒ ﻃﻮل ﺣ‬¤ ‫ﻪ واﻋﻄﻴﻨﺎ ﻟﻪ ﻋﻼج راح‬¹‫ﺪ وﻣﺎ ﻟﺤﻘﻨﺎ ﻋﻠ‬ñ‫ اﻟﺜﺎﻳﺮو‬bd c ‫ﺎن ﻋﻨﺪە اﻧﺨﻔﺎض‬È ‫ ﻻااازم اي ﻣﻮﻟﻮد ﻧﻌﻤﻞ ﻟﻪ وﻇﻌﺌﻒ اﻟﻐﺪة ﻻن ﻟﻮ‬.
‫ﺐ‬é‫ﺎﺧﺘﺼﺎر اﻟﺴ‬- ‫ﻫﺬا‬. ‫ﻞ ﻧﻌﻤﻠﻪ‬¹‫ﻪ ﻫﺬا اﻫﻢ ﺗﺤﻠ‬¹‫ ﻋﺮﻓﺘﻮا ﻟ‬.
.... .. ....
🍒Newborn developed jaundice in first 12 hours , labs Low hb “9” Which test to order
A. HB electrophoresis. B. G6pd deficiency test. C. Fragility test✅🍒 D. Tyrosine kinase
......... .........
🍒🍒Child with jaundice investigation low he, high total bilirubin, high indirect bilirubin,positive direct and
indirect comp test what is diagnosis = A.Spherocytosis B. Autoimmune hemolytic anaemia ✅✅
…..
🌹11 years old with with jaundice Lab test: Increase indirect bilirubin Increased total bilirubin Increase all
the lft ? A- obstructice B- gilbert C- autoimmune hepatitis
...
🌹What indicate hemolysis ? A. increase conjugated B. Increase unconjugated
🌻in extravascular hemolysis plasma levels of unconjugated bilirubin increase because the hepatocytes
cannot process the excess bilirubin.
….
🌹5d /o baby be jaundice asked about important q ask in Hx ??
A. APGAR score or B. mother transfused blood C. blood group ✅ 🌹
….
🌹16/ old with fever and RUQ pain and jaundice, No lab or radiology available:
A. Reassure and send home
B. Emergency consult a surgeon on phone
C. ask for lab results & tell him to come back tomorrow❌
D. admit to hospital & evaluate✅
Answer is: B, CHECK NOTES
Note: should be admitted and investigate since its risk of septic shock.Then IV abx
followed by ERCP
……..
Pedia 4 days with jaundice and his brother had the same thing, Direct bilirubin and total bilirubin was high
what is the diagnosis: choledochal cyst
....
🌹Child 12 h , have jaundice , HCT high , pic of spherocytosis , what is the test u will do ? osmotic fragile
..........
🍓Pedia 4 days with jaundice and his brother had the same thing Direct bilirubin and total bilirubin was
high = 🍓biliary atresia
........
🌹A newborn got jaundice and the mother mentioned that the other children had to get plasma
exchange or blood transfusion after the birth what is the important inv to do ? Mother blood group .
.......
🌹8 weeks old baby with jaundice, not responding to phototherapy, what is the cause?
A. ABO incompatibility B. billiary atresia ✅
‫ﻂ‬i‫ = ر‬photo = ‫ﺮ ﺻﻮرة‬̵‫ = ﺗﺼ‬Biliri = ‫ﻣﻦ‬By = ‫ اﺧﺬ ﻟﻚ ﺻﻮرة‬È… „ ‫ ﺧﻠﻴ‬æ ¿
… ‫ ﻗ™ﻞ ﻣﺎ ﺗﻤ‬.
.........
🌹 Pregnant with Rh-negative blood type her baby have Rh-positive blood type present with jaundice ask
about Pathophysiology =
autoantibodies against fetal RBCs (Mother’s antibodies attack fetus RBCs)
....
● Child k/c of SCA with pic chest X-ray? A. Acute chest syndrome

🌹Boy complaining of RUQ pain, he had a history of URTI couple of days ago, cbc shows low hgb and
increased retics, smear shows target cells and inclusionbodies, dx? SCA

🌹Sickle cell anemia child hb 3 severe pallor and long history given what mostly causes this:
a. Parvovirus B19✅ b. CMV. Aplastic crisis everything is low > triggered by parvovirus b19

🍒🍒SCD patient came with crises, What’s the appropriate next step to reach diagnosis (to differentiate
between aplastic and splenic sequestration?A. Reticulocytes B. CXR. C. US

🌹Patient came with sudden pain in the hands and feet what’s the most likely diagnosis? SCD

🌹 Boy complaining of RUQ pain, he had a history of URTI couple of days ago, cbc shows low hgb and
increased retics, smear shows target cellsand inclusion bodies, dx? SCD

Prevent ACS in SCA ? hydroxyurea


..
🌹Pic of CXR of rt lobe consolidation With long scenario of SCD pt presented with chest pain and dyspnea
and back pain, What is the Dx? Acute chest syndrome
....
🌹What is the medication will prevent these symptoms(Acute chest syndrome ) for the prev Q?
Hydroxyurea
....
SCD young patient came to the ER with toxic looking, enlarged liver andbspleen, drop in Hgb?-
sequestration crisis
.........
🌹A patient with acute chest syndrome and upper/lower limbs vaso occlusive crisis, what’s the effective
drug proven to reduce the frequency of painful crisis? Hydroxyurea🌹
... .
🍇Child with SCD most Important long tami treatment: Hydroxyurea

🍇Child with sickle cell anemia presented with shortness of breath and chest pain on
is the best initial step in A.lv fluid and analgesic✅

🍓6yr Sickler with fatigability for 2days. found anemic Spleen 6 Cm below costal margin
A. multiple transfusion B. Splenectomy✅ but hydration and transfusion first

🍒patient with a decrease in all cell line WBC, Hgh, and plt (labs) asking about dx?
Aplastic anemia🍒🍒
🌹Case of child studies given microcytic anemia he has high Hba2 on electrophoresis what type of anemia
Beta thalassemia minor
✴ Hg A2 higher in minor b thalassemia
✴HgF in major thalassemia
‫ ر•ﻂ =ﺣﺮف‬F = ‫’ة‬ª‫ﺎرﺛﺔ ﻛﺒ‬È =major
...
12M fatigue, failure to thrive, stunt growth, large forehead or something?
A. beta-thalassemia ✅ B.alpha-thalassemia C. SCA D. IDA

🌹Child lab showed microcytic and hypochoromic anemia with reticulocyte count high (2%) , ferritin
normal, and his 2 siblings have the same presentation what is the diagnosis ?
A. Sickle cell disease B. IDA C. Alpha Thalassemia trait
..
🌹Child pale and lethargic, with no specific S/s , labs shows only microcytic anemia (low hb,lowMCV) what
is the diagnosis?A. thalassemia trait B. lead poisoning C. SCD
….
....
🌹pale child came with MCV : 68 + Lowe plattlet + low ferritin and have 2 siblings with same condition =
Thalasemia minor ( Dont choice iron defincy anemia)
‫ وﻫﻨﺎ ﻗﺎل اﻧﻪ ﻋﻨﺪە اﻗﺎر•ﻪ ﻧﻔﺲ اﻟﻤﺸ•ﻠﺔ وﻋﻨﺪە اﻟﺼﻔﺎااﺋﺢ‬، ‫’ة‬ª‫ﺎ اﻟﺤﻤﺮاء ﺗﻜﻮن ﺻﻐ‬¤‫ ﻻن اﻟﺨﻼ‬، ‫ﺪ‬¤‫ﺎ ﻧﻘﺺ اﻟﺤﺪ‬¹‫ﻤ‬¹‫ ﻣﻌﻬﺎ اﻧ‬C
d … ¤ ‫ﺎ‬¹‫ﻤ‬¹‫اﻟﺜﻼﺳ‬
c
‫ﻤﺎ‬¹‫ﺴ‬D‫ﺎ ﺛﻼ‬œ‫ ﻓﻐﺎﻟ‬، ‫ﻌﺾ‬- ‫ ﻣﻊ‬ݪ‫ﺪد اﻻﺛﻨ‬¤‫ ﻧﺎزﻟﺔ واﻟﺤﺪ‬.
......
🌹Child ingest nutritional supplement then black stool = iron
....
🌹child ingest iron tablets and come with symptoms= IV deferoxamine

🍓child 4 years old loss of weight for long time , most comon cause in this age :
A. lymphoma. B. leukema✅✅ (2_8 years), C. retinoplastoma. D. neuroplastoma

🌹 Child eating paper = IDA🌹


….
🌹 58 days baby have G6PD they give lab value low HB level : what are the causes of this:
A. anemia of chronic disease. B. hemolytic anemia
..
🌹 2 yo girl with diarrhea and dehydration , splenomegaly , Hb is low , direct and indirect coomb is +ve , Dx
? A. spherocytosis B. fanconi anemia D. G6PD C. Autoimmune hemolytic anemia✅ .
..
🌹 Ptn came for routine F/U Lab show: High RBC low HB Low MVC Normal Reticulocyte A-SCA B-G6PD C-
Anemia of chronic disease D-hemolytic anemia

Female had son with SCA. Remarried and for screening : A. Parental B.Mother C. father ✅

🌹 Can be cured by splenectomy?
A. scd B. a thalassemia C. b thalassemia Answer is: ITP, Hereditary spherocytosis

🌹 What is treated after splenectomy?
A. Alpha thalassemia, B. beta thalassemia, C. sickle cell trait D. ITP
..
🌹Case of child studies given microcytic anemia he has high Hba2 on electrophoresis what typeanemia ?
beta thalassemia

...
🌹Baby born a few days ago at home presented with multiple bruises, diagnosis?
A. Von Willebrand. B. neonatal hemorrhage dis.✅✅٣

🍒🍒A child with ALL came to the ER with febrile neutropenia, management? All septic workup with IV
ABx🍒🍒
...... .....
🍒 A young boy came with hemarthrosis asking about dx? Hemophilia🍒
..
🌹child with hemarthrosis = Hemophilia

🌹Direct and indirect coombs test are positive: immune hemolytic anemia

🌹Classic ring-shaped/headphone-shaped trophozoites are seen in case of Malaria = Plasmodium
falciparum infection


🌹Pt with pallor (anemia) with splenomegaly Lab: high retic. Blood smear: microspherocytes ++
Anisocytosis + Dx? A. sickle cell anemia. B. hereditary spherocytosis
‫ﻢ‬è‫ ﻣﻦ اﺳﻤﻬﺎ ﻗﺎﻟ‬microspherocyte .

Syndrome
🌹 prominence occipital, rocker bottom feet , cardiac = Edward’s syndrome

🌹Baby born with many Edward syndrome feature (rocker foot, etc) genetic
testing asking about possible diagnosis based on dysmorphic feature:. Edward

.....
🌹 long case of child doesn’t control his hungry : .Prader-Willi syndrome
….
- Obese child mother complaining of hyperphagia on examination he had dysmorphic features +
hypotonia + ascended testes. What is the most likely diagnosis? -Prada
classic sign of Prader-Willi syndrome is a constant craving for food, resulting in rapid weight gain, starting
around age 2 years. Constant hunger leads to eating often and consuming large portions
….. sz‫ﻴ‬é‫ﻞ ﻛﺜ‬ê‫ﻞ ا‬n‫ﺸ‬Ð ‫ =ﺑﺮاد =ﺑﺮادة اﻟﺜﻼﺟﺔ‬sz‫ﻞ ﻛﺜ‬ê‫ﻂ =ﻓﺠﺎﻋﺔ =ا‬i‫🍅🍒🍑🍅🥑🥥🍆 ر‬
……………..
🌹Child eats alot and he is obese with undecended testis , facial malformation and cleft plalte? pradder
willi syndrome
….
2cases Turner syndrome (one with primary amenorrhea and other phenotypical features and second one
presenting with short stature and the typical phenotype) ✅
‫ ﻣﺘﻼزﻣﺔ‬Turner syndrom ‫ﺘﺎن‬â‫ اﺳﺎﺳ‬Ý c ª ‫ﻠﻤﺘ‬È ‫ ﻟﻬﺎ‬:
1_amenohhria 2_ short stature ’ª‫ﻃﻮﻟﻪ ﺻﻐ‬
........
🌹 17 years old hypertensive not menstruating yet at clinic by her parents ,she is short stature , short
neck Most appropriate diagnosis:- turner ✅

.......
Parents are worried about the hight of their child on examinations the child look normal with deprsed
nose and short neck and large tongue what is the cause of his short stature ?
A.constitutional B. pubertal C. syndromale ✅✅

🌹 Tall thin child patients (above the 95th percintile) and has flexible joints and pectus excavatum
diagnosis? A-turner == short B-marfan == Tall + Thin C-klinefelter D-pertman
….
down syndrome: most common cardiac anomaly associated with DS?: -Endocardial Cushion Defect ✅
Endocardial cushion defects, more commonly known as atrioventricular (AV) canal
.. ...... .....
🍒 Down syndrome endocrine association = Hypothyroidism = 🍒
‫ = داون =اﺳﻔﻞ‬hypo
. . . .. . . ..
Common valvular heart disease in down syndrome? Endocardial cushion defect .(AVSD)

child with Down syndrome came with fixed S2, ejection systolic murmur and enlarged ventricles. What is
the most likely diagnosis? A.VSD B. AVSD ✅✅
….
🍓 Chart type for down syndrome? A. down syndrome growth chart

🍓Pic down syndrome?Trisomy 21🍓
….
🌹Low incidence in down syndrome: Mosaicism
‫ اﻟﻤﻮز ل اﺳﻔﻞ‬²•‫ ر•ﻂ =ﻣﻮز 🍌 ﻧﻘ‬dowen
.........
🍇 investigation of down syndrome?high Bhcg, high inhipin, low AFP, low estradiol.
..
Mother 27 years I think had Down syndrome baby what Increases hor risk for
having another baby with Down syndrome = A. age B. father chromosome C. mother chromosomes
….
Infant months of age died , they took hx from the parents ( the infant was preterm, problem with lungs ,
parents are heavy smokers and he was sleep with them ) what is the cause of death ?
A- acute respiratory failure B- sudden death infant syndrome✅✅ C-o2 insufficiency

Sudden infant death syndrome (SIDS) is the unexplained death, usually during sleep, of a seemingly healthy
baby less than a year old. SIDS is sometimes known as crib death because the infants often die in their
cribs. Although the cause is unknown, it appears that SIDS might be associated with defects in the portion
of an infant's brain that controls breathing and arousal from sleep.
….
2months old infant was found dead by his mother. Mom said he was okay without any obvious thing. On
examination, no signs of fractures, bruises, or abuse. What is the important part of history to be asked?
A. social history (think about sudden infant death syndrome >> smoking) ✅
B.history of medication used by the mother during pregnancy
,….
Which of the following decrease RDS incident the most? A. parents not smoking near their infant✅
..
Baby 2 month died in his bed no other significant problems what is important in ?
a-Allergy ‫ﻌﺪوە‬œè‫„ اﺳ‬d c ‫ﻌ‬¤ ‫ ﺗﻮە ﻣﻮﻟﻮد‬b-Maternal drug used in pregnancy ‫ﺐ ﻣﻮت ﻣﻔﺎجء‬é‫ﺴ‬±‫ ﻣﺎ‬C. Social hx✅ ‫ﻤﻜﻦ‬¤

......
🌹 15 year complaining she doesn’t have menstruation yes on examination she is short with short neck
and posterior hair = Turner syndrome
.....
-coarctation of the aorta associated with a. Turner✅ b. down
..
-Girl with short stature webbed neck.. diagnosis a. Turner✅ b. familial

....
🌹Clinical finding in pt with turner:
A. hypertonia + ankle clonus B. fold skin at nape of neck C. single palm crease

🌹17 years old medically free brought to Gynecology clinic by her mother with
history of no menstruation. On examination there was low hairline, high BP and short stature. Both
mother and father were having short stature at her age.
What is the most likely diagnosis? Turner

🍓13 years old boy presented with cushing syndrome symptoms such as central obesity and striaLab test :
high cortisone in night and salivary ACTH What next steps: MRI pituitary✅. CT abdomen
.......
🌹 child present with central obesity + moon face + striat investigation? ACTH stimulations test

🍓Signs of turner syndrome=webbed neck.
........
🌹Best lab to dx Turner syndrome= karyotype if there is = FSH choice it .

x15 y/o girl came with her mother complaining of never menstruated .. p/E ..
short stature + short neck + hypertension Parent stature below average Most likely dx?
A-Hypothyroidism B-Hypo pituitarism C-Turnur syndrome D-Familial short stature
..
🌹Case of digeorge syndrome( facial features, recurrent infection and tetany) ask about vaccines not to
give before Dx? all live vaccines
....
🌹8 years old boy don't understand in school and have many involuntary movements, he is aggressive ..
Dx = There was no ADHA in choices
Note: DD: Tourette syndrome,if with communication impaired or Lesch-Nyhan Syndrome.
.....
🍓Dtap contraindications?
A. Seizure B. Encephalopathy within one week that couldn't be attributed to another cause ✅✅

🌹 Malnutrition of african boy with Acitis or edema A. Kwashiorkor B. malesmas C. muscle atrophy
.....
🌹Child in poor area with central edema Muscle wasting = Kwashiorkor
‫ﻀﺎوي داﺋﺮي‬¹‫ﻠﻬﺎ ﺷ•ﻠﻬﺎ ﺑ‬È ‫ﻮا‘ﺐ‬õ‫’ =ﻛﻮا‘ﺐ =اﻟ‬ª‫ = ر•ﻂ =ﻛﻮاﺷﻜ‬Edema .
....
🌹Pt child with brittle hair and abdominal destination diagnosed with kwashiorkor cause? Protein
malnutrition
..
🌹 Pt with ascites and edema, severe muscle wasting, hyperkeratosis, lab show:
slightly decreased glucose, significantly decreased albumin, your Dx:
A. severe protein deficiency (kwashiorkor) B. severe carbs deficiency (marasmus)

🌹Malnutrition of african boy with Atrophy everywhere = marasmus



🍇Muscle wasting low protein = marasmus
....
🌹mumps in child most affect = parotid gland🌹

most common affected organ in pediatric with mumps: A- testes. B- parotid ✅✅
........ ...... ....‫ اﻛل = ﺑﺎروﺗﯾد ﻗرﯾب ﻣن اﻻﺳﻧﺎن ﻟﻠﮭﺿم‬mum = ‫ ﻣﺎم‬: ‫رﺑط‬

1Tumor
......
🍇4 year old boy most common cancer == leukemias
….
🌹child has swelling and redness above knee ( femur)=osteosarcoma

🌹Orthopedic tumor in pedia there is No limitation of movement? osteosarcoma

🍓Which lymph node indicate malignancy ? A. Cervical B. Supraclavicular✅ C. Submandibular
......N.B: gastric cancer

🌹lung Ca, sudden onset of back pain, neurological Ex is normal ,ttt? Steroid then MRI

🌹Pt diagnosed with small cell lung cance, presented with dehydration, serum osmo low, urine osmo
high. ‫ ﻫﺬە اﻋﺮاض اﻟﺠﻔﺎف‬. Ttt:
A. 5% dextrose B. Normal saline C. Hypotonic saline
#Flank mass on children = wilms (nephroblastoma )
(Wilms' tumor is a rare kidney cancer that primarily affects children. Also known as nephroblastoma, it's
the most common cancer of the kidneys in children.)

🌹abdominal mass with lung nodule = wilms🌹(most common metastis wilms to lung )
.
Child presented to the ER with fever and abdominal pain. After a fall 1 day ago, the mother noticed
abdominal distention mainly on the right side. On examination he is pale and hypertensive , your dx ?
A. Liver contusion. B. Pyelonephritis C. Wilm's tumor. D. Neuroblastoma
…..
🌹 mother while showering her sone noticed abdominal mass =
if central = neuroblastoma If flank = Wilms tumor
.....
🌹Pt boy with abdominal mass noticed by mother while she was bathing him, on examination healthy boy
with rt. Flank mass 1st Q what is the investigation: A. US B. CT C. MRI
🌹2nd Q what is the dx: A. wilms tumor B. neuroblastoma
...
🌹Woman was bathing her child and noticed a mass in his flank, which of the following invx is most
appropriate:
A. Abdominal radiography B. Abdominal CT C. Abdominal MRI D. Abdominal radiography and
ultrasonography
....
🌹Boy came to ER has erosions bone in the hand and in the lab has high Ca, phosphate What the
treatment you will give ? Hydration + biophosphanate🌹

1Milestones
🌹 Child jump on 2 feet , what age in years? A. 1 B. 2 C. 3✅

🌹 4 month old baby which of the following he can do= head lag

🌹Baby crawl and site and grasp things but unable to do pincer grasp: 7
„d … ‫ﺤ‬¤= ‫ = ر•ﻂ‬crawl = „d … ‫ﺤ‬¤ ‫ﺎﻧﻪ‬È ‫ﺪﻳﻦ‬¹‫ اﻟ‬,
d ’ª‫ اﻟﺬراع اﻟﺼﻐ‬Šd‫ ﺷ•ﻠﻪ ﻟﻤﺎ ﻧﺨ‬7 ‫ رﻗﻢ‬.
.........

🌹child remove cloth by himself and say dada and tell stories = 4 years 🌹

Child enters the dr’s clinic play w ball through it to doctor , catch a ball ,draw line, puts books together ?
A. 9m. B. 12m. C. 15m. D. 18m✅ ‫ﺣﻞ ﺟﻠﻮري‬

Child enters the dr’s clinic says “Hi”, mother gives him doll. Then he feeds the doll with milk bottle. Mother
moves his head then he says “No”. Mother does something and then he imitates her What’s the age?
A. 12 month. B. 15 month. C. 18 month✅. D. 24 month

🌷Child can say 3 words sentence == 3 years🌹
...... ....
🌹Baby speech understandable to stranger 75% ,says three word sentences = 3 years

🌹if 3 words > 15 months

🌷All reflex disappears after 4-6 months except stePPing at 2 months.🌷
….
🌹 age the sucking reflex disappear? 6 m

#Sit without support = 6 month =‫ﻢ‬¹‫ = ﻣﻦ ﺷ•ﻞ اﻟﺮﻗﻢ ﻃﻔﻞ ﺟﺎﻟﺲ وﻇﻬﺮە ﻣﺴﺘﻘ‬٦

Developmental milestones q: 5 words, hop on one leg ?


A. 48 m✅✅. B. 36 m. C. 24 m. D. 60 m
🌷 hop in one leg is( 48m = 4 years)🌷
‫ﺔ واﻟﻔﺨﺬ‬œ‫ اﻟﺮﻛ‬،‫ اﻟﺴﺎق‬،‫ اﻟﻘﺪم‬، ‫ﺎء‬¹‫ اﺷ‬4 ‫اﻟﻘﺪم اﻟﻮﺣﺪة ﺗﺘﻜﻮن ﻣﻦ‬: ‫ر•ﻂ‬
. . . . . .. . . .. .
Ride tricycle can not draw square= 3 years

🌹 Can say 3 sentence words can't climb up the stairs can run , milestone = 3y 🌹‫ﺗﺎﻛﺪ أﺗﻮﻗﻊ ﺣﻠﮭﺎ ﺧﻄﺎ‬
….
🌹4 month milestone:
A. Sit without support B. Head controlC. Start trying to crawl D. Follow objects
...
🍓Can support his head, smile, follows objects with eyes...= 4 mo
...
4 months old boy what you can expect to his age?
A-try to crawl. B-smile. C-sit without support. D- fix his head
‫ﻢ‬¹‫ﺎﻧﻪ ﻃﻔﻞ ﻇﻬﺮە ﻣﺴﺘﻘ‬È ‫ ﺷﻬﻮر ﻻن ﺳﺘﺔ‬٦ ‫ﺖ ﻇﻬﺮە ﻟﻤﺎ ﻋﻤﺮە‬œE‫ﻨﻤﺎ ﻳ‬â‫ ﺑ‬، ‫ واﻧﻒ وﻓﻢ‬Ý c ª ‫ﺎء ﻋﻴﻨ‬¹‫ﻪ ار• ـﻊ اﺷ‬¹‫ﺖ رأﺳﺔ ﻻن اﻟﺮاس ﻓ‬œE‫ار• ـﻊ ﺷﻬﻮر ﻳ‬
...
🍓Baby can move and fix his head when he in prone position , smile, reache object
A. 2m. В. 4m✅. С. 6m. D. 9 m
...
🍓Smile but not reach objects?. 8weaks✅
...
🍓 Child smile which month= 2 Month🍓 Ý c ª ‫ﻦ =ﺷﻔﺘ‬ñÂ‫ ﺷﻬ‬.
...
🌹 Child can run , stair , can’t use 3 word together age ? 18 month 🌹
....
What is the age expected of ride tricycle? A. 2 B. 3 years✅ C. 4 D. 5
.....
8 Child feed the doll but can't us spoon what his age ? A. 12 months B. 15months C. 18 months ✅ D. 24
.......
Child can jump by two foots, Tell 2 words in sequences
A. 2 years ✅ B. 3 years C. 4years D. 5yeas
..........
🌹Child 4y old == 1/ can speak clearly. 2/ can draw squares and cycles 🌹

Baby says dada, pincer grasp, walk by furniture, pulls himself to standing
position, sit without support A. 12 months B. 10 month C. 9month ✅. D. 8month
......
Child enters the dr’s clinic says “Hi”, mother gives him doll. Then he feeds the doll with milk bottle.
Mother moves his head then he says “No”. Mother does something and then he imitates her What’s the
age? 18 mo
...... .....
🌹can sit in his own, stand alone, walk by the furniture, say word of repetitive consonant sounds like
"dada" can hold objects between his thumb and index finger = 10 month .
‫ ر•ﻂ =ﻟﻤﺎ ﻧﻀﻢ اﺻﺒﻊ‬thump and index ‫ﺎﻧﻪ ﺷ•ﻞ‬È ‫ﻈﻬﺮ‬¤ O ، ‫ وﻻﻧﻪ‬stand alone 10= ‫ ﺻﻔﺮ ورﻗﻢ واﺣﺪ‬، ‫ﺎﻧﻪ رﻗﻢ واﺣﺪ‬È .
......
Baby can walk and say 2 to 3 words rather than mama dada he crawls the stairs age?A-1 ‫ﺎ اول‬-‫ﺎ‬-= ‫ﺳﻨﺔ =ﻣﺎﻣﺎ‬
‫ﺎة ﻧﺘﻌﻠﻤﻬﺎ‬¹‫ﺎﻟﺤ‬- ‫”ء‬•
d B-2 C-3 D-4

🌹baby which is 4 years old and can hop on one leg but can't stay still for 10 seconds, can draw a square,
has a friend, can tell a story the mom is worried about the developmental milestones?
A. The baby is normal no need for referral
B. He has a gross motor delay and fine motor delay
C. He has language and speech delay

🍓3 years old with normal milestones to his age except that he doesn't know how to
speak in future tense and can't say 3-word sentence, what would you tell the mom
A. reassure. B. he has a delay in speech only but other milestones are fine
Answer is: refer to ENT for audiogram

6-Question asking about best time to do red reflex examinations


A-All age B-Before school C-6 months and above D_ at birth and age of 6 weeks.✅✅
…..
🌹Same as this pic ask which reflexA. rooting B. suckling

....
🍇Rooting reflex when disappearing? 4m

🌹 Pic of baby and physician hand. He was taping the left side of mouth When the primitive reflexes
disappear? A. 2 m B. 4m C. 6 m D. 12

🌹stepping reflex ===

🌹 child can run towards you and follow 2 step commands= How old? 2years
...
🌹A child with pincer grip but cant put pallets in a bottle. How old? 8 month
...
🌹 child whose 3 years old can talk to other children and strangers understand 75% of his speech. What
kind of delay? Or non? no delay
....
🌹A 4 year old child with a morning limb and knee arthritis and (-) ANA. When should see the
ophthalmologist to check for uveitis? 6months
.....
🍓what’s the time to say fever of unknown origin in pedia, per day : A. 7. B. 14✅ C. 21 D. 28
١٤ =٢ ‫ = ﺳﺑﻊ ﺣروف ﻧﺿرﺑﮭﺎ ب‬unknown = ‫رﺑط‬
….

Vaccine
🌹newborn needs vaccine and his sister died of immunodeficiency disease. What should you do?
evaluation by immunology team
..
🌹newborn has a brother who died from immunodeficiency, regarding his vaccination? Defer all until
immunology result
..
🌹 Child allergic to eggs. What vaccine should be avoided? Yellow fever

🌹Child has a brother who died from severe infection came for vaccines which will u defer till knowing
immunity status? A. varicellan B. influenza C. PCV
.....
🍇12 y child with mild fever and vehicular rash in chest , trunk upper lower which test has higher dx
value : varicella zoster igm
..
🌹Which vaccine is contraindicated in child with egg allergy?
A. MMR B. Pneumococcal C. Polio D. Influenza
..
🌹Baby present with with tea color urine with sore throat +140/80 Urine analysis done ( tea color, WBC,
erythrocyte ) next = creatinine.

🌹 Steroid dependent nephrotic syndrome patient needs vaccine?
A. wait 6 month after stopping the steroid B_wait 1 month after stopping the steroids
...
🌹Female has crohn's on biological and azathioprine, what about her baby vaccinations?
A. Delay All 6 months✅. B. Delay Live 12 months
....
🌹What is the absolute contraindication for DtaP vaccine:
A. seizure (just like that)
B. hx of encephalopathy within 7 days following previous dose of DtaP
...
🌹SCD patient took blood transfusion when should you give the vaccines?
A. give All vaccines B. delay 3 months C. delay 6 months
....

🌹 DTP immunization expires in?


A. 10 years B. 7years
...
🌹 Baby missed vaccines 18/24.. 2 days swollen eye pain.. Low eye acuity.. What's next most appropriate..
A. Give vaccine.. B. Mri brain to see hemorrhage . C. CT for extend of disease stuff like that..
Answer is: C, ORBITAL CELLULITIS.
….
🍇Hepatitis a vaccine for slckles who transfer blood = Give as scheduled ✅

🌹10 years old with abdominal pain and lab show high liver enzyme and indirect bilirubin.. diagnosis ?
A. Viral hepatitis B. Gilbert

🌹child with "reccurnt infection" with brother did due to septic shock what will give = Dont give live
vaccine 🌹
. . ..
Child with nephrotic syndrome on steroid for 3-6wks or 6 months has vaccine ?
A. Give thev accine B. Stop the steroids C. 3month and give appointment✅ D. Noneedtotreat
….
🌹Baby 3 hours of life. His brother died of immune condition =
A.give BCG B. Do not give BCG and only give hepatitis vaccine ✅ C.Give immunoglobulin
D.Book an appointment and review condition thoroughly
ì c
d ‫ﻜﻮن ﺧﻼص ﻣﺎﻟﻬﻮدا‬¹‫ ﻟﻮ ﻋﺪى وﻗﺘﻬﺎ ﺑ‬ôd … ‫ﺐ‬¹‫ ﻫ‬ݪ‫ ﻻن ﻓﺎ‘ﺴ‬ôd … ‫ﻦ اﻋﺘﻘﺪ ﺣﻠﻪ‬õ‫ﻘﺎ دي وﻟ‬-‫ﻣﺘﻜﺮر ﻫﺬا اﻟﺴﺆوال وﺗﻚ ﺣﻠﻪ ﺳﺎ‬
........
🌹Vaccination at age of school (6years) = Dtap , MMR, OPV, Varicella
.......
🌹Vaccines of 6 yrs old child (pre-school) in saudi arabia = OPV,MMR, dtap, varicella

🌹what to do for child came for vaccines but he on antibiotics course? DONT delay till finishing his
antibiotics.
most kids, taking antibiotics for a mild illness (like an ear infection) shouldn't keep them from getting
their vaccinations on schedule. Antibiotics do not interfere with the ingredients in vaccines or cause a bad
reaction in a child who has just been vaccinated.
.......
🌹Neonate was born the mother says she had a baby died due to probably immune deficiency and ask
about vaccines for the new baby?
A.Defer all vaccines until immuno results show B.Order the vaccines and follow up with immunology
C. Take hep B only ✅✅ D.Take all vaccines as usual

🌹2 year child didn't complete his vaccination present with fever, unable to swallow , dysphasia , enlarged
tonsils , your dx ?. Diphtheria ✅✅

🌹Mother brought her Newborn to vaccination and she said his sister died at age 3yrs immune-
compromised disease What accurate ? A. give BCGA B. Don’t give live vaccination
C. not give and refer to immunologicalteam✅
….
🍓Pregnant negative varicella antibodies= A. IViG B. give vaccine, C. avoid exposure ✅
….
🌹Born of baby with brother die due to Immunocompromised
A.Order vaccine and follow with immunity team B.Defer vaccine until immune team evaluate ✅
.....
🍓Vaccine avoid in low immune? Live

a similar case happened at school weeks ago, the parents were concerned about another
immunodeficient child at home what to do with him ?
A. Injection af specific immunoglobulin to the immunodeficient child✅
B. Start antiviral therapy to immunodeficient child
C. Vaccination. (Contraindication live vaccin) .
D.Reassure cause the condition is self limiting
….
🍓Pregnant lady on chemo what vaccine not to give to child after delivery
A. do not give the child vaccine until he is 1 year old
B. Start vaccination at the age of 6 months✅‫„ ﺗﺮﺟﻊ ﻟﻪ اﻟﻤﻨﺎﻋﺔ )ﺣﻞ ﺟﻠﻮري‬i‫ ﺷﻬﻮر ﺣ‬٦ ‫ﺘﻈﺮ‬ö‫ﻠﺔ ﻋﻨﺪە ﻓﻨ‬¹‫(ﻻن اﻟﻤﻨﺎﻋﺔ ﻗﻠ‬
C. Vaccinalions given at 2 month
..........
🍓Vaccines of 4 months baby:
hepb, dtp,Hib,MMR
hepb, dtp, Hib, PCV ✅
hepb, dtp, hepA, PCV
............
🍓(immunodeficiency ) What vaccine contraindication = vareclla🍓

🍓2 year sickle cell anemia took one dose of hepatitis A vaccine and came for second dose but she had
history of blood transfusion before 3 weeks due to vasoocclusive crisis what you will do :
A. Give the second dose✅ B. Give it after 3M C. Give it after 6M D. Check the hepA antibodies

🌹6 months with diarrhea ONLY, no vomiting or fever = vaccinations?Give all.
‫ﺎﺧﺬ اﻟﻠﻘﺎح‬v ‫ﻤﻨﻊ اﻟﻄﻔﻞ‬v‫ﻋﺎدي اﻻﺳﻬﺎل ﻟﺤﺎﻟﻪ ﻣﺎ‬.
.....
Child presented with vesicular lesions all over body Parents are concerned about another child who is
immunocompromised and contacted wih his sick brother give = special IVIG for immunocompromised
patients.
‫ =ﻣﻦ اﺳﻤﻬﺎ‬Ýc ª •&‫ﻪ اﻣﻴﻮن ﺟﻠ‬¹‫ ﻃﻔﻞ ﻋﻨﺪە ﺿﻌﻒ ﻣﻨﺎﻋﺔ ﻧﻌﻄ‬immungloplin = immuncompromise.
....
🌹1 YO vaccination : OPV, MMR, PCV, varicella
)🤐‫ﺑس( ﻋﯾﺑﮭم ﺻوﺗﮫ اﻟﻌﺎﻟﻲ = ﺑس )اﺳﻛت‬Pcv ( ،) ‫ﻛﻠﻣﺔ )أوﺑﺎ ﻟﻠطﻔل‬Opv ، ‫ اﻟطﻔل ﻋﻣره ﺳﻧﺔ ﯾﻧطق ﻣﺎﻣﺎ‬، ‫ ﻣن ﻣﺎﻣﺎ‬M ‫رﺑط = ﺣرف ال‬
….
vaccine to give in 2mo old baby? IPV hbv HIB PCV rota DtaP ‫ﻠﻬﻢ‬È

🌹 4 months old vaccine =IPV, HBV, Hib, DTaP, PCV
….
🌹Mother crohns and take biologic drugs and azathioprine what regard baby vaccinations? delayed all
6 months
‫ ﺗﺮﺟﻊ ﻣﻨﺎﻋﺘﻪ‬Èt‫ ﺷﻬﻮر ﺣ‬٦ ‫ اﻻ |ﻌﺪ‬ï „ z ‫ﻪ ﻓﺎ¸ﺴﻴ‬n‫™ﻂ اﻟﻤﻨﺎﻋﺔ ﻓﻤﺎ ﻳﻨﻔﻊ اﻋﻄ‬ð‫ﺬا ﻳ‬P= ‫ﺮاﻳﻦ‬iµ‫ﺜ‬Ì‫دواء از‬
…..
🌹Immunodeficient baby, What vaccine should you avoid: Varicella
….
🌹 Boy with throat thrush, fever and weak Which vaccine will you not give = Varicella
A. PCV. B. Varicella✅. C. IPV

🍒 What vaccine contraindicated in immunocompromised patients ? live vaccines🍒

child in vaccination clinic. His brother has immunodeficiency and died. Which vaccine to defer until this
child seen by allergy/immunology unit? Varicella

🌹Child missed 12 m and 18m vaccination came with bulging eyes and diplopia what to do = CT to see
infection infiltrate
..
🌹Mother on biological drug: Delay live vaccine 6m

🌹mother is not vaccinating her child what to do? -Explain and clear the myth

child , his brother died of immunodeficiency, what vaccinations to hold until confirm immunity of the
child: Varicella )*+(live attenuated)✅✅

🌹Child K/C BA, didn't take his vaccines, parents refused vaccines cause it is harming their child = Explain

🌹child with no vaccinnation , now he has ( pharyngitis + cervical lymphoadenopathy = Diphyheria 🌹
‫ﻂ‬ñ‫ = ر‬pharyngitis = Diphyheria
. . .. . . .
🍇2yoars old with fever cough SOB her mother confirms that she had all vaccines
without any mIssIng what is the causative organism
A. strep pneumoniae✅ B. H.Influenza C. Aspergillus pneumonia D. Klebsiella
.....
🌹A newborn baby just delivered, mother states that she had a baby that died of immunodeficiency, whats
the protocol regarding vaccination of this baby in saudi arabia
A. don't give bcg vaccine B. give and observe in the clinic after 1 week
🌹 New born to mother on adalimumab, regarding vaccine:
A. give all B. dely for 2 months C. delay live attenuated 6 month D. delay live attenuated 12 month

🌹 child came for vaccination had tonsillitis and lethargy since 4 days was started on antibiotics and is
now better what will u do== a. defer 2 weeks b. vaccinate✅ c. all except dtap
.. .
🌹-Child has brother who died from severe infection came for vaccines which will
u defer till knowing immunity status a. varicella b. influenza c. PCV

🌹 baby was born in 35 wk, parents asking about vaccines?
A. Delay them to 2 months B. Don’t give live vaccines C. Give all ✅✅

🌹 Case of pediatric patient known to have Seizure, came with syndromic feature hypotonia large head
asking for vaccinations: A. Give all B. Stop all live vaccine C. Stop Dtap✅
Contraindications to DTaP unstable neurologic disorder such as uncontrolled seizures .
….
🌹Vaccine contraindicated with pregnancy, severe illness? Varicella
….
🌹 2month infant GA at birth was 27 weeks 1.7 kg and spend 2 weeks in nursery. presented now to the
clinic for first time doing good and gaining weight appropriately best action:
A. delay all vaccines 2 months B. give all vaccines same dose C. give all vaccines half dose
D. dtap instead of Dtap (something like this)
....
🌹 Child scheduled for vaccines, mother mention old baby was having septic shock and died after vaccines,
which vaccines will delay: A. varicella B. injectable poly

🌹most common minor side effects of routine DTP vaccination?
A. injectable site erythema. B. generalized pain C. fever 38.3. D. vomiting

🌹Father refused to vaccinate his son. What to do?try to counsel him by benifits of vaccine

unvaccinated kid and his dad said he doesn’t believe in vaccinations, what to do:
A.Inform child protection B. Talk to the father about the importance of vaccinations ✅

🌹baby with diarrhia give vaccin or not give ? Its okay u can give all vaccine , Diarrhia not contraind 🌹

🍇Regarding dtap vaccines , Mother present with her 6 month, she reported that after the 4months
vaccines her baby developed rash ,fever ,What is the precaution ?
Give prophylaxis antipyretic and antihistamines ✅
…….

Respiratory
weeks old baby with strong cough and 2 episodes of him losing consciousness. on exam there is
intercostal retractions. 02 sat was 90 . What do you do?
A. Inhaled steroids B. Ventilator support C. Antibiotics D. B2 agonist ✅
......

🍓Pt with tachypnea orthopnea.. (Symptoms and signs) of Rt sided heart failure on lab investigations Liver
enzymes are very high abnormal ? A. Heart failure✅✅ B. Renal failure C. Liver failure
….
🍓Newborn after CS have respiratory symptoms , CXR shows fluid in the horizontal fissure , what’s the dx
:A. acute distress syndrome B. meconium aspiration C. Transient tachypnea of the newborn (TTN)✅
...........
🌹 Horizonal line in x ray of infant = Transient tachypnea of newborn

🌹Baby born 27 weeks after 30 mins started having tachypnea and gruntingsz‫ﺷﺨ‬
most common cause?
a. RDS✅ b. TTN c. Meconium aspiration
Respiratory distress syndrome (RDS) is when the neonate has difficulty breathing faced by preterm
infants and is directly related to structurally immature and surfactant deficient lungs
.....
🌹 30-min aged baby preterm of 27 weeks, developed tachypnea and grunting , Dx:
A. Respiratory distress syndrome B. TTNn C. pulmonary embolism D. meconium aspiration
...
Newborn withnasal flaringandsubcostal retraction? -respiratory distress syndrome
c ª ‫ﻠﻤﺘ‬Ó‫ اول ﻣﺎ *ﺸﻮف ﻫﺬە اﻟ‬، ‫’س‬i‫ﺴ‬±‫ ﻫﺬە واﺿﺤﺔ اﻋﺮاض د‬.
‫ ﻃﻮل ﻧﺨﺘﺎر اﻟﻤﺘﻼزﻣﺔ ﻫﺬە‬Š‫ ﻋ‬Ý
... ..... .....
🍒🍒Patient known case of SMA (spinal myotonic atrophy), develops respiratory muscle fatigue and needs
intubation , mother refuses as one of her children died of the same condition what will you do?
A-consult ethical committee B-intubate her ✅✅ ‫ﺤﺘﺎج اﺻﻼ ﻣﻮاﻓﻘﺔ‬¤‫🍒🍒 ﻻن ﺣﺎﻟﺘﻬﺎ ﻃﺎرﺋﺔ ﻣﺎ‬
. . . .. . .

🌹 Another case with term baby, that show *Horizontal fissure* on X- ray: TTN
.......
🌹Baby with x ray show ( TTN ) + symptoms of pneumonia + Lap high Neutrophils Ttt= Oral amoxicillin 7
day✅
....
🌹Term baby with Respiratory distress after CS Delivery CXR : fluid in horizontal fissure = Transient
tachypnea of the newborn TTN.
….
🍒 7 year old Child ingested 20 tablet baby aspirin ,what you expect regarding acid base balance ?
Respiratory alkalosis > metabolic acidosis.🍒 A. Respiratory alkalosis > respiratory acidosis
B. Metabolic alkalosis > respiratory acidosis. C. Respiratory alkalosis > metabolic acidosis.✅✅✅
D. Metabolic acidosis > respiratory alkalosis.
.....
🌹child was in picnic‫ﺔ‬P‫ ﻧﺰ‬sudden he devlop respiratory symptom = causes = lung collap( due to forgin
body) 🌹
….
🍓A 2-hr old baby = What intervention would minimize disability in the first 6 hours?
A. Respiratory support✅ B. Mild hypothermia

🌹A newly born presents with meconium aspiration. He was stabilized by intubation and given IV
inotropic fluid. After that he developed respiratory distress. His preductal O2 sat was 92% and post ductal
O2 sat was 83%. What is the next step in managing the patient? nitric oxide
.....
🌹3m baby with respiratory distress when to admit for non invasive ventolatlon : A-ER observation
B-pedia ward C-pICU

🌹2 months boy with bronchiolitis On ( non invasive mechanical ventilation) need admission... A.
Pediatric ward B. neonatal care unit C. Pediatric ICU D. ER
....
🌹 2 months baby with Bronchiolitis need non invasive respiratory support wher to keep the baby ?
PICU
....
🍇 18 m k/c of cerebral palsy presents with respiratory compromise Then put under MV Given ABG ,Pco2
normal,Po2 low ,PH 7.3 Ask what type of respiratory failure? A. hypoxic B. Hypercapnic✅ C. hypocapnic
Answer is:B since he is under MV
Neurogenic brainstem, respiratory center Neurogenic will cause hypercanic type ll
.....
🌹 Child with Cerebral palsy on mechanical ventilation , readings, what type of respiratory failure:
A. hypoxic RF B. hypercapnic RF D.Neurogenic Hypercapnic
„ t
‫ ﻻن ﻋﻨﺪە‬cerebral pulsy ƒ… ‫ ﺧﻠﻞ‬brain ‫ﺐ ﻧﻴﻮروا‬ö‫ﺴ‬õ ‫ اﻟﺮﺋﺔ ﺗﺘﺎﺛﺮ‬ó
… ‫ﻓﻤﻨﻄ‬
....
🌹CP baby on mechanical ventilation ABG ( all within normal even normal Co2 + normal o2 ) Type of
respiratory failure ? Neurogenic ( not hypoxia or hypercapnia)‫ﯾﺣﺗﺎج ﻧﺗﺎﻛد ﻣن اﻟﺣل‬
.....
🌹18 months old with cerebral palsy had episodes of vomiting and unable to breathe. Then he was moved
to ICU. He was on 100% FIO2. ABG showed NORMAL PH AND CO2 but LOW O2. what is the diagnosis?
hypoxic Rispiratory failler .
....
🌹 young girl presented with palpitation and hypercapnia. Father mentioned issues with schools and
exams. What is the diagnosis? Hyperventilation syndrome
....
🌹Child with pneumonia hemolysis + positive agglutination test, what is the organisation
A- sterpt P B-moraxella C-mycoplasma D- H influenza
...
🌹 child vaccinated up to date has high grade fever and cough, x ray shows bilateral infiltration , and
auscultation bilateral crepitation more in rt lower lobe, what is diagnosis
A - viral infection B- atypical pneumonia C- sterpt pneumonia
..
Boy came with respiratory infection symptoms, by auscultation: crackle, by precaution: stony dullness
direct under the crackles sound, Diagnosis? A. Pneumothorax B.pleural effusion C. Collapsed lung
...
🌹neonate presenting after 2 days of delivery with history of seizure and hypertonic extremities, most
likely
....
🌹Decrease mortality in first few hours after delivery of hypoxic ischemic encephalopathy baby=Mild
hypothermia
...
🌹Most beneficial action in the first 6 hours for the baby in the previous Q is?
Total body hypothermia or total body cooling
....
🌹Pre term baby diagnosed with hypoxic ischemic encephalopathy (HIE) what's the risk factor : Low birth
weight less than 1500 gm
...
🌹child came with seizure and they give CSF value " all normal " Dx? Hypoxic ischemic encephalopathy
...
🌹Mother had obstructed and difficult labor, she gave birth to a child who wasn't crying and cynosed. His
ABG: pH 6.9, HC03 7, 02 8. What's the diagnosis? A. Hypoxic ischemic encephalopathy ✅
....
🌹 6y old child k/c of asthma came with asthma exacerbation , lab showed RR = 7/minutes , Hypercapnia
best initial ? A. IV MG B. Intubation C. Albuterol D. Theophylline
.....
🍓pt on ventolin not improved, what to add ? A. ICS✅. B. Increase dose of ventolin
….
🍓Infant diagnosis with VSD present with symptoms and sign of heart failure ( hepatomegaly, SOB,
Cardiomegaly ) what's the next step in management?
a) Admitted to ICU for cardiac b) catheterization c) Give diuretics ✅
‫ﻪ ﻣﺪرات ﻟﻠﺒﻮل‬¹‫ﺐ ﻓﺸﻞ اﻟﻘﻠﺐ ﻧﻌﻄ‬é‫ﺴ‬ø ‫ اﻟﺮﺋﺔ‬Š‫ﻢ اﻟﺴﻮاﺋﻞ ﻋ‬ÿ‫ﺐ ﺗﺮا‬é‫ﺴ‬ø ‫ﺪ وﻫﺬا‬œõ‫ اﻟ‬bd c ‫ﻘﺔ ﻧﻔﺲ و ﺗﻀﺨﻢ‬¹‫ﻘﻮﻟﻚ ﻋﻨﺪە ﺿ‬¤ ، ‫ﺐ‬ñ’i‫ﺎﻟ‬- ‫ ﺧﻄﻮات‬ý •
d ‫ﻧﻤ‬
….
🍓 Mother brought her child for a check up, he is asthmatic on inhaled ventolin, she told the doctor he is
having minimum 2 cough attacks per week, and she came to the ER 2 times last month. Management?
A. continue same management B. Increased dose of Ventolin C. Inhaled corticosteroids
….
🌹Patient with recurrent dyspnea attack due to inhaling dust or perfumes, on examination has inspiratory
wheeze. Asks about initial management: Ventolin Neb
..
Asthmatic patient on inhaled corticosteroid and short acting beta 2 agonist which he use 3 times daily.
What is next step in the management?
A. oral steroid. B. Long acting beta2 agonist✅. C. Interleukininhibitors. D. Mastcellstabilizer

🍓Child with asthma and his mother also she had poor technic for inhaler use the child have extirpation
despite the treatment what is the cause= poor technic
….
🌹child with bronchial asthma her mother concerned about the prognosis of her child disease? it’s mostly
will get out in children ‫ﺮوح ﻣﻨﻬﻢ‬Ì‫ﺠﻴﻬﻢ ازﻣﺔ و‬v sz‫ اﻃﻔﺎل ﻛﺜ‬È… „ ‫ﻌ‬v .

🌹A child who had flu-like symptoms, then has a typical picture of asthma exacerbation What is the first
line of management? Beta agonist ( not Anti-viral)

🌹Pregnant lady, just delivered and she's known to have bronchial asthma. Which of the following
uterotonic medications you would avoid giving? carboprost F2 alpha

🌹Child has Asthma , and father is smoking , what to do? Counseling parents of smoking cessation

Child with asthma & mitral regurge , presented with symptoms of pneumonia ( fever ) :give abx &
reevaluate later✅

🌹child has asthma symptoms but when do splrometry was normal what is the best : Methacholine
challenge test

🌹child had mild dyspnea, when exposed to dust diagnostic test of asthma:
A. Spirometer B. Methacholine challenge test
...
🌹 12 yo girl complaining of nausea and vomiting if the flight was more than 1 hour what is the best
antiemetic for her ?
A. ginger B. 2 drugs end with steron C. diphenhydramine D. Dimenhydrinate
Note: The Dx is Motion Sickness > Tx with Anticholinergic agent like Scopolamine
Or with Antihistamines agent like Dimenhydrinate
,,,,,
🍇5 year boy with bronchial asthma on 2 medication, has syrr mother concern about him not gaining
weight. Was : Less than 5th percentile weight and At 25th percentile height choices were:?
A. reassure mother and refer to a dietitian
B. investigate more for possible asthma mimickers ✅
......
🍒 4 y child diagnosed with BA , has wheezing , failure to thrive = . refer for possible asthma
mimickers✅✅🍒
‫ﺐ‬é‫ﺴ‬î ‫ اوﻻ اﻻزﻣﺔ ﻻ‬failler to thrive ، ‫ﺐ‬é‫ﺴ‬8‫ﻌﻤﻞ ﻧﻔﺲ اﻋﺮاض اﻻزﻣﺔ و‬¤ Ûd‫ﺶ اﻟﻤﺮض ا‬±‫ﺐ ا‬¹‫ ﻃ‬failler thrive ‫ ؟ ﻫﻮ‬cystic
fibrosis , ‫ﻠﻤﺔ‬È „c‫ ف ﻣﻌ‬:
Asthma mimickers
‫ﻪ اﻋﺮاض اﻻزﻣﺔ‬œ‫ﺸ‬î ‫ﻘﺔ ﻣﺎﻋﻨﺪە ازﻣﺔ ﻋﻨﺪە ﻣﺮض اﻋﺮاﺿﻪ‬¹‫ اﺳﺎس اﻧﻪ ازﻣﺔ واﻟﺤﻘ‬Š‫ﻜﻮن اﻟﻄﻔﻞ ﺷﺨﺺ ﻋ‬ñ‫ و‬، ‫ﻪ ا اﻻزﻣﺔ‬œ‫ﺸ‬± ‫„ ﻣﺮض‬d c ‫ﻌ‬¤
‫ وﻣﻨﻬﺎ‬cystic fibrosis
... .... ..
M child C/O dry cough, has FHx of BA, he was given bronchodilator but hasn't improved, what is the most
indicating factor of cystic fibrosis ?A- Not responding to bronchodilator B_ Poor weight gain✅✅

🌹S & S of cystic fibrosis and asked about which sign u suspect to find ? Nasal polyp

Child presenting for check up clinic when to be concerned about growth ?


A-Body wt persisting at 10th percentile ✅ ‫ة‬²•‫ رﻗﻢ ﻋ‬B-At 15th percentile C-Progressing from 5th to

🌹Criteria to help diagnose HTN in a 12 YO girl? A. BP >120/80 B. BP>140/90
C. >90th percentile for age and sex
D. >95th percentile for age and sex

🌹During the evaluation of adolescent she was on the 95th percentile for age and sex, what of the
following action should be taken
A. Initiate multidisciplinary team B. Confirm size is more than average for age and sex C. Give a list of low
calorie food D. Avoid talking about weight

🍓Growth chart (similar to the pic) showing normal at birth, increasing but below the 3rd centile,
Diagnosis? failure to thrive 🍓

....
1Autoimmune
🌹Pediatric pt with ptosis and dlplopia and knee hyperreflexia with no fasciculation symptoms worse
through the day and improve next morning , what is the pathophysiology :
A- Anti-bodies against thyroid preoxidase
B- anti-bodies against Acetyle cohen receptors ‫ﯾﺣﺗﺎج ﺗﺎﻛد ﺣل‬
C- genetic mutation in specific gene ' c+s)
....
🌹Child with +ve cover test -) stabismus
This is an Important risk for (or major concern or something) as it can cause :
A-amblyopia✅ B-strabismus C-leukocria
Note:+ve cover test Diagnosis ? Strabismus

🌹Baby with AV block... Which of the following Maternal conditions is associated with the bab
presentation? SLE

🌹Active arthritis for 4 months with morning stiffness 2 h with x Ray findings
= Hydroxychloroquine and methotrexate weekly ✅

🌹Child with URTI then complaining of bleeding from nose, gum and bruising the treatment is ?
A. Prednisolone ( for mild bleeding + less than 30, 0000 + like this case all symptom is "mild bleeding " .
✅✅✅
B. IVIG (Bez there is no sever bleeding , sever bleeding mean there is intracrania Hemorhgae )
C. Antibiotics ‫ﻣﺎﻟﻪ دﺧﻞ‬
D. Supportive ‫’ف ﻣﺎﻳﻨﻔﻊ‬c c ‫ ( اﻻن ﺟﺎﻟﺴﺔ ﺗ‬we choice when there is no bleeding )
….
4 y old girl with history of limping and movement restriction for 2 months, ANA negative, she is otherwise
normal What is the screening for Uveitis ?
1) every 3 month. 2)every 6 month✅✅ 3) every 9 month. 4) no screening b/c she is ANA -ve.
Uveitis Screening in Patients with Juvenile Idiopathic Arthritis
Uveitis is a serious complication of juvenile idiopathic arthritis (JIA). Approximately 6% of all cases of uveitis
occur in children, and up to 80% of all cases of anterior uveitis in childhood are associated with JIA
، ‫ ﻻن اﻟﻣوﺿوع ھذا ﻛﺑﯾر‬، ‫طﯾب ﺑﺷرح ﺑﺷﻛل ﻣﺧﺗﺻر ﻣرة‬
‫ ھذه‬، (( Uveitis )) ‫ ھو اﻧﮫ ﯾﺣﺻل ﻟﮭم‬Juvenile Idiopathic Arthritis ‫اوﻻ ﻣن ﻣﺿﺎﻋﻔﺎت اﻟﻲ ﻛﺛﯾﯾر ﺑﺗﺣﺻل ﻟﻼطﻔﺎل اﻟﻲ ﻣﺻﺎﺑﯾن ب‬
‫ وطﺑﻌﺎ ھذا ﺧطر ﻻن ﺑﯾﺟﻲ ﻟطﻔل وﻣﻣﻛن ﯾﺳﺑب‬uvitis ‫ ﯾﺻﺎﺑوا ب‬JIR ‫ ﺑﺎﻟﻣﯾﺔ اﻟﻲ ﺟﯾﮭم اﻟﺗﮭﺎب اﻟﻣﻔﺎﺻل‬٦ ‫ﻣن اﺣد اﻟﻣﺿﺎﻋﺎﻓﺎت اﻟﻲ ﺗﻘرﯾﺑﺎ‬
‫ ﻛل ﻓﺗرة‬، ‫ ﻓﺣص دوري‬Screeing ‫ ﻻزم ﻧﻌﻣل ﻟﮭم‬Juvenile Idiopathic Arthriti ‫ ف ﻗﺎﻟوا اي اﺣد ﯾﺻﺎب ب‬، ‫اﻟﻌﻣﻰ ﻟو ﻣﺎ ﺗﻌﺎﻟﺞ‬
‫ طﯾب ﻋﻣﻠوا ﺗﻘﺳﯾﻣﺔ وﺟدول ﻛل ﻛم ﺷﮭر اﻋﻣل ﻓﺣص وﺳﻛرﯾن ف اﻟﺟدول ﻣﻘﺳم ﻋﻠﻰ ﺣﺳب اﻟﻌﻣر‬، ‫ﻣﺣددة ﺣﺗﻰ ﻧﻛﺗﺷﻔﮭﺎ ﺑدري وﻧﻌﺎﻟﺟﮭﺎ‬
٦ ‫ ﻗﺎﻟوا ﻟو اﻟطﻔل ﻋﻣره اﻗل ﻣن‬، ‫ طﯾب ﺧﻠﯾﻧﺎ ﻋﻠﻰ ﻗد اﻟﺳؤوال وراح اﺿﻊ ﻟﻛم اﻟﺟدول‬، negative ‫ وﻻ‬postive ‫ اذا‬ANA ‫وﻋﻠﻰ ﺣﺳب‬
month6 ‫ ﻓﺎﻟﺟواب ﺑﺎﻟﺳؤوال‬. ‫ ﯾﻛون ﻛل ﺳﺗﺔ ﺷﮭور‬screeing ‫ ﻓﺎل‬Negative ‫ ﻋﻧده‬ANA ‫ﺳﻧوات و‬
…..
3 years old boy his mom complained that she noticed blood spotting in the dipper,
there was another episode 3 months ago, no abdominal pain or other manifestation, on examination
patient looks pale and mucosal dryness, what’s the most likely diagnosis:
A. Juvenile polyps✅
B. Mickle diverticulum = Mainly present with abdominal pain
C. Intusscesption = (red jelly-like stools), sometimes mixed with mucus.)
Juvenile polyposis syndrome is a disorder characterized by multiple noncancerous (benign) growths
, typically develop polyps before age 20; These growths occur in the gastrointestinal tract, typically in the
large intestine (colon). The number of polyps varies from only a few to hundreds, even among affected
members of the same family. . . Polyps may cause gastrointestinal bleeding, a shortage of red blood cells
(anemia).

🍒 juvenile idiopathic arthritis case : both knees arthritis , nothing else, type?Oligoarthritis🍒

🌹Pediatric Knee+elbow pain increase in morning ? Juvenile rheumatoid idiopathic🌹
….
🌹12 y boy with right knee and wrist swelling and subcutaneous nodule = Juvenile rheumatoid arthritis

🌹Pediatric patient with rt elbow pain and lt knee pain, worse in the morning and
improve with day A. juvenile idiopathic arthritis✅. B. reactive arthritis
juvenil .‫ ﺑدون ﺗﻔﻛﯾر اﺧﺗﺎروا‬joint pain ‫ﻻن ﺑﯾدﯾﺎﺗرك وﻋﻧده ﻛذا ﻣﻛﺎن ﻓﯾﮫ‬

🌹Neonatal heart block, what is the most likely condition the mother had? SLE ( not Rubella ❌)
….
🌹 Baby with painless per rectal bleeding in the diaper Hx of same complaint 3 month?Dx?
A. Juvenile polyp B. Hirschberg dis. C. Mechels diverticulum ✅
‫ ﻣﻄﻌﻢ ﻣﺎك‬mac ‫ﺧﻠﻔﻴﺘﻪ ﻟﻮﻧﻪ اﺣﻤﺮ‬
🌹if there is familly history = Juvenile polyp

🌹pedia 3y pt have painless bleeding per rectum ?
A. Juvenile polyp B. Meckel's diverticulum (age)=typically appear before the age of two years.
…..
🌹Eye Cover test ? Strabismus
….
Pediatric Patient Dx with cystic fibrosis, sibling no hx What appropriate management?
1) test siblings‫ﺎء‬iø‫ اﻗ‬for chloride sweat test.✅ 2 test parents for chloride sweat test.
3) genetic test for CFTR gene for the sibling 4) genetic test for CFTR gene for parents

🌹 Baby with +ve sweat chloride test?
A. Sibling sweet chloride. B. Parent sweet chloride C. for brothers and parents
.....
🌹 Pediatric patient presented with sudden apnea, has a history of upper respiratory infections, on
examination the patient takes coughs several hacking cough then takes a breath with inspiratory wheeze,
between coughing she looks fatigued and exhausted. No fever. Best investigation
A. Sweet chloride text B. CXR C. ABG

..
Exactly this picture:
A. measles B. chicken pox C. henoch-schonlein purpura

….
🍇Child presented to ER with parents c/o hematuria and urine Incontinence with previous hx 2 weeks ago
with constipation and fever . Lab show Wbc; 8000 with low Hb, UA show : + protein and +++Rbcs, What's
most likely diagnosis??
A. henoch-schOnlein purpura ✅ B. Autoimmune hemolytic anaemia C. Post streptococcal
glomerulonephritis
.......
🌹A child presented with joint pain and hematuria. Mother reported history of URTI 4 weeks ago. On
exam there was petechial rash involving buttocks and thigh, otherwise normal. Platelets normal. What is
the appropriate Diagnosia? Henoch-Schonlein purpura
‫ ﻣﻌﻪ‬Cd … ¹‫ﺔ ﺗ‬¹‫ﻫﺬا اﻟﻤﺮض ﻳﺘﻜﻮن ﻣﻦ ﺛﻼث اﻋﺮاض اﺳﺎﺳ‬
Joint pain + abdominal pain + petecha
Ask about Treatment = mainly Suppurative. But with syptom active = Steroid .
…….
🌹Young pt have hx of bloody diarrhea, after 1 week he developed petechiae rash, he have hematuria and
low platelet - Pt & ptt normal=====A. HUS ✅ B. TTP C. ITP D. DIC
(HUS) is a condition that affects the blood and blood vessels. It results in the destruction of blood platelets
(cells involved in clotting), a low red blood cell count (anemia) and kidney failure due
.....
Child has bloody diarrhea after a week develop petechial rash + hematuria and low plt Pt and ptt normal
? A- hemolytic uremic syndrome. B-TTP C-ITP D-DIC
✴HUS= It results in the destruction of blood platelets (cells involved in clotting), a low red blood cell count
(anemia) and kidney failure due to damage to the very small blood vessels of the kidneys.
….
🍓years old boy with petechiae all over his body. Lab results: low platelets and high
creatinine level. What is the diagnosis? A. ITP. B. TTP. C. HUS✅✅. D. UTI
Hemolytic-uremic syndrome (HUS) often occurs after a gastrointestinal infection with E coli bacteria
(Escherichia coli O157:H7) = 3 thing happen ( low plattler + low RBC ( anemia ) + high creatinin (kidny
failler )
🍏🍏🍏🍏🍏🍏
🌹thrombocytopenia and uremia in studies and has fever and headache A. HUS B. TTP C. ITP

🌹Child with fever hematuria headache ask dx = TTP
....
🌹A child complains of Headache and petechiea , labs show decreased plt , normal creatinin.There is
attaches pic shows schizocytes. What is the probable diagnosis : TTP
.....
Rx pf TTP? plasma exchange

What is the dx: TTP


.....
🌹Treatment of thrombotic thrombocytopenic purpura = mainly by Plasmapheresis with or without
steroid
...
🌹 Sick pt with fever and coagulation profile all high: A. TTP B. ITP C. HUD D. DIc

🍇.7 years old child day 7 post appendectomy come with fever and bad general .condition but her in
mechanical ventilation . After that do ct contrast . The child bleed from lvlne trachea and wound site ,
What is the Diagnosis ? DIC✅

🌹Dehydration baby what to found= Low pulse pressure or low urine output.
.......
🌹case of child with moderate dehydration what other sign looks at it= Lower urine output ✅

🌹 Child previous URTI came with petechiae and abrasion Every thing is normal except platelets 15000 ask
about ttt ==A. Splenectomy B. Platelets transfusions (ITP CASE) ✅ C. Ivig
….
....
🌹Clear case with GBS and some sever manifest ion ask about mange?
A. immunoglobulin B. plasm exchange
...
🌹Boy post URTI a fews back develops Guillain-Barré syndrome symptoms
(similar to the above scenario) asking about the most like diagnosis:
A. Guillain-Barré syndrome
...
🌹GBS case with involvement of all lower limb and respiratory symptoms : IVIG
........
🌹prognosis of GBS = full recovers 🌹

🍒Adult boy with previous history of infection presents with progressive lower limb weakness? GBS (Gullin
barr syndrom)

Child presented with erythematous pharynx, with cervical lymph nodes and rapid strplysin test negative
and low grade fever with positive EBV . It next step ?
A. Give antibiotics and anti-pyretic ‫ﺎ‬ñ’ª‫ﻜﺘ‬- ‫ﺲ‬G‫ ﻫﺬا ﻓﺎﻳﺮس وﻟ‬B. Give antipyretic and fluids✅
C. Docultureandsensitivity D. Give Acyclovir
No effective antiviral therapy is available for Epstein-Barr virus (EBV) infectious mononucleosis in
immunocompetent persons. Acyclovir and ganciclovir may reduce EBV shedding but are ineffective
clinically. Treatment of immunocompromised patients with EBV lymphoproliferative disease is
controversial
…..
🍇Child came from a visit to Africa. neck stiffness . can not elevate head or limbs : EBV ✅
‫ = ر•ﻂ‬ebv = ‫ﺎ‬¹‫ﻘ‬ñÂ‫ﻌﺪ ﻋﻦ اﻓ‬-‫ا‬.
...
🍓11 years old child had severe diarrhea 3 weeks ago, now the child presented with
bilateral lower limbs weakness and numbness, dx?
A. Poliomyelitis B. Guillain Barre syndrome✅✅✅ C. Muscular dystrophy D. Cerebral palsy
….
🌹Child post-GI infection a few weeks back develops ascending symmetrical muscle weakness associated
with loss of reflexes. (It’s Guillain-Barré) how will you treat this child? A. Steroids B. Immunoglobulins

Child Herpes gingivostomatitis sx (lips, gums, tongue, palate vesicles) and he can’t feed orally. What will
you give him?
A. Antiviral (i'm not sure if they write acyclovir or antiviral)
B. IV fluid and antiviral✅✅ ‫„ ﻓﺎﻳﺮال‬d i ‫ﻤﺎن اﻧ‬È‫ﺔ و‬¤‫ﺎﻟﻔﻢ ﻓﻼزم ﺗﻐﺬ‬- ‫ﻞ‬ÿ‫ﺎ‬¤ ‫ﻘﺪر‬¤ ‫ﺎن‬ÿ‫ﺪ ﻻن ﻣﺎ‬ñ&‫ﻻزم ﻓﻠ‬
C. Abx
......
🌹 A 12 month old child developed painful vesicles around his lips, tongue, gu and hard palate. He is unable
to tolerate orally. What is the most appropriate management?,,
A. oral acyclovir B. IV acyclovir + fluids C. IV abx + fluids

🌹 child with gum bleeding, erythema papules in mouth. Swab showed ( multinucleated giant cell ) on
Tzanck smear , organism = Herpes simplex🌹

🌹White vesicles around mouth and gum with cervical lymphadenopathy, diagnosis? HSV

🌹5 years old presented with rash (picture provided) the rash started as one scratch and then became
generalized over the body. Inguinal and axillary lymph nodes are swollen. What will you give? Acyclovir

🌹 A young child presented with tonsillar ulcer and painful lesion in the back of her mouth and soft palate
, what the dx ? A. Herp angia 🌹

🌹 Child with fever 39 ,sore throats on examination there is white exudate over the tonsils most
appropriate step to reach the diagnosis ? A. Throat culture B. rapid antigen detection test

1Hernia
🌹Neonate , palpable mobile mass non tender , what is it ? A.Hydrocele ✅ B. Varicocele
‫ = ر•ﻂ‬hydro = ‫ﺐ اﻟﻢ‬é‫ﺴ‬±‫ﺪروا ﻣﻌﻨﺎﻫﺎ ﻣﺎء =اﻟﻤﺎء ﻣﺎ‬¹‫ﻫ‬
............
-Child came with testicular swilling wasn't painful positive transillumination normal sensations in preanal
area? A-Testicular torsion. B-Hematocele. C-Hydrocele✅✅. D-Indirect hernia
(hydrocele is a sac filled with fluid that forms around a testicle. )
….
🌹testicular asymmetry and was tender on palpation = testicular torsion = A testicle that's positioned
higher than normal or at an unusual angle. 🌹
….
5 years old child found to have one testes in the scrotum and the other in the ingunal area what to do?
A-orchiopexy ✅. B-orchiectomy. C-wait tell puberty. D-GnRH
(Orchiopexy is a surgery to move an undescended testicle into the scrotum and permanently fix it there.)..
... ..... .. .... ...
🌹Baby with painless scrotal swelling bilaterally , neg transillumination test, Dx?
A. Testicular torsion B. Orchitis C. Idiopathic scrotal edema
‫ ﻻن ﻗﺎﻟﻚ‬painless ‫ﺐ‬ö‫ﺴ‬Ð ‫ﺎرات‬n‫ﻞ اﻻﺧﺘ‬-‫ و‬pain
.....
15 years old with scrotal pain and absent cremasteric reflex diagnosis?
A-varicocele = ‫ ﻫﺬا ﻟﻮ‬cemasteric reflex postive.
B-tusticular torsion✅ C-epididmaitis ’ª‫ﺔ وﻫﺬا اﻟﻮﻟﺪ ﺗﻮە ﺻﻐ‬¹‫ﺴ‬ö‫ﺠﻞ ﻣﻊ اﻧﺘﻘﺎل ﻏﺪوى اﻻﻣﺮاض اﻟﺠ‬¤
D-indirect hernia
Epididymitis:
It's usually caused by a bacterial infection or a sexually transmitted disease (STD). The condition usually
improves with antibiotics.
...... ...... ...... ......
12 years old male with testicular pain (horizontally lying, mildly elevated, no swelling or erythema of
scrotum) what to do next: -surgical exploration ✅‫ اﺳﺘﻜﺸﺎف‬-rehydration
‫ﻋﺔ‬²ø ‫ ﻃﻮل ﻣﻦ اﻻول ﺟﺮاﺣﺔ‬Š‫ﻨﺎ ﻋ‬ñÂ‫ﺤﺼﻞ ﻟﻬﺎ ﻏﺮﻏ‬¤ ‫ﺘﻈﺮ ﻋﺸﺎن‬ö‫ﻟﻦ ﻧ‬

🌹child complain of "unilatral scrotal swelling " dose not transillium whats nexst step ?
Ÿ Discharge pt
Ÿ Give antibiotic
Ÿ Do laprotomy
Ÿ U/s and think about surgery ✅ ‫اﺻﺤﻬﻢ‬

10 month girl , her parents noticed lump in the girl while she crying, disappeared in sleeping
(umbilical hernia) she is otherwise healthy. Management?
1) reassurance ✅ 2 ‫’ة‬ª‫ )ﺗﻮﻫﺎ ﺻﻐ‬mech repair 3) simple repair. 4)laproscopic repair

🌹 18 month baby complain of umbilical hernia become huge with cry = wait and watch✅
…..
🌹Child with swelling, red, tender hernia. Dx? Incarceration (red inflammed tender is incarcerated. )

Baby abounded by his mother as he’s result of out of wedlock pregnancy
Pediatric surgeon asses him he has inguinal hernia decided he need inguinal
hernia repair. Pt need consent What’s appropriate next step?
A-refer the police B- refer to social worker B- you do the consent for the baby. C- call the ethical
committee ✅

Diaphragm hernia what’s the management?
Surgery = if ask about treatment . NG tube = if ask about whats nexst step .
‫ ﻏﺎﻟﺑﺎ اﻟﻛﻔل ﯾوﻟد‬، ‫ وﻧﺎادر ﻧﻼﻗﯾﮫ ﺑﺎﻟﻛﺑﺎر‬congintal ‫ ﻏﺎﻟﺑﺎ ﯾﺟﻲ‬diaphragm hernia ‫ اوﻻ‬، ‫طﯾب ﻻن اﻟﺳؤوال ھذا ﯾﺗﻛرر ﻛﺛﯾر ﺧﻠﯾﻧﺎ ﻧﺷرح ﻧﻘﺎط ﺑﺳﯾطﺔ ﻋﻧﮫ‬
‫ ﻟﻛن ﻗﺑل‬، ‫ ﺑﺎﺧﺗﺻﺎر اﻟﻌﻼج ﻧﺳوي ﻟﮭم ﺟرااﺣﺔ ﻓﻲ اﻗرب وﻗت‬، ‫ طﯾب ھﻲ ﺣﺎﻟﺔ ﺧطﯾؤة وﻣﻣﻛن ﺗؤدي ﻟﻠوﻓﺎة ﻟو ﻣﺎﻟﺣﻘﻧﺎ ﻋﻠﯾﮭﺎ‬، ‫وﻋﻧده ھذا اﻟﻣرض اﻟﺧﻠﻘﻲ‬
‫ ف ﻋﻠﻰ ﺣﺳب اﻟﺳؤوال ﻟﻣﺎ ﯾﻘوﻟك‬، ‫ ﻧودﯾﮫ ﻟﻠﻌﻣﻠﯾﺎت‬stablizantion ‫ ﺑﻌد ﻣﺎ ﻧﻌﻣل ﻟل ﻣرﯾض‬NG tube ‫اﻟﺟراﺣﺔ ﻧﻣﺳك ﺧطوة ب ﺧﻛوة اول ﺷﻲء ﻧﻌﻣﻠﮫ ھو‬
next step .‫اﻟﻌﻼج ﻟﻠﺣﺎﻟﺔ ﯾﺧﺗﻠف ﻋن ﻛﻠﻣﺔ‬
Congintal diaphragm hernia :
Immediately after delivery, the infant is intubated (bag-mask ventilation is avoided). A nasogastric tube is
passed to decompress the stomach and to avoid visceral distention.
How is a diaphragmatic hernia treated? Both congenital and acquired diaphragmatic hernias typically
require urgent surgery. Surgery must be performed to remove the abdominal organs from the chest and
place them back into the abdomen.
In this condition, there's an opening in your baby's diaphragm. This allows some of the organs that should
be found in your child's belly to move up into the chest cavity. This condition can cause serious breathing
problems. It is life-threatening.
. .. . . . . .. .
🌹neonate with hiatus hernia .. ?! NGT🌹
The first step in management is nasogastric tube placement and securing the airway (intubation). The baby
will usually be immediately placed on a ventilator.

🌹baby with diaphragmatic hernia what will you do after stabilization? NGT
….
🌹Baby with diaphragmatic hernia (pic provided) what is your management ?
A. Chest tube. B. Immediate Nasogastric tube C. Immediate hernia repair
‫ ﻣﺛل اﻟﺻورة وااھﮭﮭﮭﮭم ﺧطووووة اذا ﺷوﻓﺗوا ھذا اﻟﻣرض ﻧﻌﻣل ﻟﮫ‬، ‫ ﺗﻛون اﻻﻣﻌﺎء ﻣوﺟودة ﺑﺎﻟﺻدر‬diaphegramtic hernia ‫ﺷوﻓوا‬
.‫ واﺿﺢ ﺑﺎﻻﺷﻌﺔ‬NGT ‫ﺣﺗﻰ ﺗﻼﺣظوا اي ﺻورة ﻓﯾﮭﺎ دﯾﺎﻓﯾﺟراﻣﺗك ھﯾرﻧﯾﺎ ﺑﺟوﺟل ﻻزم ﺗﻼﻗون ن ﺗﯾوب‬NGT
…..
🍓congenetal diaphragmatic hernia, after stabilization, management is: - large NGT ✅

🍓Full term infant of diagnosis as diaphragmatic hernia. Voyageurs support done at delivery... TTT is
A. SURGICAL REPAIR URGENT B. ANTIBIOTICS C. CHEST TUBE
….
🍓Child with bilateral inguinal hernia what is the treatment : A. Herniotomy .✅( bez child) B.
laparoscopic mesh repair

1GIT
🌹Baby 6 weeks, direct bilirubin high. Dx? A- Crlglar najjar B- ABO C- Rh Incompatibility D- Choledochal
cyst ✅

🍇 child w/ gastroenteritis and they asked diagnostic test = Stool antigen✅

🌹Scenario about baby fall down on his RIGHT abdomen from 1 day then develop abdominal pain and
fever= liver contusion
..
🌹Pt baby with abdominal distention and difficulty passing stool , on PR examination empty rectum, but
after finger pulled out there is diarrhea, DX: A. hirschsprung B. intussusception

🍇3 Days neonate passed meconlum and since then he is passing seedy yellow
stool and on breastfeeding he has intestinal obstruction what is the diagnosis? Hirschsprung Disease
...
🌹Sx of hirschsprung disease with plc asking for a dx = Delayed passed meconium = Diagnostic by = Biopsy
Teated by resection and anastomoses, leveling colectomy
.......

🌹 Patient came with gastroentrietis When start normal diet after the ORT( in pedia
A. After 24 h B. After 5 days

🌹Patient complained of Rt groin pain for 2 weeks and exacerbated by exercise, physical examination
showed normal groin and genitalia, next step?
A. CT abdomen B. MRI abdomen C. Us D. Re-evaluate after 2 wks
‫ﺎ ﺣﺼﻞ ﻟﻠﻌﻀﻠﺔ ﺷﺪ‬œ‫ﺎﺿﺔ ف ﻏﺎﻟ‬ñÂ‫ﺪ ﻣﻊ اﻟ‬ñ$‫ﻗﺎﻟﻚ اﻧﻪ ﻳ‬
overstretching the muscles during sport.t'= ‫ﺤﺘﺎج اﺷﻌﺔ‬¤‫ﻣﺎ‬
..
🌹 How to decrease the risk of airbag injury in children younger than 12 years: Restrain to back seat

What’s your management: A. open cyst deerofing. B. albendazole


….
🌹 Child with his parents start to develop fever lethargy, parents mention 2 days before changing his eye
color to yellow to he became ictrus, which type of viral hepatitis he had:
A. Hepatitis A B. Hepatitis B C. Hepatitis C D. Hepatitis D

🌹Child with attacks of severe midline abdominal pain with facial pallor. Poor appetite .. hx 2 = sisters with
migraine abdominal migraine
….
🌹10 years old boy with fever and lethargy and mild abdominal pain + recurrent diarrhea sometimes
bloody + loss of weight and on Examination child looks pale = ulcerative colitis(bloody diarrhia + anemia
go more with UC not crohns )
….
🍓When say this pain from organ ( organic pain) ?
A. Befor sleep time. B. Before awakening✅. C. Two choices I forgot it

🍓A child presented with 3 days of vomiting and stooling. Other information were given. What will be the
electrolyte derangement?
A. Hyperchloremic metabolic acidosis
B. Hypochloremic matabolic alkalosis✅
C. Hyperkalemia matabolic alkalosis
D. Hypokalemic metabolic acidosis

🌹 gastric cancer what is of high diagnostic value? fasting gastrin level 🌹

10 y With diarrhea and sometimes bloodyl
A. Celiac ‫ﻌﻤﻞ اﺳﻬﺎل دم‬¤‫ ﻣﺎ‬B. Cohns c. uc ✅ D. Dysentery
...........
celiac , what food should avoid? Barely
....
🌹12 year with maldigestion. He has greasy foul smelling diarrhea and trouble gaining weight+ feels
lethargic most of the time. BMI: 16, Hg low the diagnostic test=
Antiendomysial antibodies (presint in celiac disease)
‫ﺎك‬¹‫ﻠ‬¹‫ﻞ =ﺳ‬¹‫ﺴ‬±‫ر•ﻂ ﻣﺎ‬
......
...
🌹Caucasian pediatric presenting with failure to thrive, abdominal pain, and distention. He was normal
until he was started on regular food‫ﻨﺎ‬P Û‫ﻠﻤﺔ اﻟ‬-. diagnosis = Celiac disease
‫ﻞ‬ÿ‫ﺎك =ﺳﻠﻖ =اﻻ‬¹‫ﻠ‬¹‫ ر•ﻂ =ﺳ‬.
...
🌹 celiac diseas , indicates activity of the disease= Anti Tissue transglutaminase Ab
c ª ‫ = ﻣﻦ اﺳﻤﻬﺎ ﺟﻠﻮﺗ‬glutin ‫ ال‬Hc ‫ﻣﺮ‬ciliac Ý
Ý c ª ‫ﺎﻟﺠﻠﻮﻟﺘ‬- ‫ﻋﻨﺪﻫﻢ ﻣﺸ•ﻠﺔ‬
......

🌹 13 Years old girl has type 1 dm , her weight 40 kg ( below 50th percentile ) and his hight 150 cm (
below 95th percentile ) , she has no signs of secondary sexual characteristics of puberty, you want to
perform annual screening in clinic for ? (common Q ) ( celiac dis screening) 🌹

.............
🌹16 years old male with 3 years history of diarrhea ( food containing )= Anti endomysial antibodies ✅

Q was high risk illness baby, from whom do you take details ? A.Father B. Mother ✅. C. House
🍒 Signs of obstruction in a child best initial modality of diagnosis = A- US✅. B- chest x-ray C- CT

Pediatric patient with PUD, what’s your management? A. PPI. B. PPI + Amoxacillin + lmidazole ✅✅

4 years old presented with preforated divertcular. laprtomy cleaning done. after days the patient
presented with fever, abdomen was soft, wound was .clean in DRE found anterior bulging and apropreate
management:
A- us drainage (mostly abscess and need drainage not only antibiotics )✅
B- abdminal tract watchout. C- iv antibiotic. D- iv paracetamol
initial management is ABx But the best is drainage + Regarding the size, its all about if the patient needs OR
drainage or US

….
🌷GERD CASE symptom increase at night only what to give her?Bed elevation & PPI🌹

🍓 Pediatric had gastroenteritis and she took metoclopramide that leads to involuntary movement facial
grimace and tongue protruding what to give: A. Diphenhydramine✅. B. Epinephrine. C. Cyproheptadin.
= . ‫ = دوﻟﻔﯾن = ﯾﻌﻣل ﺣرﻛﺎت ﻛﺛﯾرة = ﺗﺷﻧﺟﺎت‬diphen‫داﯾﻔﯾن‬

(Acute dystonic reaction, common side effect of metacroplomide. treated by IV anticholinergic such as
benztropine or diphenhydramine)
.........
Obesity complications if pediatric = hypertenstion
‫ﻢ‬¹‫ﺔ ﺗ•ﻤ‬¹‫ﻋﺔ او ﻋﻤﻠ‬²ø ‫ ﻫﻨﺎ ﻧﺨﺎف وﻻزم ﻳﻨﺤﻒ‬Ûd‫ﺐ ﻣﻀﺎﻋﻔﺎت ؟ اذا ﻟﻘﻴﻨﺎ ﺿﻐﻂ اﻟﺪم ﻋﺎ‬¹‫ﺪﺋﺖ ﺗﺠ‬- ‫ﺎﻻﻃﻔﺎل‬- ‫„ اﻗﻮل اﻧﻪ اﻳﻮا اﻟﺴﻤﻨﺔ‬i‫ ﻣ‬.

🍓 4week old neonate full term Co projectile vomitting- Ex there is mass olive shape Investigation the:
A. SONOGRAPHY ✅ ‫ﻜﻮا‬¤‫ ﻫﻮوﻧﻔﺴﻪ اﻻ‬B. SINCITY C. ENDOSCOPY

&Projectile non-bilious vomiting ,olive like abd mass: dx? A-pyloric stenosis ✅
….
🍓 Olive mass :Pyloric stenosis
......
🍓Case of pyloric stenosis : pylomyotomy. After siblization Yes pylomytomy

🌹Child with projectile non biliary vomiting + mass in epigastric most accurate investigation? US what
treatment?pyloromyotomy
...
🌹 Child came constipation and bilious vomiting for 1 week pain and mild tenderness in left side and
abdominal distension and ancient bowl sounds ,the reason for surgical referral ?
A. absent bowel sounds B. bilious vomiting C. abdominal distension D. constipation

pyloric stenosis what occurs the diagnosis? - failure to thrive✅


….
🌹Pyloric stenosis what is electrolytes abnormalities:-hypochloremia hypokalemic metabolic alkalosis
….
🌹Baby withprojectile vomitingand ugre to drink ( feeling hungry)== pyloric stenosis ✅
….
Child with repetitive vomting and laps shows hypocholrmic hypokalemic metabolic alkilosis and eant the
daignosis Pyloric stenosis
….
🌷hypochloremic hypokalemic metabolic alkalosis = pyloric stenosis🌷

🌹Baby 2 yer Came with apnea -vomiting - dehydration- lethargic-FTT- not gaining wt what is the
problem: A) not relaxed Upper esophageal sphincter B) not relaxed lower esophageal sphincter
C) not relaxed pyloric muscles D) hypertrophy of pyloric m
….
🌹scenario of pyloric stenosis , best diagnostic? A. Barium swallow B. Ct chest or abdomen C. US✅

🌹Child with Metronidazole and Omeprazole for eradication of H.Pylori drug you want to add =
Clarithromycin

liability of diagnosed necrotizing enterocolitis : A.male sex. B. pre- term ✅ C.young age

most important risk factor for Necrotizing enterocolitis: - birth weight < 1.5 kg ✅

Baby preterm 32 weeks , wight 1200g .. symptoms of bloody diarrhea, he has necrotizing enterocolitis ..
what related to that ? A. Male B. Bloody diarrhea C. Weight below 1500g
....
🌹NEC case asking about factor If its absent risk for NEC ?? Full term .

🌹🌷continuous bilious vomiting , Abd destention, Passes meconium after birth, now yellowish thin
diarrhea===.,Colic a-hirschsprung disease b-toxic megacolon c-enterocolitis✅✅
‫ﻂ‬œ‫ﺎﻟﻀ‬- ‫ﻫﺬە اﻋﺮاض‬. == necrotizing enterocolitis 🌷🌹
Necrotizing enterocolitis (NEC) is a medical condition where a portion of the bowel dies. It typically occurs
in newborns that are either premature or otherwise unwell.
….

1Over dose
🌹acetaminophen ingestion they mother says that the bottle had a small amount and he spilled some of it
on his clothes after 4 hours observation in ER he is still stable
A. Discharge home with instruction. B. acetylcysteine. C. Watch for another 4 hours
N.B: Should be serum paracetamol level

Boy came with history of alkali causative ingestion with no symptoms. What is the management:
A. Induce vomiting B. put nasogastric tube. C. neutralization by water. D. close observation

9 years girl ingested two boxes (each 20 tablets) of paracetamol one dya ago after fight with her mother?
Mx: -n-acetlcystine. -gastric lavage. -observation and discharge pt ✅
‫ﻞ‬œ‫ اﻗﻞ ﻣﻦ ار• ـﻊ ﺳﺎﻋﺎت ﻗ‬bd c ‫ ﻟﻮ ﺣﺎت‬، ‫ﺎن ﻣﻤﻜﻦ اﻋﻜﻴﻬﺎ اﻟﺪواء‬È ‫ ﺳﺎﻋﺎت‬٨ ‫”ء‬• c
d I‫ﺎر ﺧﻼص ﺧﻠﺺ وﻗﺘﻬﺎ ﻟﻮ ﺟﺎت اﻗ‬¹‫ اﺧﺘ‬ôd ‫ﻻن اول وﺛﺎ‬
‫ﺖ‬¹‫ﺔ وﻧﻄﻠﻌﻬﺎ ﻟﻠﺒ‬ñ&‫ﻪ ﻟﻠﻌﻼﻣﺎت اﻟﺨﻴ‬œ‫ﺘ‬ö‫”ء ﻧ‬ • c
d ‫ ف اﻫﻢ‬، ‫ﺲ ﻣﻼص ﻓﺎت اﻻوان‬ø ‫ﻞ ﻣﻌﺪة‬¹‫ﺎن ﻣﻤﻜﻦ اﻋﻤﻞ ﻟﻬﺎ ﻏﺴ‬È ‫ ﻟﻼﻣﻌﺎء‬I
d ‫ﻣﺎ اﻟﻤﻌﺪة ﺗﻔ‬
......
🍓Child ingested iron Serum iron 90 Came with nausea and vomiting I think 9 hours ago, Mx?
A. Gastric lavage B. IV deferoxamine✅

🍓Paracetamol poisoning with symptoms ttt? N-acetyl salicylate

child not talking Arabic, before operation he seems very anxious, how should you act as an intern:
auscultate his ches then let him auscultate yours ✅
- paracetamol overdose for a child stage 1-2, one day ago, management:
A- don’t give anti dote cause it will have no effect by now ✅✅
‫ ﻻن ﻏﺳﯾل اﻟﻣﻌدة ﯾﻛون اﻗﺻﻰ ﺷﻲء ارﺑﻊ ﺳﺎﻋﺎت‬، ‫ﺑﺎراﺳﯾﺗﺎﻣول ﻣرت ﻋﻠﯾﮭﺎ ﯾوم ﻛﺎﻣل وھو اﺧذ اﻟﺟرﻋﺔ اﻟزﯾﺎدة ف ﺧﻼص ﻣﺎ ﯾﻧﻔﻊ ﻣﻌﮭم ﻏﺳﯾل ﻣﻌدة‬
‫ ﺳﺎﻋﺎت ﻗﺑل ان‬٨ ‫ ﯾﻧﻔﻊ ﻓﻘط ﻟو ﺧﻼل‬N-acetylcysteine ‫ اﻟﻲ ھو‬antidote ‫ ودواء‬، ‫ ﻻن اﻟﻣﻌدة ﺧﻼص ﺗﻔﺿﻲ ﺑﻌد ارﺑﻊ ﺳﺎﻋﺎت‬، ‫ﻣن ﺗﻧﺎول اﻟﺟرﻋﺔ‬
. ‫ ﺳﺎﻋﺔ ف ﻣﺎ ﻧﻌطﯾﮫ ﺷﻲء‬٢٤ ‫ ف ﺧﻼص ھذا ﻣر ﻋﻠﯾﮫ‬، ‫ﯾﺻل اﻟدواء ﻟﻠﻛﺑد وﯾﺣﺻل ﻟﮫ اﻣﺗﺻﺎص ﻓﻲ اﻟدم‬
..... .....
● Female ingested 20 tablets of acetaminophen then came with N/V and right UQP what to do?
N-acetylcysteine. ✅✅. B.No need for antidote.

6 years old drank a bleaching agent 30 mins ago, how will u manage ‫ﻜﺲ‬ñ‫ﻠﻮر‬È ‫ب‬ú•?
1- Close observation. 2- Cleaning stomach with NGT. 3- drinking water to dilute the bleaching agent
4- endoscopy ✅

🌹 8 years old presented with nausea vomiting diarrhea she was (dehydrated) then they do rapid fluid
correction after that she deteriorate and become comatose. How to replace her fluid deficit?
A. over 48 h B. Over 24 h C. Over 12 h D. Over 6 h ....NB: If with electrolytes correction > 48..

🌹12 years old boy ingested bleach‫ﺾ‬nÍ‫ |ﻠﻊ ﻣﻮاد ﺗ‬by mistake they started IV fluids and he is vitally stable
close observation as i remember Gastric irrigation or something like this I totally forgot but there is other
reasonable which? observation

1Kidney disease
🌹Dialysis requirement (high potassium unresolving to treatment) =Ca-gluconate🌹
..
🌹25 yo male his sister has adult polycystic kidney asking about screening for him
a. anti cysteine b. Ultrasound abd. c. CT abdomen

Dark urine, Proteinuria 2+, HTN. ttt? A- Steroidvv B- Diuretic Frusemide✅. D- Antibiotic
….
child i think 4y with fever 39.9 , dark urine , irritability, o/e tender abdomen with no organomegaly,
investigation( WBC 16.000 , urinalysis show proteinuria +2 and erythrocyte 18 normal is 0-3 or 2 .. what
you will give the child : A- furamide B- ceftriaxone ‫ﺪ‬œõ‫ﻖ اﻟ‬ñÂ‫ﺴﺘﻘﻠﺐ ﻋﻦ ﻃ‬± C- steroid D- salt restriction
….
🌹Young with fever and dark urine has history of constipation , urinalysis: high leukocyte, high
erythrocytes, high protein = pyelonephritis ( not acute glomerulonephritis )

● Pediatric pt c/o of dark urine and fever BP ,Was 125/80 ,Urine analysis ,Protein +2 Diagnosis
?Polynerites✅
….
🌹child came with his parents to ED complaining of fever and dark urine the parents gave hx of the pt have
constipation on ex pt have abdominal tenderness on urinalysis +wbcs +Rbcs +proteins What’s most likely
dx? A. Hypertensive nephropathy B. acute glomerulonephritis C. pyelonephritis D. cystitis
…..
Child with edema in eyelid and ankle with pale and dark urine . What you need in hx ?
A-impetigo in last month ✅B-SCA
PSGN, For GABH ) usually in IO days
But for skin infection it takes 3weeks

🍇An infant presents with a 2 day hx of lethargy, malaise, and dark urine. What is tho Important thing in
hx that you must ask? A. Recurrent UTls B. Recent impetigo contagiosa
.....
🌹 Patient with impetigo when to do nursy what is best answer
A. Drug continues five days. B. Drug continues 3 days. C. Drug continues until dry
….
What’s the diagnosis? Impetigo

...
🍓child developed generalized edema with fever and dark urine, all labs normal except low calcium and
low albumin, what is dx?
A. Minimal change disease✅ B. Mesangial Disease C. Nephropathy D. May be Glomerulonephritis
........
most common type of nephrotic syndrome in children? Minimal change disease
‫ = ر•ﻂ‬minimal = child
....
🌹most common in Adult =Focal segmental glomerulosclerosis
.....
🍒child with puffy eyes, hypoalbuminemia? Minimal change glomerulonephritis🍒
..
- Treatment of patients with minimal change GN? . Steroid

🌹 10 years boy c/o puffy eyes for 1 week with a history of recent infection, no edema no urine changes
and labs completely normal what to give him?strong steroid✅
.....
🌹How to know steroid resistant nephrotic syndrome: A. after 4 wk B. 4 months

🌹child about 12 or 13 y with edema bilateral periorbital edema.diagnose with nephrotic syndrome ..
what’s the management medications : prednisone .
….
🌹 Patient child with nephrotic syndrome, contraindication use: = NSAID
….
🌹 Child e Nephrotic syndrome What medication to avoid in management:
A. aspirin B. amoxicillin C. acetaminophen D. ibuprofen

🍇Child came lethargic ,fever,abd pain hls family say they have history of mldotrlnlan disease ,Labs normal
except ratlculocytes high which of tho following drug contralndlcatad = aspirin .
….
🌹pediatric nephrotic syndrome with edema ( Face edema ) and proteinuria without hematuria =Steroid
trial🌹

🌹 Q about Steriod resistnce nephrotic syndrome :- no remission in 4 w 🌹

🌹 when you said that child have steroid resistant =Induce remission in 4 weeks
….
🍓A case of pediatric nephrotic syndrome with edema (Face edema) and proteinuria without hematuria,
what will you give him? A. Steroid trial✅. B. Diuretic. C. Antibiotics. D. Biopsy

🌹Child with nephrotic (diagnoses given), after ttt given when can you confirm the baby completely
resolve?
A. After normal dipstick 3 Consecutive time. B. After normal depstic 5 Consecutive time. C.edema resolved
...
🌹Another Same Q but asking about effect of ttt when? A- 2 weeks B- 4 weeks
....
child diagnosed as nephrotic syndrome and given steroids, question is what's the best indicator for
remission ?
a-edema resolved b-urine analsyis negative for protien for 3 days✅ c-same as choice 2 but for 5 days

🌹- child with facial and lower limb edma with protinurea on thiasid and other medication i don’t
remember wt best next step?- switch thiaside to fursmide✅🌹

🌹 pediatric nephrotic syndrome with edema ( Face edema ) and proteinuria without hematuria , what
will you give him ? A. Steroid trial✅✅ B. Diuretic. C. Antibiotics D. Biopsy
….
🌹 Child with tonsillitis.. I think the question was about complication.
A. pharyngitis B. bronchitis C. glomerulonephritis
.....
🌹A 16 year old presents with cola colored urine and has fever that followed a rash. Urinalysis was
provided + creatinine was high, Most likely diagnosis is ,, A. acute glomerulonephritis. B. IgA nephropathy
....
🍒🍒Child had UTI one day next develop hematuria' ? Poststreptococcal glomerulonephritis ( Not IgA
nephro) 🍒🍒

:‫ =ﺗﯾﺟﻲ ﺑﺳﺑﻲ اﻧواﻟﻣرﯾض ﻋﻧده اﻣراض ﻣﻧﺎﻋﯾﺔ ﻣﺛل‬IgA nephropathy


as rheumatoid arthritis and systemic lupus erythematosus (SLE)
، ‫ ﻓﻲ أي ﻣﻛﺎن ﺑﺎﻟﺣﺳم ﺳواء اﻟرﺋﺔ‬infection ‫ ﻣن اﺳﻣﮭﺎ ﺗﯾﺟﻲ ﺑﻌد اﺻﺎﺑﺔ اﻟﺟﺳم ب‬Poststreptococcal glomerulonephritis ‫ﺑﯾﻧﻣﺎ‬
strepto . 🍒🍒 ‫ ﻟﻛن ﻧوع اﻟﺑﺗﻛﯾرﯾﺎ ﻣن ﻓﺻﯾﻠﺔ و‬، ‫اﻟﺣﻠق‬
…..
🌹Baby with ankle and knee arthritis, he had UTI prior 2 wk what to give ? A. Azathioprine B. methotrexate

🌹Pediatric complain of arthralgia, SOB, rash in lower extremity , fever past hx of dental procedure before
2 month and sore throat before 2 week . on lap proteinuria and hematuria , On examination murmur ..
what diagnosis A. infective endocarditis. B. post- streptococcal infection C. SLE

🌹Girl 7 years with suprapubic pain No rebound no guarding. Tx? discharge with oral ( Not admitted)
....

🌹Child has abnormal movement and hematuria, had skin infection weeks ago. Now he looks drowsy,
Labs: Creatinine high, BUN high, C3 Low Urine: Protein +, RBC +++ == PSGN ( its come after skin or URTI by
1 to 3 week.)
.....
🌹Child has abdominal mass, aniridia and undescended testicle =wagner disease

🍓URTI after 3 day develop hematuria = lgA nephropathy 🍓

Boy with glomerulonephritis ( hematuria ), after week he developed hemoptysis what is the dx ?
A. Henochschenolein Purpra. B. Good pasture syndrome✅✅. C. Rapid deterotion D. IgANephropathy

🌹Child with glomerulonephritis then develop hemoptysis ? goodpasture syndrome
……
- Child 6 years old present with asymptotic hematuria and every thing normal What u should to do
A)repeated urinanalysis ✅ ‫ﺎرت‬¹‫اﺻﺢ وﺣﺪة ﻣﻦ اﻻﺧﺘ‬. B)cystoscope. C) biopsy

🌹Asymptomatic hematuria in a child Urinanalysis : +ve RBC What to do next ?
A. Repeat urinalysis ✅B. Urine cytology C. Kidney biopsy D. Cystoscopy

🌹Child presents with hematuria. history of constipation for 1 week and presents with hematuria and
urinary incontinence. Upon examination he had tender abdomen, Labs: Low Hb, RBC in urine, Protein in
urine, WBC in urine == UTI (not PSGN)
......
🌹Case of uti and asks what go with urinary tract infection as a risk factor: Fever

🌹 Most common cause of vomiting in pediatrics ? A. Celiac dis B. UTI C. DKA D. cyclic vomiting syndrome
.... ‫ ﯾﻛون ھو اﻻﺻﺢ‬GERD ‫…… ﻟو ﻓﯾﮫ ﺑﺎﻻﺧﺗﯾﺎرات‬.
🌹Most common couse of vomiting in infant? GERD , gastroenteritis

1Skin lesion
🌹 Lower lid swelling, painful erythema not affect vision : A- Cellulitis B- Subperiosteal hematoma

Sole spots

🌹This child with same condition group of children in school .. (i suggest its chicken box)
A. sole spot B. skin scratch C. tonsillitis
..
🌹Pedia present with case of impetigo ask when can return to day care:
A. not contagious return now
B. not contagious return after 3 days
C. contagious return after 5 days
D. contagious return when it gets dry.
‫ اﯾﺎم ﻣن اﻟﻌﻼج ﯾﺑدء‬٣ ‫ ﻣﻌدﯾﺔ وﯾﻌد‬imperiga ‫ ﺳﺎﻋﺔ ﺧﻼص ﻣﺎﯾﻛون‬٢٤ ‫ ﻣﺗﻰ ﻣﺎ ﯾﻛون ﻣﻌدي ؟ ﺑﻌد اﺧذ اﻟدواء ب‬impetiga ‫ﺑﺎﺧﺗﺻﺎر‬
.‫ﯾﺗﻌﺎﻓﻰ اﻟﻣرﯾض‬
…..
🌹Mom comes with her boy who had body rash for 2 days which was eruptions then became vesicular
then crusted with yellow secretions, asks when she can let him go to nursery? What you tell here?
A. The total is 5 days so she can let him go after 3 days B. she can let him go after 5 days from now
C. it is not contagious and she let him go any time
D. the vesicles secretions are contagious and she can let him go to nursery when they get dry
...
Boy has a cat developed itching for a month with red eye and watery with discharge no lymphadenopathy
and general exam normal ? A. Cat scratch. B. Dermatitis allergic✅ C. Conjunctivitis D. Seborrheic

🌹Child with mass on her face = hemangioma 🌹

🌹Baby developed truncal rash= Reassure🌹

-Oral thrush and napkin dermatitis what to give ? A-Oral 7 dys , B-oral and topical 5 dys , ✅ ✅. C-systimc
napkin dermatitis

🌹Neonate presents with erythematous rash all over the body, asking about the management ?
Reassure🌹
....
🌹Child with poor oral intake reveals oral thrush and diaper rash, what to do :
5 days systemic antifungal

..
🌹Child was brought to hospital with air way swelling and skin lesion all over the body the mother stated
that he was in a party at his friends house:=-Food allergy✅
….
🌹Kid with stridor and rash on birthday = allergy food 🌹

🌹 kid at party had (symptoms and sign of allergy) what cause? food allergy

🌹 6 - year - old boy went to sleep with friends home and returned with swelling around his eyes and feet ,
diagnosis? allergy🌹

🌹Child with vascular malformation of lower limb , when to interfere :
A. Pain?✅B. Claudication?
(Pain,pressure sores and ulcers, and possibly bleeding indicates rapid progression of AV malformation
…..
_neonate _ less than one year common area eczema==== * scalp *
....
_Child more than one year area and Adults common area eczema=== - *Flexors* -
....
🌹9 years old boy (long senior..not related to question) what is most part effect for eczema in the age
Goup? Flexor
‫ = ﻛرﺳم ﻓﻛس ﻟﻌﻼج اﻛزﯾﻣﯾﺎ‬flexer = ‫رﺑط = ﻓﯾﻛس‬
......
6 month old long senior..not related to question) what is most part effect for eczema in the age Goup?
A.Flexor B.extensor C.cheeks D. scalp ✅ ‫اﻟﻌﻣر ﯾﻔرق ﺑﺎﻟﻣﻛﺎن ﺳﻛﺎﻟب = ﺳﺗﺔ ﺷﮭور‬...
.......
🍇9 yo with eczema + strong calmly hx of it , where is the commenters site for eczematous plaques ?
Flexors
......
🌹Eczema’s counseling regard bathing? non soap is better
...
Baby with eczema with a positive family history, what are tha common sites involved at this age? - scalp
….
Eczema in 10 y old boy where is most location: ‫ي‬
1) back head. 2) extensor. 3 ) fece and cheek. 4) dorm of foot. 5) Flexors✅✅
….
🌹 Parent’s of 6 month child they come complaining that he has erythema and plaques all manifestation of
eczema > what’s common area for eczema in this age : A. diaper area B. Upper back C. Flexor D. Dorsal of
foot
....

🌹child has eczema topical steroid i think now eczema get worse what next?
A. Medications name. B. Referral to derma. C. Parent Complince medication
....
🌹Most common place of psoriasis in childhood? A-Scalp✅ B-Diapers place C-Planter
(Elbow, knee, scalp)

Calculantion
800 with liver dis. And 800 normal doing test :Test +ve 200 with dis. And +ve 100 normal What is the
sensitivity of this test
A. 25%✅. B. 33%. C. 60%.
Sensitivity = TP/ TP+FN x100
200/800x100
🍒800 with liver dis. And 800 normal doing test :Test +ve 200 with dis. And +ve 100 normal = sensitve =
25% 🍒
....
🌹 Neonatal needs D10 fluids.. what’s the dose/kg? 2 ml/kg🌹
🌹child =5ml/kg for children🌹
🍒🍒Calculate the maintenance IVF for pediatric pt weight 18kg = Answer is : 1400/day , 56/hour === 4-2-
1 role

🍒 baby for primigravida vomiting after each meal? - physiological GER🍒

Dehydrated patient sunken eye decrease skin turger and depressed fontanile percentage?
A) 5. B) 10✅ moderate. C) 15. D) 20 => Sever usually altered mental state and hypotension

🌹calculation hight in a boy ( 178 father hight + 155 mother hight ) = 173 🌹

...
🍇maintenance for child 10kg /24 hour?
1000(10×100=100)
„ „ „
‫ وﻧﺠﻤﻊ‬50 ‫ﺔ „"ب‬n‫ة اﻟﺜﺎﻧ‬Û¿‫ واﻟﻌ‬، ١٠٠ ƒ… ‫ة‬Û¿‫ اول ﻋ‬٢٠ ‫ﻪ ﻣﺜﻼ‬n‫ ﻟﻮ ﻓ‬،‫ة‬Û¿‫ﺲ ﻋ‬õ ‫ﻓﻬﻨﺎ وزﻧﻬﺎ‬, 100 ƒ… ‫ ـﻬﺎ‬iü„‫ اﻟﻮزن ﻧ‬ƒ… ‫ة‬Û¿‫اول ﻋ‬
٢٠ ‫ ـﻬﺎ ب‬iü„‫ة ﻧ‬Û¿‫ ﺛﺎﻟﺚ ﻋ‬٣٠ ‫ وﻟﻮ ﻣﺜﻼ وزﻧﻪ‬، ï „ z ‫ اﻟﻨﺎﺗﺠ‬.
100 first 10 kg , 50 scond 10 , 20 third 10
.....
🍇maintenance 18 kg per hour? 56 minutes

🌹 Child with percentage of burn and asks about the duration of dehydration replacement (deficit should
be replaced over?): A. 24 hrs B. 48 hrs C. 6 hrs D. 16 hrs

Fetal- bradykardia with sinusoidal what causes ?
a-Anemia b-Head compression c-Utroplacental Note: sinusoidal > anemua
🌹Fetal- bradykardia with sinusoidal what causes ?anemua🌹

🌹 Young with vaginal itching and few vaginal bleed?A. foreign body B. sexual abuse
...
🌹Same but with fecal incontinence:
Sexual abuse
....
🌹 I think 3m young pt with ear drum perforation.. what infection cause?
A. adenovirus B. rhinovirus C. rotavirus D. coronavirus

......
Case of chiled his weight is 10 Kg calculate the maintenance : 1000ml ( 10 × 100 = 1000)
🌹 The screening for haematological diseases in children who are low risk , age in months = A. 6 B. 12 C.
24 D. 48

🌹 Fluid replacement for Dehydrated child about 3 questions Maintenance
Answer is: First 10 kg > 10*100. Second 10 kg> 10*50. The rest * 20

🍇 after slnusitis surgery affect sensation in lower eye and upper lip nerve ? Infra-orbital

1Diabetic inspidis
🌹Child after brain trauma developed polyuria . Investigations shows elevated serum osmolality and
decreased urine osmolality .A. Central DI
….
🍓Child had Head trauma that develop polyuria = Central DI

🌹Case about a child with increased water consumption and going to the toilet, not to mention Urine and
serum osmolality, not mention any electrolyte, with no response to water deprivation test, no other
complain:
A. central DI B. peripheral DI C. SIADH
D. medical terminology sound like drinking too much water.🌹
‫ ﻣﺎ اﻗﺪر اﺷﺨﺺ‬DI ‫ |ﺪون ﻣﺎ اﻋﺮف‬Osmolorty
...

🌹child after RTA , admitted in ICU , labs show High serum osmo and low urine osmo Dx ?
A. Central DI ✅ B. Nephrogenic DI C. SIADH

🌹 Case about a child with increased water consumption and going to the toilet, didn’t mention Urine and
serum osmolality, with no response to water deprivation test, no other complain:
A. central DI B. peripheral DI C. SIADH
...
🌹Child came to emergency dehydrated, history of increased thirst and wets diapers a lot after recent
seizure attack, Urine osmolality low. Blood osmolality high. Whats management?
Desmopressin ✅✅🌹(Diapetis insipids)
.. . . . . .
🌹11 years old boy , wetting his bed. All discussion and rewards done. What to do ? Desmopressin
…...
🌹 7 years old boy , mother concern because he still wet his bed despite he is fully toilet trained.. he is a
shamed by that and ask for help (the boy ), what is the best management :
A. Avoid banishment B. Desmopressin C. Bed Alarm and positive reinforcement D. Family centered
approach

🍓8-7 years old child with bedwetting came asking for help with his mom:
A. don’t punish him B. positive re enforcement and alarm✅ C. intranasal desmopressin
........
🌹Bedwetting is considered normal until age: A. 5 B. 6 C. 7 D. 8
....
🌹 Mother came with her son, and concern about recurrent bed wetting, the treatment effect will be on
which muscle: A. Gluteus B. Perianal C. Pelvic Floor
1Ethic
🌹 BA child and her father smoking, your action? -Education to the father

🌹Mom is worried bout her son who has lost his friend 3 months ago and he is talking about killing
himself but he will not actually do it action ?
A-do nothing normal grief B-take detailed hx to prevent suicide ✅
..
🌹 2 Q about child with separation anxiety and school phobia
Q1 A. Focus on unlikely bad things could happen (i
B. punish him when react with sym of anxiety
C. stay at home 3 months and don't go to school
D. wait for supe
….
🌹 Which of the following most Induced Fabricated child illness? Mother

🍇 7 years old child complains of pain that last for 15 minutes and then resolved. It is always in the
morning (school time) what it the advice for the mother.
positive enforcement by forcing on the things the child does rightat school✅

‫ﺎرات‬L‫ اﻻﺧﺘ‬bd c ”• c i
d ‫ﺸﺎرك وﻻ‬M‫ﻼس ﻣﺎ‬Ó‫ اﻟ‬bd ‫ و‬، ‫ﺖ‬è‫ﻄﻪ ﺻﺎر ﻣﺸ‬L‫ﺴ‬ø ‫ﻔ’ە‬- ‫ﻌﺪﻫﺎ‬- ‫ﻪ‬L‫ﻠﺪە ﺛﺎﻧ‬- Û‫ﺸﻪ واﻧﺘﻘﻞ ا‬K‫’ ﻣ•ﺎن اﻟﻤﻌ‬ª‫ﻃﻔﻞ ﻏ‬: ‫ﺆال‬c ‫ ﺳ‬bd
c
bd ‫ﺎن‬È
A. depression B. Hypomania
C. Dysthymia D. 4th option Adjustment disorder
......

Girl 8 yrs old with bald area on head. Mother says she keeps picking her hair due to exams and stress.
(trichotillomania) What medication will u give? a. Olanzapine b. Fluoxetine✅
(No CBT or psychotherapy in options)

🌹2 mo infant is was found dead in his bed , was born with no complications, no medical illness ,mother
denies any family hx of sudden death , what would you ask more about?
A-Social hx✅ B-Family hx C-Mother meds during pregnancy

🌹Radiologist suggest something for pt case.. who should take the pt consent for radiologist opinion?
A. radiologist B. Head nurse C. Medical resident D. Any one of treatment team
....
🌹70 yo man, present with skin change, report revealed malignant melanoma. Who can make the pt
comply to your medical advice:
A. this is a serious issue. B. you are professional so he trusted you C. explain to the pt by medical term.
D. take time to tell him and try to gain his trust.
...
🌹 Pt came with s&s of violence what do:
A. admit call social worker. B. treat and find the cause of her sadness. C. focus Hx and PE.
...
🌹When you should consider during examination of other gender:
A. ask for nurse B. expose necessary area. C. be professional
...
🌹40 yo female, pregnant at 20wks gestation, complain of fever and lower abd pain, she didn’t seek for
medical advice, what is the reason: A. neglect B. fear of physician. C. cost of consult
...
🌹Elderly patient with Cancer, their family ask the doctor to increase the dose of analgesia, Doctor respond
after clarify to them that may cause patient Death .. principle of what:
A. Totality B. subsidity C double effect

🌹 doctor explaining to patient and ask does everything make sense ?
Check understanding of pt

🍓A scenario of a 7 year old girl child brought on account of passage of blood stained stools, bedwetting
after attaining prior continence and complaining of vulval itch. Perineal examination revealed some labial
bruises. What's mosst likely diagnosis?
A. Candidal infection of vulva. B. Bacterial vaginosis. C. Pinworm infection. D. Abuse✅
… 🍏🍏🍏🍏
Psychiatrist asking a patient with depression about the possibility of him having a mental illness, what is he
assessing? insight
...
🍓The scenario talks about how the child always complain about abdominal pain
everytime he goes to school and what to inform the parents/teacher on how to
deal with this school phobia by positive reinforcement?
A. Let him have more TV watch time
B. campliment him on the good work he has done✅ ‫ﺪ‬¹‫ﻌﻤﻞ ﺟ‬- ‫ﻪ اﻟﻤﺪرس اﻧﻪ ﻗﺎم‬¹‫„ ﻋﻠ‬d c ‫ﺠﻠﺲ ﻳﺜ‬¤
c. give him less or moderate work
D. Mark the absence from school days
........
🌹child his parent brought to the ER with 2nd degree burns after accidentally jump to hot water and splash
it, O/E u notice other healed injuries of defiant times. What is the step-by-step guide to Child Protection?
A.Observe, document, report, self care✅
B. Observe, report, referral, follow up.
C. Document, confirm, report, consoling parents.
D. Observe, report, referral, follow up.
‫ﺔ‬¤‫ﺎب اﻟﺤﻤﺎ‬œ‫ﺒﻮرت ﺛﻢ ﻧﺎﺧﺬ ب اﺳ‬ñ‫ﻪ ر‬¹‫ﻨﺎە وﻧﻌﻤﻞ ﻋﻠ‬-‫ ﻛﺘ‬Ûd‫ ﺛﻢ ﻧﺮﺳﻞ ا‬، ‫ ﻻﺣﻈﻨﺎە و*ﺴﺠﻠﻪ‬Ûd‫ ﻧﻜﺘﺐ ا‬ýö‫ ﺛﻢ ﻋﺸﺎن ﻣﺎ ﻧ‬، ‫”ء ﻧﻼﺧﻈﻪ‬ •
d ‫اول‬
‫ﻘﻊ ﺑﻨﻔﺲ اﻟﺨﻄﺎ‬¤‫ﺔ ﻋﺸﺎن ﻣﺎ‬-‫ ﻣﻦ اﻻﺻﺎ‬.
......
,,,,,
Female child with vaginal discharge and fecal incontinence most likely diagnosis:
A-Sexual abuse ✅ B- FB

Pregnant with indication of CS she refused? A- Husband consent ✅ ‫ اﻟﺰوج اﻫﻢ‬ôd N ‫„ ﻫﺬا ر‬i‫ﺣ‬
......
Case of amputation, pt refused. What to do?
A- refer. B- discharge because he refused ✅ No respect wish in choices
.....
Where 12 year old must set in car to avoid injury in accidents?
A-in the front unrestrained ‫ﺠﻠﺲ وراء‬¤ ‫ ﻻ‬B-in the back restrained✅✅‫ﺪ‬¹‫ ﻣﻘ‬C-in child chair in the
back‫” اﻻﻃﻔﺎل ﻻ‬ d ‫ﻛﺮ‬
….
🌹 What are the 4 principles for effective reporting in child abuse?
A-observe, document, report, self protection✅ B-observe, document, referrals, follow up
C- Document, confirm, report, conference parents.
….
7 y Child with vomiting dx as school phobia and GAD ( Generalize anxity disorder) tx? Cognitive
behavioral therapy

Female child with vaginal discharge and fecal incontinence most likely diagnosis:
Sexual abuse.
.... ..... .....
🍒 Female child with sexual ababuse.. site of injury? At 6 o’clock🍒

🍒A mother comes to the clinic she is concerned about her son for the past 3 months after the death of his
friend. He one time said to her “i wish i was dead, but I won't do anything about it”. What to do?Ask
detailed Q about suicidality for prevention.🍒
….
...........
🍒Child Head trauma subarachnoid, hge, periorbital edema, bruises and LOC Father said he found her like
this, what to do? Call child protection 🍒🍒
........
Wife and her husband in the clinic for some test, the doctor finds out that the husband is HIV positive who
should he inform:
A-The wife B-The husband ✅✅ C-ignore the result D-The ethics committee

......
Most sexual harassment‫ ﺗﺤﺮش‬for girl in work place ?
A. Coworker
B. supervisor ✅
C. Clients
D. Owner or boss
🍒🍒Most sexual harassment‫ ﺗﺤﺮش‬for girl in work place = supervisor🍒🍒
.....
Women wearing revealed clothes talking to you in quiet tone with unknown language the sitting nurses
didn’t understand her language?
A-find nurse she understands her language ✅
B- call for security
.....
🌹postive rinforcement = focous on the right things he is doing in the class 🌹
‫ﺔ وﻧﻌﺰزە ؟‬ñ&‫ اﻟﻄﻔﻞ ﻗ‬Šd‫ﻒ ﻧﺨ‬¹‫„ ﻛ‬d c ‫ﻌ‬¤
....
🍓Child with symptoms of ADHD( Attention Deficit Hyperactivity Disorder ) interrupting the class room
and neighbor what is the treatment= dexmethylphenidate
**if the Child 6 years and older is methylphenidate
** If less than 6 years CBT
.....
🌹attention deficit hyperactivity disorder treat by = Methylphenidate 🌹

🌹Young child with bruises in genital area , bleeding = abuse 🌹



🌹 Most Child fabrication symptoms by A. Mother B. Father C. Step mother D. Care provider

🌹Child with poor weight gain mother is anxious and busy with the other children in poor socioeconomic
status: Refer for social worker

🌹Child complaining of abdominal pain on school days to avoid going to school, what to do? Focus on
good things he did ✅
….
..
🌹Patient child with signs and symptoms in (clinical finding ) neurofibromas type1 “Did NOT mention the
diagnosis” and mother side a lot his family with same signs:
A. Family Consultation. B. Referral to genetic test. C. start counseling
….
🌹What is the problem of a 12 old child response to her parent's illness according to her developmental
stage:
A. difficulty to express emotions
B. Refractory behaviour: fussing, violences
C. Imitating her parents symptoms when she plays
D. Does not understand their illnesses nature, cause, diagnosis, treatment
….
🌹Girl 8 yrs old with bald area on head. Mother says she keeps picking her hair due to exams and stress.
(trichotillomania) What medication will u give? A. Olanzapine B. Fluoxetine C. (No CBT or psychotherapy)

🌹First sign of female puberty A. menarche B. Thelarche

🌹 first sign of male puberty?scrotum enlargement✅

You might also like